Download as pdf or txt
Download as pdf or txt
You are on page 1of 66

Drishti IAS Presents...

E conomy (March 2023 — March 2024)

MCQs MCQs
MCQs MCQs
MCQ
MCQs
Multiple
MCQs MCQs
MCQs MCQs
MCQs
MCQs
Choice MCQs MCQ
MCQs
MCQs
MCQs Questions MCQs
MCQs

MCQs and MCQs MCQ


MCQs
MCQs
MCQs
Answers MCQs
MCQs
MCQs

MCQsMCQs MCQs MCQs MCQ


Drishti IAS, 641, Mukherjee Nagar, Drishti IAS, 21 Drishti IAS, Tashkent Marg, Drishti IAS, Tonk Road,
Opp. Signature View Apartment, Pusa Road, Karol Bagh Civil Lines, Prayagraj, Vasundhra Colony,
New Delhi New Delhi - 05 Uttar Pradesh Jaipur, Rajasthan
e-mail: englishsupport@groupdrishti.com, Website: www.drishtiias.com
Contact: Inquiry (English): 8010440440, Inquiry (Hindi): 8750187501
2 PT SPRINT (2024) ECONOMY QUESTIONS AND ANSWERS www.d rish ti IAS.co m

QUESTIONS
1. Consider the following statements with respect to 5. Consider the following statements about NBFC :
‘Government Security (G-Sec)’: 1. NBFCs provide diverse financial services like personal
1. It is a tradable instrument issued by the Central loans, home loans, vehicle loans, gold loans,
Government or the State Governments. microfinance, etc.
2. It is a type of debt instrument issued by the 2. They are part of the payment and settlement system
government to borrow money from the public to and can issue cheques drawn on themselves.
finance its Fiscal Deficit.
3. NBFCs cannot accept demand deposits.
3. They carry practically no risk of default and, hence,
are called risk-free gilt-edged instruments. How many of the statements given above are correct?
How many of the statements given above are correct? A. Only one
A. Only one B. Only two B. Only two
C. All three D. None C. All three
D. None
2. Consider the following statements about Non-Banking
Financial Company (NBFC): 6. Consider the following statements about Palm Oil :
1. An NBFC is a company registered under the 1. India is one of the largest importer of edible oil
Companies Act, 1956, involved in various financial globally, importing approx 96% of its palm oil
activities. requirement.
2. NBFCs include institutions primarily engaged in 2. Indonesia and Malaysia are among the largest global
dealing in immovable property. palm oil producers nations.
How many of statements is/are correct? Which of the statements given above is/are correct?
A. Only 1 A. 1 only
B. Only 2
B. 2 only
C. Both 1 and 2
C. Both 1 and 2
D. Neither 1 nor 2
D. Neither 1 nor 2
3. Regarding the key unemployment indicators, consider
the following statements: 7. Consider the following statements:
1. Labour Force Participation Rate (LFPR) is defined as 1. India is the world's second-largest producer of crude
the percentage of employed persons in the steel in FY2023.
population. 2. Stainless steel is a type of steel alloy that contains
2. Worker Population Ratio (WPR) is defined as the a minimum of 10.5% chromium by mass.
percentage of labour force in the population. Which of the statements given above is/are correct?
3. Unemployment Rate (UR) is defined as the A. 1 only
percentage of persons unemployed in the total B. 2 only
population.
C. Both 1 and 2
How many of the above statements are correct?
D. Neither 1 nor 2
(A) Only one (B) Only two
(C) All three (D) None 8. With reference to Regulatory Sandbox (RS), consider the
following statements:
4. Regarding India's Foreign Direct Investment (FDI)
landscape, consider the following statements: 1. It refers to live testing of new products or services
in a controlled regulatory environment.
1. India experienced a nearly 31% rise in net FDI inflows
during the fiscal year 2023-24. 2. RS in the fin-tech sector is regulated by the Reserve
2. The Netherlands, Singapore, Japan, the USA, and Bank of India (RBI).
Mauritius account for approximately 70% of the Which of the statements given above is/are correct?
total FDI equity inflows into India. A. 1 only
How many ofstatements is/are correct? B. 2 only
A. Only 1 B. Only 2 C. Both 1 and 2
C. Both 1 and 2 D. Neither 1 nor 2 D. Neither 1 nor 2
w w w. d r i s h t i I A S.c om PT SPRINT (2024) ECONOMY QUESTIONS AND ANSWERS 3
9. Consider the following statements: 13. Regarding the Financial Action Task Force (FATF),
1. Recently, the renowned Cuttack Rupa Tarakasi (Silver consider the following statements:
Filigree) has been granted the Geographical 1. It is the global money laundering and terrorist
Indication (GI) tag. financing watchdog set up out of a G-7 meeting of
2. GI Tag guards against imitation and lasts for 15 years developed nations in Paris.
once registered. 2. Recently, the United Arab Emirates (UAE) has been
Which of the statements given above is/are correct? included in the FATF greylist.
A. 1 Only Which of the statements given above is/are correct?
B. 2 Only A. 1 Only B. 2 Only
C. Both 1 and 2 C. Both 1 and 2 D. Neither 1 nor 2
D. Neither 1 nor 2
14. Regarding India’s Spending on Research and
10. Consider the following statements: Development (R&D), consider the following statements:
1. Cooperative banks in India, are registered under the 1. India's expenditure on R&D has increased in recent
Cooperative Societies Act of the State concerned or years as a percentage of GDP.
the Multi-State Cooperative Societies Act, 2002. 2. The Science, Technology, and Innovation Policy 2013
2. The Reserve Bank regulates the banking functions aimed to raise Gross Expenditure on Research and
of Urban Cooperative Banks under the provisions of Development (GERD) to 2% of India’s GDP.
Sections 22 and 23 of the Banking Regulation Act, 3. Most developed countries allocate between 2% and
1949. 4% of their GDPs to R&D.
Which of the statements given above is/are correct? How many of the above statements are correct?
A. 1 Only (A) Only one (B) Only two
B. 2 Only (C) All three (D) None
C. Both 1 and 2
15. Regarding the Employees’ Provident Fund Organisation
D. Neither 1 nor 2
(EPFO), consider the following statements:
11. Which of the following statements about coal use in 1. It is a government organisation that manages
India is not correct? provident fund and pension accounts for the
A. Coal accounts for more than 50% of India's energy workforce engaged in both organised and
needs. unorganised sectors in India.
B. Odisha is the largest producer of lignite coal in India. 2. It is administered by the Ministry of Labour &
C. India's per capita coal consumption is lower than Employment, Government of India.
developed countries. Which of the statements given above is/are correct?
D. Coal imports are unrestricted under India's current A. 1 Only B. 2 Only
import policy.
C. Both 1 and 2 D. Neither 1 nor 2
12. Regarding the Milk Production in India, consider the
16. Regarding the Gross Value Added (GVA), consider the
following statements:
following statements:
1. India is the highest milk producer in the world.
1. GVA measures the value producers add to goods
2. India contributes to around 54% of global milk and services during production.
production.
2. It's calculated by adding the cost of inputs to the
3. The Indian dairy sector is growing by 6% per year. total output.
How many of the above statements are correct? 3. If we deduct the depreciation from GVA we get Net
(A) Only one Value Added (NVA).
(B) Only two How many of the above statements are correct?
(C) All three A. Only one B. Only two
(D) None C. All three D. None of the above
4 PT SPRINT (2024) ECONOMY QUESTIONS AND ANSWERS www.d rish ti IAS.co m

17. Consider the following statements: 21. With reference to the Payment Banks, consider the
Statement-I: The Monetary Policy Committee’s (MPC) following statements:
accommodative stance means the central bank is 1. Payment banks are a specialized type of bank
prepared to expand the money supply to boost introduced by the Reserve Bank of India (RBI) in
economic growth. 2014.
Statement-II: The MPC aims to withdraw accommodation
2. They are designed to promote financial inclusion by
gradually to align inflation with the target while
supporting growth. offering basic banking services to the unbanked and
underbanked population.
Which one of the following is correct in respect of the
above statements? Which of the statements given above is/are correct?
(A) Both Statement-I and Statement-II are correct, and A. 1 only B. 2 only
Statement-II is the correct explanation for C. Both 1 and 2 D. Neither 1 nor 2
Statement-I.
(B) Both Statement-I and Statement-II are correct, and 22. Consider the following:
Statement-II is not the correct explanation for Statement-I: Bilateral Investment Treaties (BITs) are
Statement-I. reciprocal agreements between two countries to
(C) Statement-I is correct, but Statement-II is incorrect. promote and protect foreign private investments in each
(D) Statement-I is incorrect, but Statement-II is correct. other’s territories.
18. Regarding India’s Animation, Visual Effects, Gaming & Statement-II: India’s bilateral treaties have dried up
Comics, and Extended Reality (AVGC-XR) sector, consider since the adoption of the Model Bilateral Investment
the following statements: Treaties (BIT) in 2016.
1. Presently, India shares 7% of the total output of the Which one of the following is correct in respect of the
global AVGC-XR sector. above statements?
2. The National Education Policy (NEP) 2020 fosters a A. Both Statement-I and Statement-II are correct and
favorable ecosystem for cultivating talent in the Statement-II is the correct explanation for
AVGC-XR industry. Statement-I.
Which of the statements given above is/are correct?
B. Both Statement-I and Statement-II are correct and
A. 1 only B. 2 only Statement-II is not the correct explanation for
C. Both 1 and 2 D. Neither 1 nor 2 Statement-I.
19. Regarding RuPay, consider the following statements: C. Statement-I is correct but Statement-II is incorrect
1. It is a payment system and financial services product D. Statement-I is incorrect but Statement-II is correct
developed by the National Payments Corporation
of India (NPCI). 23. Consider the following statements:
2. It is a domestic card payment network that can be Statement-I: India is witnessing 'premature
used at Automated Teller Machines (ATMs), Point deindustrialization'.
of Sale (POS) devices, and e-commerce websites Statement II: The Indian economy is encountering
across India. premature transitions to the services sector, impeding
Which of the statements given above is/are correct? the growth of the manufacturing industry in the country.
A. 1 Only B. 2 Only Which one of the following is correct in respect of the
C. Both 1 and 2 D. Neither 1 nor 2 above statements?
20. Consider the following statements: (A) Both Statement-I and Statement-II are correct, and
1. A recession entails two consecutive quarters of GDP Statement-II is the correct explanation for
decline. Statement-I.
2. A technical recession occurs when a sustained (B) Both Statement-I and Statement-II are correct, and
decline in economic activity persists. Statement-II is not the correct explanation for
Which of the statements given above is/are correct? Statement-I.
A. 1 only B. 2 only (C) Statement-I is correct, but Statement-II is incorrect.
C. Both 1 and 2 D. Neither 1 nor 2 (D) Statement-I is incorrect, but Statement-II is correct.
w w w. d r i s h t i I A S.c om PT SPRINT (2024) ECONOMY QUESTIONS AND ANSWERS 5
24. Consider the following agencies: 28. Regarding the Indian Spice Market, consider the
1. Reserve Bank of India (RBI) following statements:
2. Securities and Exchange Board of India (SEBI) 1. India is the world’s largest spice producer, consumer,
3. Insurance Regulatory and Development Authority and exporter of spices.
of India (IRDAI)
2. India produces about 75 of the 109 varieties which
How many of the above are the key regulatory bodies
are listed by the International Organization for
for Fintech in India ?
Standardization (ISO).
A. Only one
B. Only two Which of the statements given above is/are correct?
C. All three A. 1 only
D. None of the above B. 2 only
25. Consider the following statements: C. Both 1 and 2
1. Article 270 of the Indian Constitution outlines the D. Neither 1 nor 2
distribution of net tax proceeds between the Union
government and the States. 29. Consider the following statements:
2. Currently, the share of States from the divisible pool 1. Bitcoin is an open-source digital currency, that
(vertical devolution) stands at 41% as per the facilitates instant payments without central
recommendation of the 15th Finance Commission.
authority issuance.
3. Cess and surcharge, collected by the Union
government, are not part of the divisible pool and 2. Cryptocurrencies in India fall under the Virtual Digital
therefore not shared with the States. Assets (VDAs) category and are not subject to
How many of the statements given above are correct? taxation.
A. Only One B. Only Two Which of the statements given above is/are correct?
C. All Three D. None A. 1 Only
26. Consider the following statements: B. 2 Only
1. India's Atomic Energy Act, 1962 governs the subject C. Both 1 and 2
of 'nuclear energy', with the Government of India
playing a central role in the development, operation, D. Neither 1 nor 2
and decommissioning of nuclear facilities. 30. With reference to ‘Rural Electrification Corporation
2. The Consolidated Foreign Direct Investment (FDI) Limited (REC Limited)’, consider the following statements:
policy of India allows foreign investment in the
atomic energy sector up to 75%. 1. It is a Miniratna Central Public Sector Enterprise
Which of the statements given above is/are correct? under the Ministry of Commerce.
A. 1 Only B. 2 Only 2. It is registered with the Reserve Bank of India (RBI)
C. Both 1 and 2 D. Neither 1 nor 2 as a Non-Banking Finance Company (NBFC) and
27. Consider the following statements: Infrastructure Financing Company (IFC).
1. India currently is the biggest player in the global 3. It has also been made the nodal agency for certain
space economy with the Indian Space Research States and Union Territories for the Revamped
Organisation (ISRO) ranking at the top of all space Distribution Sector Scheme (RDSS).
agencies globally.
How many of the statements given above is/are not
2. In India, 100% FDI is permitted in the space sector,
correct?
aiming to attract potential investors to Indian space
companies. A. Only one
Which of the statements given above is/are correct? B. Only two
A. 1 Only B. 2 Only C. All three
C. Both 1 and 2 D. Neither 1 nor 2
D. None
6 PT SPRINT (2024) ECONOMY QUESTIONS AND ANSWERS www.d rish ti IAS.co m

31. Consider the following statements: 34. Consider the following statement:
Statement-I: The Monetary Policy Committee’s (MPC) 1. India is the 2nd largest producer of steel in the world
accommodative stance means the central bank is after China.
prepared to expand the money supply to boost 2. In India, West Bengal leads among all steel-producing
economic growth. states, followed by Jharkhand and Chhattisgarh.
Statement-II: The MPC aims to withdraw accommodation Which of the statements given above is/are correct?
gradually to align inflation with the target while
A. 1 only B. 2 only
supporting growth.
C. Both 1 and 2 D. Neither 1 nor 2
Which one of the following is correct in respect of the
above statements? 35. Regarding Reverse Flipping, consider the following
(A) Both Statement-I and Statement-II are correct, and statements:
Statement-II is the correct explanation for 1. It is the process of shifting the domicile of an Indian
Statement-I. company back to India after it had moved its
(B) Both Statement-I and Statement-II are correct, and headquarters overseas.
Statement-II is not the correct explanation for 2. Reverse flipping has become a trend among Indian
Statement-I. startups, especially in the fintech sector, as they plan
(C) Statement-I is correct, but Statement-II is incorrect. for Initial Public Offers (IPOs).
(D) Statement-I is incorrect, but Statement-II is correct. Which of the statements given above is/are correct?
32. Regarding Inflation in India, consider the following A. 1 only B. 2 only
statements: C. Both 1 and 2 D. Neither 1 nor 2
1. Cost-push inflation occurs when the demand for
36. Regarding Section 132 of the Income-Tax Act, 1961,
goods and services exceeds their supply.
consider the following statements:
2. Demand Pull inflation is driven by an increase in the
1. It allows tax authorities to conduct searches and
production costs for goods and services.
seizures without a warrant if there's a "reason to
3. The headline inflation figure is reported through the
believe" that a person has hidden or avoided
Wholesale Price Index (WPI) in India.
income.
Which of the statements given above is/are not correct?
2. The constitutionality of this provision was challenged
A. 1 and 2 only B. 2 and 3 only in the case of Pooran Mal vs Director of Inspection
C. 1 and 3 only D. 1, 2 and 3 (1973).
33. Consider the following statements: Which of the statements given above is/are NOT
Statement-I: The Reserve Bank of India (RBI) has recently correct?
revised guidelines regarding inoperative accounts and A. 1 only B. 2 only
unclaimed deposits, aiming to streamline the C. Both 1 and 2 D. Neither 1 nor 2
classification and activation processes.
Statement-II: The revised instructions apply to all
37. Regarding the Indian Toy Industry, consider the following
Commercial Banks and not to Cooperative Banks. statements:
Which one of the following is correct in respect of the 1. The Indian toy industry exhibited a substantial
above statements? decline in exports in recent years.
(A) Both Statement-I and Statement-II are correct, and 2. A substantial increase in Basic Customs Duty (BCD)
Statement-II is the correct explanation for on toys aims to protect the domestic toy industry.
Statement-I. 3. Ministry of Commerce and Industry mandated
(B) Both Statement-I and Statement-II are correct, and sample testing for each import consignment to
Statement-II is not the correct explanation for prevent the import of substandard toys.
Statement-I. Which of the statements given above is/are correct?
(C) Statement-I is correct, but Statement-II is incorrect. A. 1 and 2 only B. 2 and 3 only
(D) Statement-I is incorrect, but Statement-II is correct. C. 1 and 3 only D. 1, 2 and 3
w w w. d r i s h t i I A S.c om PT SPRINT (2024) ECONOMY QUESTIONS AND ANSWERS 7
38. Consider the following pairs: How many of the statements given above are correct?
GI Tag State A. One Only B. Two Only
1. Adi Kekir Arunachal Pradesh C. All Three D. None
2. Tangail Saree West Bengal
42. Regarding Point of Presence (PoP) Regulations 2023,
3. Kachchhi Kharek Gujarat consider the following statements:
How many of the pairs given above are correctly 1. This regulation makes it easier for people to join the
matched? National Pension System (NPS) by simplifying the
A. 1 only B. 1 and 2 only registration process.
C. 2 and 3 only D. 1, 2 and 3 2. Only Banks can serve as Points of Presence (PoPs)
39. Regarding the exchange rate system, consider the to help people join the NPS.
following statements: 3. Now, people require only single Registration for NPS,
1. In a stabilized arrangement, currency value is instead of multiple registrations as earlier required
determined solely by supply and demand in the and can operate with just one branch with wider
foreign exchange market. digital presence.
2. In a floating exchange rate, the central bank may How many of the statements given above are correct?
intervene in the foreign exchange market to A. Only One
maintain a target range for the currency.
B. Only Two
3. An exchange rate regime is deemed stabilized by
C. All Three
the International Monetary Fund (IMF) when the
exchange rate remains within a 2% band for a D. None
consecutive period of 6 months. 43. With reference to the India’s Civil Aviation Sector,
How many of the above statements are correct? consider the following statements:
(A) Only one (B) Only two 1. India is the second-largest aviation market globally.
(C) All three (D) None 2. The unruly passengers were governed under The
40. Regarding Goods and Services Tax (GST), consider the Aircraft Rules, 1937.
following statements: 3. The Directorate General of Civil Aviation is the
1. It is a comprehensive indirect tax that was principal regulatory body that primarily governs civil
introduced in India through the 106th Constitution aviation in India.
Amendment Act, 2016. How many of the above statements are correct?
2. The GST CounciL, established under Article 279A (1) (A) Only one
Constitution, is tasked with recommending solutions (B) Only two
for issues concerning the implementation of the
(C) All three
Goods and Services Tax (GST) in India.
(D) None
Which of the statements given above is/are correct?
A. 1 only B. 2 only 44. With reference to the ‘Geographical Indication (GI)’,
C. Both 1 and 2 D. Neither 1 nor 2 consider the following statements:
1. GI is governed under the Agreement on Trade-
41. Consider the following statements:
Related Aspects of Intellectual Property Rights
1. Foreign Portfolio Investment (FPI) refers to
(TRIPS) at the World Trade Organisation (WTO).
investments made by foreign individuals,
corporations, and institutions in the financial assets 2. GI certification is only extended to agricultural
of India, such as stocks, bonds, and mutual funds. products and not non-agricultural products.
2. FPI is mainly for the purpose of long-term gains 3. Globally, China leads in GI registrations closely
unlike Foreign Direct Investment (FDI) which followed by Germany.
involves short-term gain. How many of the above statements are not correct?
3. Luxembourg has become the third-largest region for A. Only one B. Only two
FPIs in India, surpassing Mauritius. C. All three D. None
8 PT SPRINT (2024) ECONOMY QUESTIONS AND ANSWERS www.d rish ti IAS.co m

45. Regarding the Foreign Investment in India, consider the 48. Consider the following statements:
following statements: Statement I: Real Estate Investment Trusts (REITs) are
1. Foreign Portfolio Investments (FPIs) involve investing investment vehicles that allow individuals to invest in
in the long term. large-scale, income-producing real estate without having
to directly manage or own the properties.
2. Foreign Direct Investments (FDIs) involve investing
in financial assets in short to medium-term. Statement II: They pool capital from multiple investors
to invest in a diversified portfolio of real estate assets,
3. Government’s prior approval is mandatory for FDIs
which may include residential or commercial properties,
from countries sharing land border with India.
shopping centers, office buildings, hotels etc.
Which of the statements given above is/are correct? Which one of the following is correct in respect of the
A. 1 and 2 only above statements?
B. 3 only A. Both Statement-I and Statement-II are correct and
C. 1 and 3 only Statement-II is the correct explanation for
Statement-I
D. 1, 2 and 3
B. Both Statement-I and Statement-II are correct and
46. Regarding Geographical Indication (GI), consider the Statement-II is not the correct explanation for
following statements: Statement-I
1. It is a sign used on products that have a specific C. Statement-I is correct but Statement-II is incorrect
geographical origin and possess qualities or a D. Statement-I is incorrect but Statement- II is correct
reputation that are due to that origin.
49. With reference to the 15th Finance Commission,
2. It is a powerful tool for protecting traditional consider the following statements:
knowledge, culture and can boost socio-economic
1. The 15th Finance Commission proposed maintaining
development.
the states' share in central taxes at 42% for the
3. GI is governed under the Agreement on TRIPS at the 2021-26 period.
World Trade Organisation (WTO). 2. Population (2011)' and 'Tax and fiscal efforts' were
How many of the statements given above are correct? introduced by the 15th Finance Commission.
A. Only One 3. The Commission recommended that the Centre aims
B. Only Two to limit its fiscal deficit to 4% of GDP by 2025-26.
C. All Three How many of the above statements are correct?
D. None (A) Only one
(B) Only two
47. Consider the following statements with respect to Terms
(C) All three
of Trade (ToT):
(D) None
1. ToT for Indian agriculture refers to the movement
in prices of farm commodities relative to non-farm 50. With reference to Periodic Labour Force Survey (PLFS),
goods and services. consider the following statements:

2. A rise in the terms of trade implies better purchasing 1. It is a survey conducted by the National Statistical
Organisation (NSO) under the NITI Aayog.
power for the agricultural sector in terms of
industrial goods. 2. It was launched in April 2017 by NSO.

Which of the statements given above is/are correct? 3. Its objective is to estimate employment and
unemployment indicators in both ‘Usual Status’ and
A. 1 only
CWS in both rural and urban areas annually.
B. 2 only Which of the statements given above is/are correct?
C. Both 1 and 2 A. 2 and 3 only B. 2 only
D. Neither 1 nor 2 C. 3 only D. 1, 2 and 3 only
w w w. d r i s h t i I A S.c om PT SPRINT (2024) ECONOMY QUESTIONS AND ANSWERS 9
51. Regarding the Global Stocktake Draft, consider the 55. Regarding the Social Stock Exchange, consider the
following statements: following statements:
1. The Global Stocktake (GST) is a periodic review 1. Social stock exchanges are dividend-paying
mechanism established under the Paris Agreement companies created and designed to address a social
in 2015. problem.
2. The draft of GST text that was unveiled at COP28 2. It works under the market regulator Security and
proposes ten steps to keep the global temperature Exchange Board of India (SEBI).
rise within the ambit of 2.5 degrees Celsius. 3. It raises funds through Zero Coupon Zero Principal
Which of the statements given above is/are correct? Bonds (ZCZP).
A. 1 Only B. 2 Only Which of the statements given above is/are correct?
C. Both 1 and 2 D. Neither 1 nor 2 A. 1 and 2 only B. 1 and 3 only
52. With reference to Countervailing Duty (CVD), consider C. 2 and 3 only D. 1, 2 and 3
the following statements: 56. Regarding Special Purpose Vehicle (SPV), consider the
1. CVD are tariffs levied on exported goods. following statements:
2. World Trade Organization (WTO) permits the 1. SPV is often referred to as a bankruptcy-remote
imposition of CVD by its member countries. entity.
3. Directorate General of Trade Remedies (DGTR) is 2. Venture capitalists use SPVs to gather funds and
the single national authority for administering invest in a startup.
countervailing duties. Which of the statements given above is/are correct?
Which of the statements given above is/are correct? A. 1 Only B. 2 Only
A. 1 and 2 only B. 1 and 3 only C. Both 1 and 2 D. Neither 1 nor 2
C. 2 and 3 only D. 1, 2 and 3
57. Consider the following statements regarding Scheme
53. Regarding Common Reporting Standards (CRS), consider for Remission of Duties and Taxes on Exported Products
the following statements: (RoDTEP):
1. It calls on jurisdictions to obtain information from 1. RoDTEP covers taxes, duties, and levies at the
their financial institutions and automatically central, state, and local levels, which are not
exchange that information with other jurisdictions refunded through any other existing mechanisms.
on an annual basis. 2. The RoDTEP Committee operates within the
2. World Bank’s Automatic Exchange of Information Department of Revenue.
(AEOI) is also a part of the CRS framework. Which of the statements given above is/are correct?
Which of the statements given above is/are not correct? A. 1 Only B. 2 Only
A. 1 Only C. Both 1 and 2 D. Neither 1 nor 2
B. 2 Only
58. Which of the following statements is/are true about the
C. Both 1 and 2 different causes of inflation?
D. Neither 1 nor 2 1. Demand-pull inflation occurs when there is an
54. Regarding the Telecom Industry in India, consider the increase in aggregate demand.
following statements: 2. Cost-push inflation occurs when there is an increase
1. The Telecom Industry of India is the fourth largest in the cost of production.
in the world. 3. Structural inflation occurs when there is a mismatch
2. It is also the 4th largest sector in terms of Foreign between demand and supply.
Direct Investment (FDI) inflows. 4. Built-In inflation is often described as a feedback
3. India has an overall teledensity of 100%. loop between wages and prices.
How many of the statements given above are correct? Select the correct answer using the code given below:
A. Only one B. Only two (a) 1 and 2 only (b) 2 and 3 only
C. All three D. None (c) 1, 2 and 3 only (d) 1, 2, 3 and 4
10 PT SPRINT (2024) ECONOMY QUESTIONS AND ANSWERS www.d rish ti IAS.co m

59. With reference to the Alternative Investment Fund (AIF), 62. With reference to Universal Basic Income, consider the
consider the following statements: following statements:
1. AIF refers to a fund established or formed in India, A. It is a social welfare proposal in which all the
serving as a privately pooled investment mechanism. beneficiaries regularly receive a guaranteed
2. These investment vehicles adhere to the SEBI income in the form of an unconditional transfer
(Alternative Investment Funds) Regulations, 2012. payment.
3. Category III AIFs are those that invest in startups, B. It is specifically mentioned in the Indian constitution
early-stage ventures, social initiatives, Small and under the Directive Principle of State Policy (DPSP).
Medium Enterprises (SMEs), infrastructure, etc. Which of the statements given above is/are correct?
How many of the statements given above are correct? (a) 1 only
A. Only one (b) 2 only
B. Only two (c) Both 1 and 2
C. All three (d) Neither 1 nor 2
D. None
63. With reference to Worker and Labour Productivity,
60. With reference to ‘Insurance For All by 2047’, consider consider the following :
the following statements:
1. Productivity is measured in terms of the labor-
1. The Life Insurance Corporation (LIC) of India in output ratio or change in Net Domestic Product per
alignment with the 'Insurance For All by 2047' worker in each sector where working hours are
initiative, is introducing tailored products for rural assumed to be 8 hours per day.
areas.
2. The difference between Worker Productivity and
2. The focus is on extending insurance coverage to
Labor Productivity is that the ‘work’ in worker
maximum rural masses, aligning with the vision of
productivity describes mental activities while the
making India a developed nation by 2047.
‘work’ in labour productivity is mostly associated
3. LIC acknowledges the Insurance Regulatory and with manual activities.
Development Authority of India (IRDAI) proposed
How many of the statements given above are correct?
'Bima Vistar,' for realising the Vision of 2047.
A. Only one
How many of the statements given above are not
correct? B. Only two
A. Only one C. Both
B. Only two D. None
C. All three 64. Regarding the Non-Banking Finance Companies (NBFCs)
D. None consider the following statements:
61. Regarding the Financial Stability Board (FSB), consider 1. NBFCs are restricted from accepting demand
the following statements: deposits from the public.
1. FSB was established in 2005 under the aegis of the 2. NBFCs form an important part of the payment and
Organisation for Economic Co-operation and settlement system.
Development (OECD). 3. NBFCs primarily finance their operations through a
2. India is an active Member of the FSB. mix of market borrowing and bank loans.
Which of the statements given above is/are correct? How many of the statements given above are correct?
A. 1 Only A. Only one
B. 2 Only B. Only two
C. Both 1 and 2 C. All three
D. Neither 1 nor 2 D. None
w w w. d r i s h t i I A S.c om PT SPRINT (2024) ECONOMY QUESTIONS AND ANSWERS 11
65. Which of the following statements about silk production 69. Regarding the Insolvency and Bankruptcy Code (IBC)
and sericulture in India is/are correct? 2016, consider the following statements:
1. Telangana is the leading state in India's silk 1. IBC aims to provide a time-bound and creditor-
production, contributing 32% to the total production driven process for insolvency resolution and to
in the fiscal year 2021-22. improve the credit culture and business environment
2. India is the world's largest producer of raw silk, in the country.
surpassing China in the fiscal year 2020-21. 2. The Code is quite different from the earlier
3. The Central Silk Board (CSB) is responsible for the resolution systems as it shifts the responsibility to
overall development and promotion of the the creditor to initiate the insolvency resolution
sericulture and silk industry in India. process against the corporate debtor.
Provide correct using the options given below: Which of the statements given above is/are correct?
A. Only statement 1 is correct. A. 1 Only B. 2 Only
B. Statements 1 and 2 are correct. C. Both 1 and 2 D. Neither 1 nor 2
C. Statements 2 and 3 are correct.
70. With reference to ‘De-Dollarisation’, consider the
D. All three statements are correct. following statements :
66. Consider the following statements: 1. De-dollarisation refers to a deliberate or unintentional
1. A tax haven is a country that offers foreigners very process undertaken by a country or region to reduce
low tax liability in a politically and economically reliance on the US dollar in its financial system or
stable environment. economy.
2. An offshore company is incorporated in a jurisdiction 2. This can involve various measures aimed at
other than its home country. decreasing the use of the dollar in transactions,
Which of the statements above is/are correct? reserves, trade, or as a standard for pricing goods
and services.
A. 1 only B. 2 only
3. Strategies for de-dollarisation can include diversifying
C. Both 1 and 2 D. Neither 1 nor 2
currency reserves, promoting the use of alternative
67. With reference to Investor Risk Reduction Access (IRRA), currencies in trade agreements, establishing
consider the following statements: currency swap agreements among others.
1. It was launched in order to provide a ‘safety net’ for How many of the statements given above are not
investors in case of technical glitches faced by a correct?
trading member or a stockbroker. A. Only one B. Only two
2. It can also be used for taking fresh positions or orders C. All three D. None
and canceling the pending orders.
3. IRRA has been developed by the Reserve Bank of
71. With reference to Fraud Loan Apps and Digital Lending,
India. consider the following statements:

Which of the statements given above is/are correct? 1. Fake loan apps are unauthorized and illegal digital
lending platforms that offer loans, from as little as
A. 1 and 2 only B. 1 only
Rs.1,000 to Rs.1 lakh targeting low-income and
C. 3 only D. 1, 2 and 3 only financially untrained individuals.
68. Consider the following statements: 2. Fraud Loan Apps are completely regulated by the
1. Gondwana coal makes upto 98% of the total reserves Reserve Bank of India and no other institution is
and 99% of the production of coal in India. authorized to regulate it.
2. Gondwana coal forms India’s metallurgical grade as 3. The RBI has not released any guidelines until now
well as superior quality coal. to regulate Digital Lending.
Which of the statements given above is/are correct? How many of the statements given above are correct?
A. 1 only B. 2 only A. Only one B. Only two
C. Both 1 and 2 D. Neither 1 nor 2 C. All three D. None
12 PT SPRINT (2024) ECONOMY QUESTIONS AND ANSWERS www.d rish ti IAS.co m

72. With reference to Sovereign Gold Bonds (SGB) , consider 75. With reference to the Fair Price Shop (FPS), consider the
the following statements: following statements:
1. These Bonds are issued by State Bank of India on 1. FPS is a government-run or government-regulated
behalf of Government of India. retail outlet or store in India.
2. Persons resident in India as defined under Foreign 2. The e-PoS devices have been integrated with
Exchange Management Act, 1999 are eligible to electronic weighing machines to ensure the
invest in SGB. beneficiaries get the correct amount of ration.
Which of the statements given above is/are correct? 3. These shops are aimed at ensuring food security and
reducing the economic burden on backward classes.
A. 1 Only
How many of the statements given above are not
B. 2 Only
correct?
C. Both 1 and 2
A. Only one
D. Neither 1 nor 2
B. Only two
73. Consider the following statements regarding C. All three
International Financial Services Centres (IFSC): D. None
1. An IFSC is a financial centre that caters to customers
76. Consider the following statements:
outside the jurisdiction of the domestic economy.
1. A wilful defaulter means a borrower or a guarantor
2. The IFSC in India is regulated by the International
who has committed wilful default and the
Financial Services Centres Authority (IFSCA), a
outstanding amount is Rs 25 lakh and above.
statutory authority that was established under the
2. A large defaulter refers to a borrower with an
IFSC Act, 2019.
outstanding balance of Rs 1 crore or more, whose
Which of the statements given above is/are correct? account has been categorized as doubtful or a loss.
A. 1 only Which of the statements given above is/are correct?
B. 2 only A. 1 only
C. Both 1 and 2 B. 2 only
D. Neither 1 nor 2 C. Both 1 and 2
74. Consider the following statements: D. Neither 1 nor 2
Statement-I: Under the FDI policy, e-commerce 77. Regarding the Geographical Indication (GI) tag consider
comprises products, both digital and physical, and the following statements:
services traded on digital and electronic networks. 1. Geographical indications are recognized as a part of
Statement-II: Business-to-consumer (B2C) is the retail intellectual property rights (IPRs) under the Paris
part of e-commerce on the Internet, that is, the sale of Convention and Trade-Related Aspects of Intellectual
goods and/or services to the end consumer through Property Rights (TRIPS) Agreement.
digital means. 2. India, as a World Trade Organisation (WTO) member,
Which one of the following is correct in respect of the implemented the Geographical Indications of Goods
above statements? Act, 1999.
(A) Both Statement-I and Statement-II are correct, and 3. Arunachal Pradesh has recently received the
Statement-II is the correct explanation for Geographical Indication (GI) tag for Arunachal Yak
Statement-I. Churpi, Khaw Tai (Khamti rice), and Tangsa Textiles.
(B) Both Statement-I and Statement-II are correct, and Which of the statements given above is/are correct?
Statement-II is not the correct explanation for (a) 1 and 2 only
Statement-I. (b) 2 only
(C) Statement-I is correct, but Statement-II is incorrect. (c) 3 only
(D) Statement-I is incorrect, but Statement-II is correct. (d) 1, 2 and 3 only
w w w. d r i s h t i I A S.c om PT SPRINT (2024) ECONOMY QUESTIONS AND ANSWERS 13
78. Regarding Consumer Food Price Inflation (CFPI), 82. With reference to Minimum Support Price in India,
consider the following statements: consider the following statements:
1. CFPI is a sub-component of the broader Wholesale 1. The Commission for Agricultural Costs and Prices
Price Index (WPI). (CACP) approves MSPs for various crops in India.
2. It tracks the price changes of a specific basket of 2. CACP recommends MSPs for 22 mandated crops and
food items that are commonly consumed by fair and remunerative price (FRP) for sugarcane.
households. Which of the statements given above is/are correct?
Which of the statements given above is/are correct? A. 1 only B. 2 only
(A) 1 only (B) 2 only C. Both 1 and 2 D. Neither 1 nor 2
(C) Both 1 and 2 (D) Neither 1 nor 2
83. With reference to the Multilateral Development Banks
79. With reference to the Navratna companies, consider the (MDBs), consider the following statements:
following statements: 1. They are international institutions comprising both
1. As of 2023, India has 16 Navratna Companies. developed and developing countries.
2. To attain Navratna status, a firm must initially hold 2. Developed countries contribute to MDB lending,
the Miniratna designation . while developing nations typically borrow from
Which of the statements given above is/are correct? them for development projects.
A. 1 only B. 2 only Which of the statements given above is/are correct?
C. Both 1 and 2 D. Neither 1 nor 2 A. 1 only B. 2 only
C. Both 1 and 2 D. Neither 1 nor 2
80. With reference to Angel Tax, consider the following
statements: 84. Consider the following statements regarding Capital
1. Angel tax is an income tax levied at the rate of 50% Expenditure:
when an unlisted company issue shares to an 1. Expenses related to acquiring, improving, or
investor at a price higher than its fair market value. extending long-term assets or investments expected
2. Angel tax is applicable only to investments made by to benefit for more than one financial year.
resident investors. 2. It includes -acquiring new machinery, Routine
3. Recently, the Finance Ministry exempted investors machinery maintenance, employee salaries,
from 21 countries from the angel tax levy for non- advertising costs, utility bills.
resident investments in Indian start-ups. 3. It is subject to depreciation or amortization, leading
Which of the statements given above is/are correct? to a delayed tax impact and often lower taxable
(a) 1 and 2 only (b) 3 only income in the year of purchase.
(c) 2 and 3 only (d) 1, 2 and 3 only How many of the above statements is/are not correct?
A. Only one B. Only two
81. With reference to the Vizhinjam International Seaport
C. All three D. None
Project, consider the following statements:
1. The Vizhinjam International Transhipment 85. With reference to the Organisation for Economic Co-
Deepwater Multipurpose Seaport is an ambitious operation and Development (OECD), consider the
project taken up by the Government of Kerala. following statements:
2. The port is currently being developed with a Public 1. Most OECD members are high-income economies
Private Partnership with a component structured with a very high Human Development Index (HDI)
on a design, build, finance, operate, and transfer and are regarded as developed countries.
("DBFOT") basis. 2. India is not a member, but a key economic partner
3. It is positioned to compete with global transshipment of the organisation.
hubs like Colombo, Singapore, and Dubai, reducing 3. A report named “Government at a Glance” is
the cost of container movement. released by OECD.
How many of the above statements are correct? How many of the statements are correct?
(A) Only one (B) Only two A. Only one B. Only two
(C) All three (D) None C. All three D. None
14 PT SPRINT (2024) ECONOMY QUESTIONS AND ANSWERS www.d rish ti IAS.co m

86. The term Bidenomics was sometimes in the news in 89. With reference to the the Directorate of Revenue
relation to economic policies. Which of the following Intelligence (DRI), consider the following statements:
statements correctly reflects its meaning? 1. DRI serves as India's premier anti-smuggling agency,
A. It is a set of policies that emphasizes tax cuts, operating under the Central Board of Indirect Taxes
deregulation, domestic spending cuts and inflation & Customs, Ministry of Finance.
reduction.
2. It is responsible for detecting and preventing various
B. It is a set of policies that emphasize relief measures,
forms of smuggling, including illicit wildlife trade.
investments in infrastructure, and strengthening the
safety net, funded by tax increases on higher-income Which of the statmenets given above is/are not correct?
individuals and corporations. A. 1 only
C. It is a set of policies that emphasizes trade B. 2 only
protectionism, immigration restriction, and
C. Both 1 and 2
isolationism from global affairs.
D. Neither 1 nor 2
D. It is a set of policies that emphasize environmental
sustainability, social justice, and human rights, 90. Which of the following statements is true about Fiscal
funded by a global wealth tax. Deficit ?
87. With reference to the Global Minimum Tax (GMT), 1. Fiscal deficit is the difference between the
consider the following statements: government's total expenditure and its total revenue
1. A GMT applies a standard minimum tax rate to a (including borrowings).
defined corporate income base worldwide. 2. A high fiscal deficit can lead to inflation, appreciation
2. The framework of GMT aims to encourage nations of the currency and an increase in the debt burden.
from tax competition through lower tax rates that A. 1 only
result in corporate profit shifting and tax base
B. 2 only
erosion.
3. In 2021, a group of 136 countries, including India, C. Both 1 and 2
set a minimum global tax rate of 15% for MNCs and D. Neither 1 nor 2
sought to make it harder for them to avoid taxation.
91. The term 'Impossible Trinity' has been in the news
How many of the statements given above are correct?
recently, and it pertains to which of the following
A. Only one
economic aspects:
B. Only two
a. This concept suggests that an economy cannot
C. All three
simultaneously have its own monetary policy, a fixed
D. None exchange rate, and unrestricted capital movement
88. Regarding the Monetary Policy Committee (MPC), across its borders.
consider the following statements: b. This concept means that an economy can't have its
1. It is provided under Section 45ZB of the amended own fiscal policy, keep its currency's value fixed, and
RBI Act, 1934. let money move freely in and out of the country all
2. It determines the policy repo rate required to at once.
achieve the inflation target. c. This concept means that an economy cannot
3. The MPC functions under the chairmanship of the simultaneously manage a current account deficit,
Union Finance Minister. maintain a fixed exchange rate, and permit
How many of the statements given above are correct? unrestricted capital flows.
A. Only one d. This concept implies that an economy cannot handle
B. Only two a current account deficit, maintain a fixed interest
C. All three rate, and allow unrestricted capital flows all at the
D. None same time.
w w w. d r i s h t i I A S.c om PT SPRINT (2024) ECONOMY QUESTIONS AND ANSWERS 15
92. With reference to Central Bank Digital Currencies 95. With reference to the Incremental Capital Output Ratio
(CBDCs), consider the following statements: (ICOR), consider the following statements:
1. Unlike cryptocurrencies, which lack regulatory 1. ICOR has evolved from the Harrod-Domar Growth
oversight, CBDCs are official legal tenders issued and Theory and examines the relationship between fresh
supported by a central bank. investments and economic growth.
2. It differs from fiat currency but is exchangeable one- 2. A lower ICOR signifies greater efficiency and
to-one with it. productive use of capital.
3. El Salvador made history in 2020 by becoming the Which of the statements given above is/are correct?
first economy to introduce its nationwide CBDC, A. 1 only
known as the Sand Dollar. B. 2 only
How many of the above given statements are correct? C. Both 1 and 2
a. Only one D. Neither 1 nor 2
b. Only two
96. Gresham's Law, sometimes in news, is related to which
c. Only three
of the following?
d. None of the above
A. Fiscal Policy
93. Consider the following statements: B. Monetary Policy
1. Hitachi Payment Services is the company that C. Currency Exchange Rate
launched India's first UPI-ATM. D. Trade Agreements
2. The UPI-ATM is a White Label ATM (WLA) developed
97. Consider the following states-
in collaboration with the Reserve Bank of India (RBI).
1. Kerala
3. The ATM was designed to enable "seamless cash
withdrawals" and eliminate the need to carry 2. Tamil Nadu
physical ATM cards. 3. Tripura
How many of the statements given above are correct? 4. West Bengal
A. Only one How many of the above are generally rubber producing
B. Only two states?
C. All three a. Only one state
D. None b. Only two states
c. Only three states
94. Consider the following statements with respect to
d. All of the above
‘Global Fintech Fest (GFF)’:
1. It has been organized by the National Payments 98. With reference to the Insurance Regulatory and
Corporation of India (NPCI), Payments Council of Development Authority of India (IRDAI), consider the
India (PCI), and Fintech Convergence Council (FCC). following statements:
2. Its aim is to provide a singular platform for fintech 1. IRDAI is a non statutory body.
leaders to foster collaborations and develop a 2. It comes under the jurisdiction of Ministry of
blueprint for the future of the industry. Commerce.
3. The theme of GFF’ 2023 is “Global Collaboration for 3. It regulates and sees to the development of the
a Responsible Financial Ecosystem”. insurance industry.
How many of the above statements are correct? How many of the statements given above are correct?
(A) Only one A. Only one
(B) Only two B. Only two
(C) All three C. All three
(D) None D. None
16 PT SPRINT (2024) ECONOMY QUESTIONS AND ANSWERS www.d rish ti IAS.co m

99. Consider the following statements: 103. With reference to the Foreign Direct Investment (FDI),
1. India's Gross Domestic Product (GDP) is calculated consider the following statements:
using three distinct methods. 1. It is a business strategy in which a domestic firm
2. The factor cost method indicates how different areas expands its operations to a foreign country.
of the economy are performing, such as trade, 2. FDI can take various forms, such as acquiring shares,
investments etc.
establishing a subsidiary or a joint venture, or
3. The expenditure-based method assesses the providing loans or technology transfers.
performance of different industries.
Which of the statements given above is/are not correct?
How many of the statements given above are correct?
A. 1 only
A. Only one B. Only two
C. All three D. None B. 2 only
C. Both 1 and 2
100. Which of the following statements is/are correct?
D. Neither 1 nor 2
1. In 2021-2022, India was the second largest producer
of milk in the world. 104. Consider the following statements regarding Gross
2. Uttar Pradesh is the largest producer of milk in India. Domestic Product (GDP):
3. Increasing temperatures may reduce milk production 1. Nominal GDP is adjusted for inflation and is
in India’s arid and semi-arid regions. considered a more precise representation of a
Select the correct answer using the code given below: country's economic progress.
(a) 2 and 3 only (b) 1 and 2 only 2. The formula for calculating GDP using the
(c) 3 only (d) 1, 2 and 3 expenditure method is GDP = Real GDP (GDP at
101. Consider the following statements: constant prices) - Taxes + Subsidies.
1. Indian pharmaceutical industry is the 3rd largest 3. GDP calculated based on the income method is
pharmaceutical industry in the world by volume. known as GDI, which stands for Gross Domestic
2. Recently, the governmnet has launched initiatives Investment.
namely National Policy on Research and How many of the statements given above are correct?
Development and Innovation in Pharma-MedTech A. Only one
Sector in India and Scheme for Promotion of
B. Only two
Research and Innovation in Pharma MedTech Sector
(PRIP). C. All three
Which of the statements given above is/are correct? D. None
A. 1 only 105. With reference to the Cash Reserve Ratio (CRR), consider
B. 2 only the following statements:
C. Both 1 and 2 1. It is the percentage of cash required to be kept in
D. Neither 1 nor 2 reserves as against the bank's total deposits.
102. Q. With reference to the Surety Bonds, consider the 2. All banks in India, except Regional Rural Banks (RRBs)
following statements: and Local Area Banks (LABs), have to deposit the
1. Surety bond is provided by the insurance company CRR money to RBI.
on behalf of the contractor to the entity that is 3. Banks can’t lend the CRR money to corporates or
awarding the project. individual borrowers.
2. A surety bond can be defined in its simplest form as How many of the above statements are not correct?
a written agreement to guarantee compliance,
payment, or performance of an act. A. Only one
Which of the statements given above is/are correct? B. Only two
A. 1 only B. 2 only C. All three
C. Both 1 and 2 D. Neither 1 nor 2 D. None
w w w. d r i s h t i I A S.c om PT SPRINT (2024) ECONOMY QUESTIONS AND ANSWERS 17
106. Consider the following statements: 109. Consider the following statements:
1. India is the second largest cotton producer in the 1. AMC Repo Clearing Limited (ARCL) offers triparty
world, after China. repo services to facilitate repo transactions in
2. In India, cotton production accounts for corporate bonds, which are long-term borrowings
approximately 25% of the world's total output and backed by securities as collateral.
is often referred to as "White-Gold". 2. Triparty repo services are referred to the financial
3. Mating disruption using synthetic pheromones is transactions where a third party, often a custodian
one technique that can be used to control the or clearing agent, acts as an intermediary between
population of pink bollworm moths and reduce the the two parties involved in a repo agreement.
damage they cause to cotton crops. Which of the statements given above is/are not correct?
How many of the statements given above are correct? (a) 1 only (b) 2 only
A. Only one (c) Only 1 and 2 (d) Neither 1 nor 2
B. Only two 110. Consider the following statements regarding the state
C. All three of formal employment in India:
D. None 1. As per the NSO’s Periodic Labour Force Survey (PLFS)
report for the year 2021-22, the unemployment rate
107. Consider the following statements regarding T+1 for 2021-22 was 4.1%.
settlement cycle: 2. Between 2017 and 2022, the number of regular
1. T in T+1 represents the trade date. contributors to Employees Provident Fund’s (EPF)
2. This cycle means that trade-related settlements consistently increased.
occur within one hour of the actual transaction. Which of the given above statements is/are correct?
3. India became the first country to implement the T+1 a. Only 1 b. Only 2
settlement cycle in top-listed securities. c. Both 1 and 2 d. Neither 1 nor 2
How many of the above statements are correct?
111. Regarding the scheme “Vivad se Vishwas II”, consider
A. Only one
the following statements:
B. Only two 1. The scheme “Vivad se Vishwas II” is launched by the
C. All three Ministry of Finance.
D. None 2. The scheme aims to resolve contractual disputes
efficiently and promote ease of doing business with
108. Consider the following statements about MSMEs:
the government.
1. Their general definition is based on Investment in
3. Government e-Marketplace (GeM) portal will be
Plant & Machinery/equipment and Annual Turnover.
used for the implementation of the scheme.
2. The eligibility criteria for the Micro category require
How many of the statements given above are correct?
that the investment should not exceed Rs. 1 crore,
and the annual turnover should not exceed Rs. 5 A. Only one B. Only two
crores. C. All three D. None
3. The Small category mandates that the investment 112. With reference to Sugarcane in India, consider the
should not exceed Rs. 10 crores, and the annual following statements:
turnover should not exceed Rs. 50 crores. 1. India is the second largest producer of sugarcane
4. For the Medium category, the permissible after Brazil.
investment limit is not more than Rs. 50 crores, and 2. Maharashtra is one of top sugarcane producing
the annual turnover should not exceed Rs. 250 states in India.
crores. 3. Fair and Remunerative Price (FRP) is set by the
Which of the given above statements is/are correct? government that sugar mills are obligated to pay to
a. Only 1 and 2 farmers for the sugarcane procured from them.
b. Only 2 and 3 How many of the statements given above are correct?
c. Only 3 and 4 A. One only B. Two only
d. All the above C. All three D. None
18 PT SPRINT (2024) ECONOMY QUESTIONS AND ANSWERS www.d rish ti IAS.co m

113. Which of the following statements is/are correct? 116. Which of the following are a part of India’s Foreign
1. India ranks 4th globally in installed wind energy Exchange Reserves:
capacity as of April 2023. 1. Foreign Currency Assets
2. Gujarat has the highest wind power potential at 120 2. Gold reserves
meters above ground level. 3. Foreign Assets Held by Indian Individuals or
3. The objective of National Offshore Wind Energy Businesses
Policy is to develop wind energy in the Indian 4. Reserve position with the International Monetary
Exclusive Economic Zone (EEZ) along the Indian Fund (IMF).
coastline. Select the correct answer using the code given below:
Select the correct answer using the code given below: A. 1, 2 and 4 only
1. 1 and 2 only B. 1 and 2 only
2. 2 and 3 only C. 2, 3 and 4 only
3. 1 only D. 1, 2, 3 and 4
4. 1, 2 and 3
117. Consider the following statements regarding the
114. Production Linked Incentive Scheme (PLI):
Statement-I: 1. Under the scheme, both Domestic and Foreign
Deflation refers to a sustained and general decrease in companies receive financial rewards for
the overall price levels of goods and services in the manufacturing in India, based on a percentage of
economy. their revenue over up to five years.
Statement-II: 2. The incentives provided under the scheme are
calculated on the basis of both incremental sales
In a deflationary environment, consumers have to buy
and local value addition done over the period of five
less goods and services for the same amount of money
years.
over time.
3. The subsidy in the scheme for Mobile and allied
Which one of the following is correct in respect of the
Component Manufacturing is paid only for finishing
above statements?
the phone in India, not on how much value is added
A. Both Statement-I and Statement-II are correct and by manufacturing in India.
Statement-II is the correct explanation for How many of the statements given above are correct?
Statement-I
A. Only one
B. Both Statement-I and Statement-II are correct and
B. Only two
Statement-II is not the correct explanation for
Statement-I C. All three
C. Statement-I is Correct but Statement II is incorrect D. None
D. Statement-I is incorrect but Statement-II is correct 118. With reference to Payment Banks in India, consider the
following statements :
115. With reference to the Floating rate loans, consider the
following statements: 1. Nachiket Mor committee recommended setting up
of 'Payments Bank' to cater to the lower income
1. These loans have an interest rate that changes
groups and small businesses.
periodically, depending on a benchmark rate or the
2. They can accept both demand deposits and time
base rate.
deposits from their customers.
2. They are common for credit cards, mortgages, and
3. They are not allowed to accept deposits from Non-
other consumer loans.
Resident Indians (NRIs).
3. They are at least 1-2% more expensive than the fixed
How many of the above statements are correct?
interest loans.
A. Only one
How many of the statements given above is/are correct?
B. Only two
A. Only one B. Only two
C. All three
C. All three D. None
D. None
w w w. d r i s h t i I A S.c om PT SPRINT (2024) ECONOMY QUESTIONS AND ANSWERS 19
119. Consider the following statements regarding Real Estate 123. Recently the Reserve Bank of India (RBI) has initiated a
(Regulation and Development) Act, 2016: pilot programme aimed at evaluating the feasibility of
1. Under the Act, all real estate projects with a a 'Public Tech Platform for Frictionless Credit'. What will
minimum plot size of 1000 sq.mt or eight apartments be the possible benefits of this programme:
must be registered with RERAs before launch. 1. The platform's data consolidation might lead to
2. Promoters are required to deposit 70% of the increased vulnerability in credit risk assessment and
collected funds in a separate escrow account for the less efficient credit portfolio management.
specific project’s construction and land costs. 2. Access to accurate information supports informed
3. Appellate Tribunals are mandated to adjudicate and swift credit assessments.
cases within 90 days. 3. The platform exacerbates operational challenges by
How many of the statements given above are correct? increasing the need for multiple visits and extensive
A. Only one B. Only two documentation, resulting in heightened costs for
both lenders and borrowers.
C. All three D. None
4. The platform's streamlined processes lead to quicker
120. Consider the following: disbursement and scalability, resulting in a more
1. Benefit for the creditors efficient credit ecosystem.
2. Undervaluation of stock prices How many of the above statements is/are correct?
3. Decrease in export competitiveness a. Only one
4. Wage Price Spiral b. Only two
How many of the above could be the impact/s of rising c. Only three
Inflation? d. All four
A. Only one B. Only two
124. Which of the following can be the implications of Excess
C. Only three D. All four Liquidity?
121. With reference to the Account Aggregator (AA), consider 1. Higher Interest Rates
the following statements: 2. Increase in Investment
1. It is a framework that simply facilitates sharing of 3. Rise in Income Inequality
financial information in a real-time and data-blind 4. Inflationary Pressure
manner (encrypted) between regulated entities.
Select the correct answer using the code given below:
2. AAs enable flow of data between Financial
A. 2, 3 and 4 only
Information Providers (FIPs) and Financial
Information Users (FIUs). B. 1, 3 and 4 only
Which of the statements given bocce is/are correct? C. 2 and 3 only
A. 1 only B. 2 only D. 1, 2 and 3 only
C. Both 1 and 2 D. Neither 1 nor 2 125. With reference to the terms associated with Economy,
consider the following statements:
122. The term Worldcoin which was recently seen in news,
is a/an? 1. Corporate bonds are debt securities issued by
corporations to raise capital for various purposes.
A. Global digital currency backed by a consortium of
central banks. 2. Triparty repo services are financial transactions
where a third-party acts as an intermediary between
B. Non-profit organization working to provide internet
the two parties involved in a repurchase agreement.
access in remote areas.
Which of the statements given above is/are correct?
C. Blockchain-based social media platform aiming to
compete with existing giants. A. 1 only
D. Initiative to create a digital network in which B. 2 only
everyone can claim some kind of stake, and join the C. Both 1 and 2
digital economy. D. Neither 1 nor 2
20 PT SPRINT (2024) ECONOMY QUESTIONS AND ANSWERS www.d rish ti IAS.co m

126. Consider the following statements regarding Index of 129. Consider the following Pairs:
Industrial Production (IIP): Subsidy Boxes
1. Base Year for IIP is 2017-2018. Nature of subsidies included
2. It is released by the Ministry of Commerce and 1. Amber Box - Those subsidies that can distort
Industries. international trade by making a country's products
cheaper
3. The core industries have a combined weight of
around 40% in the Index of Industrial Production 2. Blue Box - Domestic support measures that don’t
cause trade distortion or at most cause minimal
(IIP).
distortion
Which of the statements given above are not correct?
3. Green Box - Any support that would normally be in
a. 1 and 2 only the amber box is placed in the g re e n b ox i f i t
b. 2 and 3 only requires farmers to limit production.
c. 1 and 3 only How many pairs give above are matched correctly?
d. 1, 2 and 3 a. Only 1 pair b. Only 2 pairs
c. Only 3 pairs d. All 4 pairs
127. Consider the following statements regarding Steel
Industry in India: 130. Consider the following statements regarding Debt for
Climate Swaps:
1. Green Steel refers to steel that is produced using
1. These swaps involve reducing debt in exchange for
renewable energy sources.
policy commitments or spending to mitigate the
2. India is the 2nd largest producer of crude steel after effects of climate change by debtor countries.
China. 2. Only official bilateral debt can be involved in debt-
3. The National Steel Policy 2017 allows 100% Foreign for-climate swaps.
Direct Investment (FDI) in the steel sector through Which of the statements given above is/are correct?
automatic route. a. 1 only b. 2 only
Which of the statements given above are correct? c. Both 1 and 2 d. Neither 1 nor 2
a. 1 and 2 only
131. With respect to the Foreign Contribution Regulation Act
b. 2 and 3only (FCRA), 2010, consider the following statements:
c. 1 and 3only 1. It requires every person or NGO wishing to receive
d. 1, 2 and 3 foreign donations to be registered under the Act.
2. The Act prohibits receipt of foreign funds by
128. With reference to Central Counterparties (CCPs), which candidates for elections and journalists.
the statements given below is not correct?
Which of the statements given above is/are correct?
1. CCP is a financial institution that acts as A. 1 only B. 2 only
an intermediary between buyers and sellers in
C. Both 1 and 2 D. Neither 1 nor 2
various derivatives and equities markets.
2. CCPs reduce risks associated with counterparty, 132. Consider the following statements:
operational, settlement, market, legal, and default 1. The Monopolies and Restrictive Trade Practices Act,
issues. 1969 (MRTP Act) was repealed and replaced by the
Competition Act, 2002.
3. In India, they are regulated by both Reserve Bank
2. The Competition Commission of India (CCI) is a
of India (RBI) and Security and Exchange Board of
quasi-judicial body which gives opinions to statutory
India (SEBI). authorities and also deals with other cases.
Select the correct answer using codes given below: Which of the statements given above is/are correct?
a. 3 only A. 1 only
b. 1 and 2 only B. 2 only
c. 2 and 3 only C. Both 1 and 2
d. None of the above D. Neither 1 nor 2
w w w. d r i s h t i I A S.c om PT SPRINT (2024) ECONOMY QUESTIONS AND ANSWERS 21
133. If US federal reserve increases Interest rates, what can 137. Consider the following statements:
be the possible impacts of it on Indian economy? 1. Graphite, Lithium and Cobalt are among the major
1. Costlier imports critical minerals.
2. Surge in stock prices 2. India is a member of Minerals Security Partnership
3. Increase in India’s external debt burden (MSP).
Which of the options given above is/are correct? Which of the statements given above is/are correct?
A. 1 and 2 only B. 2 only A. 1 only B. 2 only
C. 1 and 3 only D. 1, 2, and 3 C. Both 1 and 2 D. Neither 1 nor 2

134. Consider the following statements about India’s 138. Which of the following statements is/are not correct?
Agriculture sector in FY2023: 1. Leaked biometric details of Aadhar card can be used
to bypass the need of One Time Passwords (OTPs)
1. Marine products, rice, cotton and sugar have been
for secured bank transactions in Aadhaar-enabled
the driving forces behind India's agricultural exports.
Payment System (AePS).
2. Owing to Success of National mission on edible oils,
2. Aadhaar-enabled Payment System (AePS) was taken
India’s vegetable oil imports have dropped in past
up by the National Payments Corporation of India
two years.
(NPCI) which is a joint initiative of Reserve Bank of
Which of the statements given above is/are correct? India (RBI) and Indian Banks’ Association (IBA).
A. 1 only Select the correct answer using the code given below:
B. 2 only A. 1 only
C. Both 1 and 2 B. 2 only
D. Neither 1 nor 2 C. Both 1 and 2
135. With reference to Multi Model Logistics Parks in India, D. Neither 1 nor 2
consider the following statements: 139. With reference to the status of the Fisheries sector in
1. Logistic division under Ministry of Commerce & India, consider the following statements:
Industry has been authorized to develop Multi 1. India is the third largest fish producer and the second
Model Logistics Parks under the Bharatmala largest aquaculture producer.
Pariyojana. 2. Fishing and aquaculture industries are regarded as
2. It will be solely developed by a Special Purpose sunrise sectors.
Vehicle (SPV) called the National Highways and 3. Andhra Pradesh is the largest producer of fish in
Logistics Management (NHLML). India.
Which of the statements given above is/are not correct? Which of the statements given above is/are correct?
a. 1 only A. 1 and 2 only
b. 2 only B. 2 and 3 only
c. Both 1 and 2 C. 1 and 3 only
d. Neither 1 nor 2 D. 1, 2 and 3
136. Which of the following statements is/are not correct? 140. Consider the following statements regarding Wholesale
1. Financial Action Task Force (FATF) is the global Price Index (WPI):
money laundering and terrorist financing watchdog 1. WPI does not capture changes in the prices of
set up out of a G-20 meeting in Paris. services.
2. FATF considers efforts to counter the financing of 2. In WPI, more weightage is given to manufactured
proliferation of Weapons of Mass Destruction goods, while in CPI (Consumer Price Index), more
(WMD). weightage is given to food items.
Select the correct answer using the code given below: Which of the statements given above is/are correct?
A. 1 only B. 2 only A. 1 only B. 2 only
C. Both 1 and 2 D. Neither 1 nor 2 C. Both 1 and 2 D. Neither 1 nor 2
22 PT SPRINT (2024) ECONOMY QUESTIONS AND ANSWERS www.d rish ti IAS.co m

141. With reference to the Liberalised Remittance Scheme 145. Consider the following statements:
(LRS), consider the following statements: 1. Insurance Regulatory and Development Authority
1. It is the scheme of Reserve Bank of India (RBI). of India (IRDAI) is planning to launch Bima Trinity to
2. The scheme is not available to corporations and make insurance activities hassle free.
partnership firms. 2. IRDAI Vision 2047 aims to make the Indian insurance
3. Trading in foreign exchange abroad is prohibited sector globally attractive.
under this scheme. Which of the statements given above is/are correct?
Which of the statements given above is/are correct? A. 1 only B. 2 only
A. 1 and 2 only B. 2 and 3 only C. Both 1 and 2 D. Neither 1 nor 2
C. 1 and 3 only D. 1, 2 and 3 only 146. In March 2023, banks from 18 countries were allowed
142. Which of the following statements is/are correct? by the RBI to open Special Rupee Vostro Accounts
(SRVAs) to settle payments in Indian rupees. Which
1. The withdrawal of the Rs 2000 notes is part of the
among the following country’s banks is allowed to open
RBI's Clean Note Policy.
Special Rupee Vostro Accounts (SRVAs) by RBI?
2. The demonetization process in India requires the
A. China B. Malaysia
recommendation of the Reserve Bank of India (RBI).
C. Thailand D. Indonesia
3. The fifty paise (half a rupee) coins is not a legal
tender in India. 147. Consider the following statements regarding Trademark:
Select the correct answer using the code given below: 1. Trademark is a logo used by a company. It is
a. 1 and 2 only governed by the Patents Act, 1970
b. 2 and 3 only 2. Every ten years, trademark must be renewed as they
c. 1 only expire.
d. 1, 2 and 3 Which of the statements given above is/are correct?
A. 1 only B. 2 only
143. Consider the following statements:
C. Both 1 and 2 D. Neither 1 nor 2
1. The transfer of RBI's surplus to the government is
governed by banking conventions and global 148. Consider the following statements related to Non-
practices, as there is no specific statutory provision Banking Financial Companies:
available for it. 1. They are registered under the RBI Act, 1934.
2. The surplus transfer by the RBI is continuously 2. Deposit insurance facility of Deposit Insurance and
increasing due to the healthy monetary policy of the Credit Guarantee Corporation is not available to
country. depositors of NBFCs.
Which of the statements given above is/are not correct? Which of the statements given above is/are correct?
A. 1 only B. 2 only A. 1 only B. 2 only
C. Both 1 and 2 D. Neither 1 nor 2 C. Both 1 and 2 D. Neither 1 nor 2

144. Regarding the new tax rules for online gaming platforms 149. Consider the following statements:
in India, consider the following statements: 1. Solar Energy Corporation of India Limited has been
1. Online gaming platforms are not required to deduct recognized as a miniratna Category -I company.
tax if net winning is less than Rs 100. 2. A central Public Sector Enterprise can be recognised
2. Bonus and referral bonus provided by the online as miniratna category-I company when it has made
gaming company are not considered as taxable profit continuously for 3 years and has not defaulted
deposits. in the repayment of loans to the government.
3. The new tax rules have been introduced by the Which of the statements given above is/are correct?
Central Board of Direct Taxes (CBDT). a. 1 only
Which of the statements given above is/are correct? b. 2 only
a. 1 and 2 only b. 1 and 3 only c. Both 1 and 2
c. 3 only d. 1, 2 and 3 only d. Neither 1 nor 2
w w w. d r i s h t i I A S.c om PT SPRINT (2024) ECONOMY QUESTIONS AND ANSWERS 23
150. With reference to the Virtual Digital Assets (VDAs), 154. What is the Nifty Reits & InvITs Index launched by
consider the following statements: National Stock Exchange Indices Ltd?
1. They are defined under the newly inserted clause 1. The index tracks the performance of Reits and InvITs
under the Income Tax Act, 1961. that are publicly listed and traded on the NSE.
2. They are created using cryptography with the 2. The index will be reviewed and rebalanced on an
annual basis.
representation of having inherent value.
Select the correct answer using the following code:
Which of the statements given above is/ are correct?
(a) 1 only
A. 1 only
(b) 2 only
B. 2 only
(c) Both 1 and 2
C. Both 1 and 2
(d) Neither 1 nor 2
D. Neither 1 nor 2
155. Consider the following statements related to
151. Consider the following statements regarding India- ‘Infrastructure Investment Trust’
Russia trade ties: 1. They are regulated by the Reserve Bank of India.
1. Russia is India’s Seventh biggest trading partner. 2. They are listed on exchanges just like stocks through
2. The balance of trade currently tilts in favor of India. initial public offerings.
Which of the statements given above is/are correct? Which of the statements given above is/are correct?
(a) 1 Only A. 1 only
(b) 2 Only B. 2 only
C. Both 1 and 2
(c) Both 1 and 2
D. Neither 1 nor 2
(d) Neither 1 nor 2
156. Consider the following statements about India’s Generic
152. Consider the following statements: medicines:
1. Off-Budget liabilities refer to debts taken by private 1. India is the largest provider of generic medicines
sector entities to finance government programs and and the leading vaccine manufacturer, globally.
subsidies outside the traditional budget. 2. DCGI acts as an appellate authority in case of
2. Bondholders of off-budget liabilities are willing to disputes regarding the quality of drugs.
forego their high-yield bonds and accept prepayment 3. According to World Trade organization (WTO),
offers from the government. generic copies of drugs are legal from the patent
Which of the statements given above is/are not correct? point of view when they are made after the patent
has expired.
(a) 1 Only
How many ofis/are correct?
(b) 2 Only
A. 2 and 3 only
(c) Both 1 and 2
B. 1 and 2 only
(d) Neither 1 nor 2
C. 1, 2 and 3 only
153. Consider the following statements: D. 3 only
1. India is among the top 5 tea exporters in the world. 157. Which organization is responsible for administering the
2. Tea Board of India is the statutory body under the Geographical Indications of Goods (Registration &
Ministry of Agriculture and Farmers' Welfare. Protection) Act, 1999 in India?
Which of the statements given above is/are correct? A. Registrar of Companies
A. 1 only B. Controller General of Patents, Designs, and
Trademarks
B. 2 only
C. Ministry of Commerce and Industry
C. Both 1 and 2
D. Geographical Indications Registry
D. Neither 1 nor 2
24 PT SPRINT (2024) ECONOMY QUESTIONS AND ANSWERS www.d rish ti IAS.co m

158. With reference to QR-Code Based Coin Vending Machine 162. With reference to ‘Transmission sector in India’, consider
(QCVM), consider the following statements: the following statements:
1. It is a cashless coin dispensation machine which 1. Transmission sector in India is regulated by Ministry
would dispense coins using a credit card. of New and Renewable Energy.
2. It would eliminate the need for physical tendering 2. India is the third largest producer and consumer of
of banknotes and their authentication. electricity in the world.
Which of the statements given above is/are correct? Which of the statements given above is/are correct?
A. 1 only B. 2 only a. 1 only
C. Both 1 and 2 D. Neither 1 nor 2 b. 2 only
159. Which of the following statements is/are correct c. 2 and 1 only
regarding the Foreign Contribution Regulation Act d. None of the above
(FCRA), 1976?
163. Which of the following can be a potential way to mitigate
1. An NGO seeking to receive foreign donations under Bank Run?
FCRA must open a bank account in a public sector
1. Deposit Insurance
bank.
2. FCRA registration for NGOs is valid for 7 years and 2. Limit delay or shut down withdrawals
must be renewed within 3 months of the date of 3. Borrowing more money from other banks
expiration. Select the correct answer using the code given below:
Which of the statements given above is/are correct? a. 1 and 2 only
A. 1 only B. 2 only b. 2 and 3 only
C. Both 1 and 2 D. Neither 1 nor 2 c. 2 only
160. Consider the following statements regarding SWAMIH d. 1,2 and 3
(Special Window for Affordable and Mid-Income 164. With reference to Critical Minerals, consider the
Housing) Investment Fund: following statement:
1. It has been set up as a Category-II Alternate 1. These are the elements that are the building blocks
Investment Fund (AIF) debt fund registered with the of essential modern-day technologies.
Reserve Bank of India.
2. Only Copper, lithium and nickel are examples of
2. The only eligibility criteria for the real estate projects critical minerals.
to obtain loan from SWAMIH Fund is that they must
Which of the statements given above is/are correct?
be registered under RERA.
A. 1 only
Which of the statements given above is/are correct?
B. 2 only
A. 1 only
B. 2 only C. Both 1 and 2
C. Both 1 and 2 D. Neither 1 nor 2
D. Neither 1 nor 2 165. With reference to Trade Infrastructure for Export
Scheme (TIES), consider the following statements:
161. With reference to Virtual Digital Assets and Prevention
of Money Laundering Act (PMLA) 2002, consider the 1. It provides grants-in-aid only to central government-
following statements: owned agencies or their joint ventures for
infrastructure projects.
1. VDA service providers must adhere to the same
reporting requirements and KYC requirements as 2. It includes Border Haats, Land customs stations but
other regulated companies, such as banks. not Special Economic Zones within its scope.
2. The PMLA controls the transfer of virtual digital assets. Which of the statements given above is/are correct?
Which of the following statements given above is/are A. 1 only
correct? B. 2 only
A. 1 only B. 2 only C. Both 1 and 2
C. Both 1 and 2 D. Neither 1 nor 2 D. Neither 1 nor 2
w w w. d r i s h t i I A S.c om PT SPRINT (2024) ECONOMY QUESTIONS AND ANSWERS 25
166. Regarding the India’s Digital Ecosystem, consider the 170. Consider the following about Insolvency and Bankruptcy
following statements: code (IBC):
1. It has the 3rd largest telecom market globally. 1. The IBC covers all individuals, companies, Limited
Liability Partnerships (LLPs) and partnership firms.
2. In the last few years, India's digital economy grew
10 times faster than the national economy. 2. Under IBC, adjudicating authority for companies and
LLPs is Debt Recovery Tribunal (DRT).
3. It is the most connected democracy in the world,
How many ofis/are correct?
where 8 billion UPI (Unified Payment Interface)
A. 1 only
transactions are made every month.
B. 2 only
Which of the statements given above is/are correct?
C. Both 1 and 2
A. 1 and 2 only B. 2 only
D. Neither 1 nor 2
C. 3 only D. 1, 2 and 3 only
171. Regarding the Digital Public Infrastructure (DPI),
167. With reference to Evergreening of Patents, consider the consider the following statements:
following statements: 1. It aims to empower the citizen with a real ability to
1. It is a practice of tweaking drugs to extend their control their data.
patent term. 2. It could lead to the authoritarianism and digital
2. Indian government has allowed the practice of colonization.
“evergreening” of patents. 3. India became the first country to develop Digital
identity (Aadhar), Real-time fast payment (UPI) and
3. It will aid millions of people who can't afford the
Account Aggregator.
expensive modified drugs.
Which of the statements given above is/are correct?
Which of the statements given above is/are correct?
A. 1 and 2 only
A. 1 and 2 only B. 2 and 3 only B. 2 and 3 only
C. 1 only D. 1, 2 and 3 only C. 1 and 3 only
168. With reference to the IMF Bailouts, consider the D. 1, 2 and 3
following statements: 172. With reference to Assamese Gamocha , consider the
1. Countries seek help from the IMF usually when their following statements:
economies face a major macroeconomic risk or 1. It is a rectangular piece of cloth which comes in
political instability. various colors and designs, and the most popular
2. The IMF lends money to the troubled economies among them are red-and-white with chequered
pattern.
often in the form of Bonds.
2. Assamese Gamocha has been granted the
Which of the statements given above is/are correct?
Geographical Indication (GI) tag.
A. 1 only Which of the statements given above is/are correct?
B. 2 only a. 1 only
C. Both 1 and 2 b. 2 only
D. Neither 1 nor 2 c. Both 1 and 2
d. None of the above
169. Consider the following statements:
1. Both IMF and World bank were established during 173. Which of the following is NOT an objective of NPCI?
Bretton woods conference in 1944. (a) To provide safe, secure, and efficient retail payment
2. The IMF has universal membership. systems in India.
(b) To promote digital payments and financial inclusion
3. Membership in the IMF is a prerequisite to
in the country.
membership in the IBRD.
(c) To provide a range of products and services to banks
Which of the statements given above is/are correct? and financial institutions.
a. 1 and 2 only b. 2 only (d) To regulate the payment and settlement systems in
c. 1 and 3 only d. 1, 2 and 3 only India.
26 PT SPRINT (2024) ECONOMY QUESTIONS AND ANSWERS www.d rish ti IAS.co m

174. With reference to the Funds for Startups (FFS) Scheme, 177. Consider the following statements:
consider the following statements: 1. IPEF is an India-led initiative that aims to strengthen
1. The Ministry of Finance is the operating agency for economic partnership among participating
FFS. countries.
2. It aims to make capital available for startups at an 2. It is a Free Trade Agreement (FTA).
early stage only. Which of the statements given above is/are not correct?
Which of the statements given above is/are correct? A. 1 only
A. 1 only B. 2 only
B. 2 only C. Both 1 and 2
C. Both 1 and 2 D. Neither 1 nor 2
D. Neither 1 nor 2
178. With reference to Hallmarking, consider the following
175. with reference to the report “Falling Long-Term Growth statements:
Prospects: Trends, Expectations, and Policies”, consider 1. It is the accurate determination and official recording
the following statements. of the proportionate content of precious metal.
1. The report Falling Long-Term Growth Prospects: 2. In India, it consisted of 3 marks, the BIS logo, purity
Trends, Expectations, and Policies (2020-2030) was of the article and six-digit alphanumeric HUID only.
released by International Monetary Fund (IMF). Which of the statements given above is/are correct?
2. According to the report, India has lost growth A. 1 only
momentum over the past two decades.
B. 2 only
Which of the statements given above is/are correct?
C. Both 1 and 2
A. 1 only
D. Neither 1 nor 2
B. 2 only
C. Both 1 and 2 179. What is the purpose of the Social Stock Exchange (SSE)?
D. Neither 1 nor 2 1. To provide a platform for social enterprises to raise
funds from the public and increase transparency
176. With reference to the Aviation Sector of India, consider about fund mobilisation and utilisation.
the following statements: 2. Corporate foundations can get listed on a stock
1. It is currently the world's 5th-largest civil aviation exchange.
market. Which of the statements given above is/are correct?
2. Over the past few years, India’s international A. 1 only
passenger traffic had remained stagnant.
B. 2 only
Which of the statements given above is/are correct?
C. Both 1 and 2
A. 1 only B. 2 only
D. Neither 1 nor 2
C. Both 1 and 2 D. Neither 1 nor 2
w w w. d r i s h t i I A S.c om PT SPRINT (2024) ECONOMY QUESTIONS AND ANSWERS 27

ANSWERS

1. Ans: C z Unemployment Rate (UR): UR is defined as the


Exp: percentage of persons unemployed among the
z Government Securities (G-Secs) are tradable persons in the labour force. Hence, statement 3 is not
instrument issued by the Central Government or the correct.
State Governments. Hence, statement 1 is correct. z Activity Status- Usual Status: The activity status of a
z A G-Sec is a type of debt instrument issued by the person is determined based on the activities pursued
government to borrow money from the public to by the person during the specified reference period.
finance its Fiscal Deficit. Hence, statement 2 is correct. ‹ When the activity status is determined based on
‹ It acknowledges the Government’s debt the reference period of the last 365 days
obligation. preceding the date of the survey, it is known as
€ Such securities are short-term (usually called the usual activity status of the person.
treasury bills, with original maturities of less z Activity Status- Current Weekly Status (CWS): The
than one year- presently issued in three activity status determined based on a reference
tenors, namely, 91-day, 182 days and 364 period of the last 7 days preceding the date of the
days) or long-term (usually called survey is known as the CWS of the person.
Government bonds or dated securities with
4. Ans: B
original maturity of one year or more).
Exp:
z G-Secs carry practically no risk of default and, hence,
are called risk-free gilt-edged instruments. Hence, The Finance Ministry has released a comprehensive review
statement 3 is correct. shedding light on India’s foreign direct investment (FDI)
landscape, revealing both declines and hopeful prospects.
2. Ans: A z India’s net FDI inflows dropped by almost 31% to USD
Exp: 25.5 billion over the first ten months of 2023-24.
Non-Banking Financial Company (NBFC): Hence, statement 1 is not correct.
z An NBFC is a company registered under the Companies z Around 65% of India’s FDI equity inflows were observed
Act, 1956, involved in various financial activities such in services, drugs and pharmaceuticals, construction
as providing loans and advances, acquiring shares, (infrastructure activities), and non-conventional
stocks, bonds, debentures, and securities issued by the energy sectors.
government or local authorities. Hence, statement 1 is z The Netherlands, Singapore, Japan, the USA, and
correct. Mauritius accounted for approximately 70% of the
z NBFCs do not include institutions primarily engaged total FDI equity inflows into India. Hence, statement 2
in: is correct.
‹ Agricultural or industrial activities
5. Ans: B
‹ Purchase or sale of goods (other than securities)
Exp:
‹ Providing services
‹ Dealing in immovable property. Hence, statement ‹ NBFCs provide diverse financial services like
2 is not correct. personal loans, home loans, vehicle loans, gold
loans, microfinance, insurance, and investment
3. Ans: D management. Hence, Statement 1 is correct.
Exp: ‹ NBFCs do not form part of the payment and
z Labour Force Participation Rate (LFPR): LFPR is settlement system and cannot issue cheques
defined as the percentage of persons in the labour drawn on themselves. Hence, Statement 2 is not
force (i.e. working or seeking or available for work) in correct.
the population. Hence, statement 1 is not correct. ‹ They can accept public deposits for a minimum of
z Worker Population Ratio (WPR): WPR is defined as 12 months and a maximum of 60 months.
the percentage of employed persons in the However, NBFCs cannot accept demand deposits.
population. Hence, statement 2 is not correct. Hence, Statement 3 is correct.
28 PT SPRINT (2024) ECONOMY QUESTIONS AND ANSWERS www.d rish ti IAS.co m

6. Ans: C z The Reserve Bank of India (RBI) set up an inter-


Exp: regulatory Working Group in 2016 to look into and
z India is the second-largest consumer of edible oil
report on the granular aspects of FinTech and its
implications so as to review the regulatory framework
globally and one of its largest importers. India
and respond to the dynamics of the rapidly evolving
currently struggles with achieving self-sufficiency in
FinTech scenario.
edible oils, importing 96% of its required palm oil,
which makes up 67% of the country’s edible oil import z The report recommended introducing an appropriate
bill. Hence, Statement 1 is correct. framework for a Regulatory Sandbox (RS) within a
well-defined space and duration where the financial
z Indonesia and Malaysia are the main global palm oil
sector regulator will provide the requisite regulatory
producers, followed by Thailand, Colombia, and
guidance, to increase efficiency, manage risks and
Nigeria. Hence, Statement 2 is correct.
create new opportunities for consumers.
7. Ans: C 9. Ans: A
Exp: Exp:
z India is the world’s second-largest producer of crude z The renowned Cuttack Rupa Tarakasi (Silver Filigree)
steel, with an output of 125.32 million tonnes (MT) of has been granted the Geographical Indication (GI) tag,
crude steel and 121.29 MT of finished steel production marking its distinct heritage and craftsmanship. gntiaD
in FY23. denroda eergilfi erehw ,aimatoposeM tneicna ot kcab
‹ India is also a net exporter of steel witnessing an kcattuC ot yenruoj sti ,ECB 0053 sa ylrae sa yrellewej
export of 6.72 MT of finished steel against the semulov skaeps aisenodnI dna aisreP aiv yllatinetop
import of 6.02 MT in 2022-23. Hence, statement edart emtiiram hguorht egnahcxe larutluc tuoba
.tcerroc si 1 tnemetats ,ecneH .setuor
1 is correct.
‹ Filigree is ornamental work especially of fine wire
z Stainless steel is a type of steel alloy that contains a
of gold, silver, or copper applied chiefly to gold
minimum of 10.5% chromium by mass. Hence, and silver surfaces.
statement 2 is correct.
z A GI tag is a label used on products linked to a specific
‹ It is known for its exceptional corrosion resistance, geographic area, ensuring only authorized users from
making it highly suitable for various applications that region can use the product’s name.
where durability and resistance to rust and ‹ It guards against imitation and lasts for 10 years
staining are essential. once registered. Hence, statement 2 is not
correct.
8. Ans: C
‹ The Department for Promotion of Industry and
Exp: Internal Trade (DPIIT) manages GI registration in
z A Regulatory Sandbox (RS) refers to live testing of new India under the Geographical Indications of
products or services in a controlled regulatory Goods Act, 1999, in line with the TRIPS
environment for which regulators may or may not agreement.
permit certain regulatory relaxations for the limited
10. Ans: C
purpose of testing. Hence, statement 1 is correct.
Exp:
z The RS is an important tool that enables more dynamic,
Cooperative banks in India:
evidence-based regulatory environments which learn
z Cooperative banks are financial institutions that are
from and evolve with emerging technologies. owned and operated by their members, who are also
z Recently, the Reserve Bank of India (RBI) revised the the bank’s customers.
timeline for the completion of various stages of a z Cooperative banks in India, are registered under the
Regulatory Sandbox (RS) to nine months from the Cooperative Societies Act of the State concerned or
previous seven months. the Multi-State Cooperative Societies Act, 2002.
‹ To enable regulated and orderly growth of FinTech Hence, statement 1 is correct.
ecosystem in India, the Reserve Bank in August z The Reserve Bank regulates the banking functions of
2019 became one of the few countries that have Urban Cooperative Banks under the provisions of
their very own Regulatory Sandbox (RS) Sections 22 and 23 of the Banking Regulation Act,
ecosystem. Hence, statement 2 is correct. 1949. Hence, statement 2 is correct.
w w w. d r i s h t i I A S.c om PT SPRINT (2024) ECONOMY QUESTIONS AND ANSWERS 29
11. Ans: B ‹ Initially, its objective was to examine and develop
Exp: measures to combat money laundering.
z Coal is the most important and abundant fossil fuel in ‹ After the 9/11 attacks on the US, the FATF in 2001
India. It accounts for 55% of the country’s energy expanded its mandate to incorporate efforts to
needs. combat terrorist financing.
z Lignite coal is characterized by its carbon content of z The recent removal of the United Arab Emirates (UAE)
40% to 55% and high moisture levels, and is primarily from the Financial Action Task Force (FATF) grey list has
found in areas including Tamil Nadu, Puducherry, sparked optimism for investment landscapes,
Gujarat, Rajasthan and Jammu & Kashmir. particularly in India’s Non-Banking Financial Companies
(NBFCs). Hence, statement 2 is not correct.
‹ Tamil Nadu (not Odisha) was the largest producer
of lignite and produced 22.480 Million Tonnes 14. Ans: B
(MT) in 2022-23 (Ministry of Coal, Government of Exp
India) India’s Spending on Research and Development (R&D) :
z Current per capita consumption of coal in India stands z India’s Declined R&D Expenditure:
at around 350 kilograms of oil equivalent per year, ‹ India’s expenditure on Research and Development
lower than developed countries. (R&D) has declined to 0.64% in 2020-21 of GDP
z Present import policy allows for the unrestricted (Gross Domestic Product), down from 0.8% in
import of coal under Open General License. 2008-2009 and 0.7% in 2017-2018. Hence,
‹ Consumers, including the steel, power, and statement 1 is not correct.
cement sectors, as well as coal traders, can import € This decrease is concerning, especially
coal based on their commercial requirements. considering repeated calls from government
z Hence, option B is correct. agencies to double R&D spending.
‹ The 2013 Science, Technology, and Innovation
12. Ans: B
Policy aimed to increase Gross Expenditure on
Exp: R&D (GERD) to 2% of GDP, a goal reiterated in the
z Milk Production in India: 2017-2018 Economic Survey. Hence, statement 2
‹ India is the highest milk producer i.e., ranks first is correct.
position in the world. Hence, statement 1 is € However, the reasons for the reduction in
correct. R&D spending are unclear. Potential factors
‹ India contributes 24% of global milk production in may include insufficient coordination among
the year 2021-22. Hence, statement 2 is not government agencies and a lack of strong
correct. political will to prioritise R&D expenses.
‹ In the last 10 years, milk production increased by z Developed Countries’ R&D Expenditure:
almost 60%, and per capita milk availability has ‹ Comparatively, most developed countries
increased by about 40%. allocate between 2% and 4% of their GDPs to
€ The top 5 milk-producing states are R&D. Hence, statement 3 is correct.
Rajasthan, Uttar Pradesh, Madhya Pradesh,
15. Ans: B
Gujarat and Andhra Pradesh.
Exp:
‹ The Indian dairy sector is growing by 6% per year
as compared to the global average of 2%. Hence, Employees’ Provident Fund Organisation:
statement 3 is correct. z It is a government organisation that manages the
provident funds and pension accounts for the
13. Ans: A workforce engaged in the organised sector in India.
Exp: Hence, statement 1 is not correct.
Financial Action Task Force (FATF): z It implements the Employees’ Provident Fund and
z FATF is the global money laundering and terrorist Miscellaneous Provisions Act, 1952.
financing watchdog set up in 1989 out of a G-7 meeting z It is administered by the Ministry of Labour &
of developed nations in Paris. Hence, statement 1 is Employment, Government of India. Hence, statement
correct. 2 is correct.
30 PT SPRINT (2024) ECONOMY QUESTIONS AND ANSWERS www.d rish ti IAS.co m

‹ It is one of the World’s largest Social Security ‹ The National Education Policy (NEP) 2020 has
Organisations in terms of clientele and the volume integrated creative arts, design, and sports into
of financial transactions undertaken. the school curriculum from Class 6 onwards,
fostering a conducive environment for nurturing
16. Ans: B talent in AVGC-XR. Hence, statement 2 is correct.
Exp:
19. Ans: C
Gross Value Added (GVA) :
z GVA measures the value producers add to goods and Exp:
services during production. Hence, statement 1 is RuPay:
correct. z RuPay is a payment system and financial services
z It’s calculated by subtracting the cost of inputs product developed by the National Payments
(intermediate consumption) from total output. Corporation of India (NPCI). Hence, statement 1 is
Hence, statement 2 is not correct. correct.
z It’s a key component of Gross Domestic Product z It is a domestic card payment network that can be
(GDP), reflecting economic growth. GVA growth rates used at Automated Teller Machines (ATMs), Point of
provide insights into sectoral performance, aiding Sale (POS) devices, and e-commerce websites across
economic analysis and policymaking. India. Hence, statement 2 is correct.
‹ GVA = GDP + subsidies on products - taxes on z The provision under the Payment and Settlement
products. Systems Act, 2007, empowered the Reserve Bank of
z If we deduct the depreciation from GVA we get Net India (RBI) and the Indian Banks’ Association (IBA) to
Value Added (NVA). create a secure electronic payment and settlement
system in India.
‹ NVA is the value of output less the values of both
intermediate consumption and consumption of 20. Ans: D
fixed capital.Hence, statement 3 is correct. Exp:
17. Ans: B The National Bureau of Economic Research (an American
Exp: NGO) defines recession as “a significant decline in economic
activity spread across the economy, lasting more than a few
z The Monetary Policy Committee’s (MPC)
months, normally visible in production, employment, real
accommodative stance means the central bank is
income, and other indicators.
prepared to expand the money supply to boost
z A recession occurs when a sustained decline in
economic growth. Hence, statement I is correct.
economic activity persists. It is of long duration. Hence,
z The MPC aims to withdraw accommodation gradually statement 1 is not correct.
to align inflation with the target while supporting
z A technical recession specifically entails two
growth. Hence, statement II is correct.
consecutive quarters of GDP decline. Hence, statement
z Hence, option B is correct because both Statement I 2 is not correct.
and Statement II are correct but statement II is not the
correct explanation of statement I. 21. Ans: C
Exp:
18. Ans: B
Payment Banks:
Exp:
z Payment banks are a specialized type of bank
India’s AVGC-XR Sector: introduced by the RBI in 2014. Hence, statement 1 is
‹ The AVGC-XR sector, currently employing 2.6 lakh correct.
individuals, anticipates creating 23 lakh direct
z They are designed to promote financial inclusion by
jobs by 2032, with revenues expected to soar to
offering basic banking services to the unbanked and
over USD 26 billion by 2030 from the current USD
underbanked population. Hence, statement 2 is
3 billion.
correct.
‹ India’s contribution to the global AVGC-XR sector
z They were introduced on the recommendations of the
is a mere 0.5%, as per the government data. India
Nachiket Mor committee set up by the RBI to examine
has the potential to capture 5% (USD 40 billion)
financial services for small businesses and low-income
of the global market share by the year 2025, with
households.
an annual growth of around 25-30% and creating
over 1,60,000 new jobs annually. Hence, z Example: Airtel Payments Bank, India Post Payments
statement 1 is not correct. Bank, etc.
w w w. d r i s h t i I A S.c om PT SPRINT (2024) ECONOMY QUESTIONS AND ANSWERS 31
22. Ans: B €Responsible for regulating India’s money
Exp: market and foreign exchange market.
Bilateral Investment Treaties (BITs): € Oversees fintech sectors like Digital
z Bilateral Investment Treaties (BITs) are reciprocal Payments, Digital Lending, and Digital or
agreements between two countries to promote and neo-banks.
protect foreign private investments in each other’s ‹ Securities and Exchange Board of India (SEBI):
territories. Hence, statement-I is correct. € Regulates securities markets and
‹ In the mid-’90s, the Indian government initiated intermediaries such as stockbrokers and
BITs to offer favorable conditions and treaty- investment advisors.
based protection to foreign investors and € Services like stockbroking and investment
investments. advisory fall under its jurisdiction.
z While presenting the interim Union budget 2024-25, ‹ Insurance Regulatory and Development
the Indian Finance Minister stated that India will be Authority of India (IRDAI):
negotiating Bilateral Investment Treaties (BITs) with its
€ Regulates insurers, corporate agents, web
trade partners to boost the inflow of Foreign Direct
aggregators for insurance, and third-party
Investment (FDI).
agents for insurance.
‹ This announcement comes at a time when India’s
€ Ensures compliance and integrity in the
bilateral treaties have dried up, more so, since
insurance sector.
the adoption of the Model BIT in 2016. Hence,
statement-II is correct. z Hence, option C is correct.
Hence, option B is correct because both Statement-I and 25. Ans: C
Statement-II are correct and Statement-II is not the cor- Exp:
rect explanation for Statement I.
z Article 270 of the Constitution outlines the distribution
23. Ans: A of net tax proceeds between the Union government
Exp: and the States. Hence, statement 1 is correct.
z Despite a swift post-pandemic recovery, India z Currently, the share of States from the divisible pool
experiences ‘premature deindustrialization’, (vertical devolution) stands at 41% as per the
exacerbating inequality as benefits of rapid growth recommendation of the 15th Finance Commission.
favour a small minority, widening existing disparities. Hence, statement 2 is correct.
Hence, statement I is correct. z Cess and surcharge collected by the Union government
z The Indian economy is encountering premature is estimated at around 23% of its gross tax receipts for
transitions to the services sector, impeding the growth 2024-25, which does not form part of the divisible
of the manufacturing industry in the country. Hence, pool and hence not shared with the States. Hence,
Statement II is correct. statement 3 is correct.
Hence, option A is correct because both Statement-I and
Statement-II are correct, and Statement-II is the correct ex-
26. Ans: A
planation for Statement-I.
Exp:
24. Ans: C
z India’s Atomic Energy Act, 1962 governs the subject of
Exp: ‘nuclear energy’, with the Government of India playing
z About Fintech: a central role in the development, operation, and
‹ Fintech, or financial technology, is the use of decommissioning of nuclear facilities. Hence,
digital platforms, software, and services to statement 1 is correct.
provide or facilitate financial services, such as z The Consolidated Foreign Direct Investment (FDI)
payments, lending, insurance, wealth policy of India prohibits foreign investment in the
management, and more. atomic energy sector. Hence, statement 2 is not
z Key Regulatory Bodies for Fintech in India: correct.
‹ Reserve Bank of India (RBI): z In contrast, there is no restriction on FDI in the industry
€ Regulates banks, NBFCs, PSPs, and credit for manufacturing nuclear equipment and parts for
bureaus. nuclear power plants and other related facilities.
32 PT SPRINT (2024) ECONOMY QUESTIONS AND ANSWERS www.d rish ti IAS.co m

z Recently, a NITI Aayog (National Institution for 30. Ans: A


Transforming India) panel recommended the Exp:
Government of India to allow FDI into India’s atomic z REC Limited (formerly Rural Electrification Corporation
sector. Limited), a Maharatna Central Public Sector Enterprise
27. Ans: B under the Ministry of Power, has demonstrated its
unwavering commitment to the welfare of armed
Exp:
forces personnel through a substantial contribution to
z India, currently representing 2-3% of the global space the Armed Forces Flag Day Fund (AFFDF). Hence,
economy compared to the US (40%) and UK (7%), aims statement 1 is not correct.
to increase its share to over 10% by 2030, with Indian
z REC, registered with the Reserve Bank of India (RBI) as
Space Research Organisation (ISRO) ranking among the
a Non-Banking Finance Company (NBFC) and
top six largest space agencies globally. Hence,
Infrastructure Financing Company (IFC), finances
statement 1 is not correct.
various sectors including power infrastructure,
z 100% FDI Allowed: Under the amended policy, 100% renewable energy, and emerging technologies. Hence,
FDI is permitted in the space sector, aiming to attract statement 2 is correct.
potential investors to Indian space companies. Hence,
z REC Ltd. Serves as a key player in government flagship
statement 2 is correct.
schemes such as Pradhan Mantri Sahaj Bijli Har Ghar
z Liberalized Entry Routes: The entry routes for various Yojana (SAUBHAGAYA), Deen Dayal Upadhaya Gram
space activities are as follows: Jyoti Yojana (DDUGJY), National Electricity Fund (NEF)
‹ Up to 74% under Automatic Route: Satellites- Scheme which resulted in strengthening of last-mile
Manufacturing & Operation, Satellite Data distribution system, 100% village electrification and
Products, Ground Segment & User Segment. household electrification in the country.
€ Beyond 74%, the government route applies. ‹ REC has also been made the nodal agency for
28. Ans: C certain States and Union Territories for the
Revamped Distribution Sector Scheme (RDSS).
Exp:
Hence, statement 3 is correct.
Indian Spice Market:
z India is the world’s largest spice producer. It is also the 31. Ans: B
largest consumer and exporter of spices. Hence, Exp:
statement 1 is correct z The Monetary Policy Committee’s (MPC)
z India produces about 75 of the 109 varieties which are accommodative stance means the central bank is
listed by the International Organisation for prepared to expand the money supply to boost
Standardization (ISO). Hence, statement 2 is correct. economic growth. Hence, statement I is correct.
z The most produced and exported spices are pepper, z The MPC aims to withdraw accommodation gradually
cardamom, chilli, ginger, turmeric, coriander, cumin, to align inflation with the target while supporting
celery, fennel, fenugreek, garlic, nutmeg & mace, curry growth. Hence, statement II is correct.
powder, spice oils and oleoresins. z Hence, option B is correct because both Statement I
and Statement II are correct andStatement II is not the
29. Ans: A
correct explanation for Statement I.
Exp:
z Bitcoin is an open-source digital currency, that 32. Ans: D
facilitates instant payments without central authority Exp:
issuance. Hence, statement 1 is correct. z Inflation, as defined by the International Monetary
z Cryptocurrencies in India fall under the virtual digital Fund, is the rate of increase in prices over a given
assets (VDAs) category and are subject to taxation. period, encompassing a broad measure of overall price
Hence, statement 2 is not correct. increases or for specific goods and services.
‹ The profits generated from cryptocurrency ‹ Cost-push inflation is driven by an increase in the
trading are taxed at a rate of 30%, with an production costs for goods and services. This can
additional 4% cess (Union budget 2022-23). be caused by factors such as increased incomes,
z In 2022, the RBI launched its own Central Bank Digital increased costs of raw materials, or disruptions
Currency (CBDC) known as e-Rupee which is based on in the supply chain. Hence, statement 1 is not
blockchain technology. correct.
w w w. d r i s h t i I A S.c om PT SPRINT (2024) ECONOMY QUESTIONS AND ANSWERS 33
‹ Demand Pull inflation occurs when the demand 35. Ans: C
for goods and services exceeds their supply. Exp:
When the overall demand in the economy is high, z Reverse flipping is the process of shifting the domicile
consumers are willing to pay more for the of an Indian company back to India after it had moved
available goods and services, leading to a general its headquarters overseas, usually for tax or regulatory
rise in prices. Hence, statement 2 is not correct. reasons. It is also known as ‘re-domiciling’. Hence,
‹ Headline inflation is a measure of the total statement 1 is correct.
inflation within an economy, including z Reverse flipping has become a trend among Indian
commodities such as food and energy prices, startups, especially in the fintech sector, as they plan
which tend to be much more volatile and prone for Initial Public Offers (IPOs) or seek long-term
to inflationary spikes. benefits in the home market. Hence, statement 2 is
€ The headline inflation figure is reported correct.
through the Consumer Price Index (CPI), z This strategic move is fuelled by India’s flourishing
which calculates the cost to purchase a fixed economy, a huge market, promising venture capital,
basket of goods to determine how much favorable tax structures, robust intellectual property
inflation is occurring in the broad economy. protection, young and educated population and
Hence, statement 3 is not correct. supportive government policies.

33. Ans: C 36. Ans: D


Exp: Exp:
z The Reserve Bank of India (RBI) has recently revised Section 132 of the Income-Tax Act, 1961:
guidelines regarding inoperative accounts and z Section 132 of the Income-Tax Act, 1961, empowers
unclaimed deposits, aiming to streamline the the tax authorities to conduct searches and seizures of
classification and activation processes. Hence, persons and properties, without any prior judicial
warrant, if they have a “reason to believe” that the
statement 1 is correct.
person has concealed or evaded income. Hence,
z The revised instructions apply to all Commercial Banks
statement 1 is correct.
and all Cooperative Banks and will come into effect
‹ It grants authorities the power to search
from 1st April 2024. Hence, statement 2 is not correct.
buildings, places, vehicles, or aircraft based on
34. Ans: A suspicion of hiding financial assets.
Exp: z Pooran Mal vs Director of Inspection (1973):
z Steel is one of the widely used materials all over the ‹ The constitutionality of Section 132 was
world. The iron and steel industry is the bottom-line challenged in the case of Pooran Mal vs Director
producer industry. of Inspection (1973). Hence, statement 2 is
correct.
z The steel industry plays a pivotal role in crucial sectors
such as construction, infrastructure, automobile, ‹ The Supreme Court upheld the law, citing its own
engineering, and defense. judgment in M.P. Sharma vs Satish Chandra
(1954) by emphasizing that the power of search
z Over time, there has been remarkable expansion in the
and seizure is essential for the protection of social
Steel Sector, and India has become a significant global
security and is regulated by law.
player in steel manufacturing, ranking as the world’s
second-largest producer after China. Hence, statement 37. Ans: B
1 is correct. Exp:
z India’s major steel-producing states include Odisha, z The Indian toy industry exhibited remarkable growth
which leads among all steel-producing states, between FY 2014-15 and FY 2022-23, marked by a
followed by Jharkhand and Chhattisgarh. Karnataka, substantial decline in imports by 52% and a significant
Maharashtra, Gujarat, and West Bengal also play rise in exports by 239%. Hence, statement 1 is not
crucial roles. Hence, statement 2 is not correct. correct.
34 PT SPRINT (2024) ECONOMY QUESTIONS AND ANSWERS www.d rish ti IAS.co m

z A substantial increase in Basic Customs Duty (BCD) on z IMF’s Classification of Stabilised Arrangement:
toys (from 20% to 60% in February 2020, and ‹ The International Monetary Fund (IMF) classifies
subsequently to 70% in March 2023) aims to protect an exchange rate regime as a stabilized
the domestic toy industry from cheaper imports and arrangement when it determines that the
encourage local manufacturing. Hence, statement 2 is exchange rate has not moved beyond a 2% band
correct. in 6 months and that this stability has resulted
z The Directorate General of Foreign Trade (DGFT), from market interventions rather than market
under the Ministry of Commerce and Industry, conditions. Hence, statement 3 is correct.
mandated sample testing for each import consignment 40. Ans: B
to prevent the import of substandard toys, ensuring
Exp:
better quality control. Hence, statement 3 is correct.
Goods and Services Tax (GST):
38. Ans: D z It is a comprehensive indirect tax that was introduced
Exp: in India on 1st July 2017, through the 101st Constitution
z Arunachal Pradesh- Adi Kekir: Ginger variety. Hence, Amendment Act, 2016, with the slogan of ‘One Nation
pair 1 is correctly matched. One Tax’. Hence, statement 1 is not correct.
z West Bengal-Tangail Saree: Saree style originating z The GST Council is a constitutional body responsible
from Bengal with distinct weaving patterns. Hence, for making recommendations on issues related to the
pair 2 is correctly matched. implementation of the Goods and Services Tax (GST) in
z Gujarat- Kachchhi Kharek: Product of date palm India.
harvested at Khalal (fresh stage), which are bold, ‹ As per Article 279A (1) of the amended
crisp, and sweet. Hence, pair 3 is correctly matched. Constitution, the GST Council was constituted by
z Hence, option D is correct. the President. Hence, statement 2 is correct.
39. Ans: A 41. Ans: B
Exp: Exp:
z Floating Exchange Rate: z Foreign Portfolio Investment (FPI) refers to
‹ Market-Driven: Currency value is determined investments made by foreign individuals, corporations,
solely by supply and demand in the foreign and institutions in the financial assets of India, such as
exchange market, with minimal government stocks, bonds, and mutual funds. Hence, statement 1
intervention. Hence, statement 1 is not correct. is correct.
‹ High Volatility: The exchange rate can fluctuate ‹ These investments are mainly for the purpose of
significantly in response to economic news, short-term gains and portfolio diversification,
events, or market sentiment. unlike Foreign Direct Investment (FDI) which
‹ Promotes Flexibility: Businesses and individuals involves long-term ownership of assets. Hence,
can adjust to changing economic conditions statement 2 is not correct.
through market-determined exchange rates. z Luxembourg has become the third-largest region for
z Stabilized Arrangement: FPIs in India, surpassing Mauritius, with its Assets
‹ More Managed than Purely Floating: The Under Custody (AUC) growing by 30% to ₹4.85 lakh
government or central bank may intervene in the crore. Hence, statement 3 is correct.
foreign exchange market occasionally to smooth 42. Ans: B
out excessive volatility or maintain a target range Exp:
for the currency. Hence, statement 2 is not
Point of Presence (PoP) Regulations 2023:
correct.
‹ Moderate Volatility: Aiming for greater stability z The Pension Fund Regulatory and Development
than a pure float, but still accepting some degree Authority (PFRDA) notified the Point of Presence (PoP)
of fluctuation. Regulations 2023.
‹ Offers Predictability: Businesses and individuals z This regulation makes it easier for people to join the
can plan with a more stable exchange rate National Pension System (NPS) by simplifying the
environment. registration process. Hence, statement 1 is correct.
w w w. d r i s h t i I A S.c om PT SPRINT (2024) ECONOMY QUESTIONS AND ANSWERS 35
z Banks and non-banks can serve as Points of Presence ‹ Foreign Direct Investment (FDI) entails direct
(PoPs) to help people join the NPS. Hence, statement investment, offering control and a long-term
2 is not correct. outlook. Hence, statement 1 is not correct.
z Now, people require only a single Registration for NPS, ‹ Foreign Portfolio Investments (FPI) involves
instead of multiple registrations as earlier required and investing in financial assets without control,
can operate with just one branch with wider digital typically being short to medium-term. Hence,
presence. Hence, statement 3 is correct. statement 2 is not correct.
43. Ans: B ‹ Government’s prior approval is mandatory for
FDIs from countries sharing land border with
Exp:
India (China, Bangladesh,Pakistan,Bhutan,Nepal,
About India’s Civil Aviation Sector : Myanmar and Afghanistan).Hence, statement 3
‹ India is the third-largest aviation market globally, is correct.
trailing behind the United States and China.
Hence, statement 1 is not correct. 46. Ans: C
‹ The unruly passengers were governed under The Exp:
Aircraft Rules, 1937 conjointly read with the Geographical Indication (GI):
Indian Penal Code, 1860. Hence, statement 2 is z A geographical indication (GI) is a sign used on products
correct. that have a specific geographical origin and possess
‹ The Directorate General of Civil Aviation (DGCA) is qualities or a reputation that are due to that origin.
the principal regulatory body that primarily Hence, statement 1 is correct. Hence, statement 1 is
governs civil aviation in India. It is responsible for correct.
dealing with safety issues, regulation of air z GI is a powerful tool for protecting traditional
transport services, enforcement of civil air rules knowledge, culture and can boost socio-economic
and regulations and other such tasks. Hence, development. Hence, statement 2 is correct.
statement 3 is correct. z GI is governed under the Agreement on TRIPS at the
44. Ans: B World Trade Organisation (WTO). Hence, statement 3
Exp: is correct.
z Geographical Indication (GI) is governed under the ‹ The Geographical Indications of Goods
Agreement on TRIPS at the World Trade Organisation (Registration and Protection) Act, 1999 seeks to
(WTO). Hence, statement 1 is correct. provide for the registration and better protection
‹ The Geographical Indications of Goods of geographical indications relating to goods in
(Registration and Protection) Act, 1999 seeks to India.
provide for the registration and better protection 47. Ans: C
of geographical indications relating to goods in Exp:
India.
z Terms of Trade (ToT) for Indian agriculture refers to the
z This certification is also extended to non-agricultural
movement in prices of farm commodities relative to
products, such as handicrafts based on human skills,
non-farm goods and services. Hence, statement 1 is
materials and resources available in certain areas that
correct.
make the product unique. Hence, statement 2 is not
correct. ‹ Terms of trade refers to the ratio of agricultural
prices to industrial prices, measured as price
z Germany leads in GI registrations, with 15,566
indices.
registered products, followed by China (7,247), as per
2020 data with the World Intellectual Property z A rise in the terms of trade implies better purchasing
Organization. Hence, statement 3 is not correct. power for the agricultural sector in terms of industrial
goods. Hence, statement 2 is correct.
45. Ans: B ‹ A ratio above one (or 100%) implies favorable
Exp: pricing power, in terms of what farmers sell
z Foreign investment encompasses two widely chosen versus what they buy.
investment avenues, namely Foreign Direct Investment ‹ A TOT ratio below one indicates unfavorable
(FDI) and Foreign Portfolio Investment (FPI). conditions of exchange.
36 PT SPRINT (2024) ECONOMY QUESTIONS AND ANSWERS www.d rish ti IAS.co m

48. Ans: B ‹ To estimate employment and unemployment


Exp: indicators in both ‘Usual Status’ and CWS in both
z Real Estate Investment Trusts (REITs): These are rural and urban areas annually. Hence, statement
investment vehicles that allow individuals to invest in 3 is correct.
large-scale, income-producing real estate without
51. Ans: A
having to directly manage or own the properties.
Hence, statement I is correct. Exp:
‹ REITs pool capital from multiple investors to Global Stocktake Draft:
invest in a diversified portfolio of real estate z The Global Stocktake (GST) is a periodic review
assets, which may include residential or mechanism established under the Paris Agreement in
commercial properties, shopping centers, office 2015. Hence, statement 1 is correct.
buildings, hotels etc. Hence, statement II is z The fourth draft of the GST text was unveiled at COP28.
correct. z The text proposes eight steps to keep the global
49. Ans: B temperature rise within the ambit of 1.5 degrees
Exp: Celsius. Hence, statement 2 is not correct.
The Finance Commission in India is a constitutional body 52. Ans: C
established under Article 280 of the Indian Constitution. Exp:
The Fifteenth Finance Commission was constituted on 27th
z Countervailing Duty: CVD are tariffs levied on
November, 2017. It made recommendations covering the
imported goods to offset subsidies made to producers
period of six years commencing on 1st April, 2020 through
its Interim and Final Reports. of these goods in the exporting country. Hence,
statement 1 is not correct.
z The Commission proposed maintaining the states’
‹ CVDs are meant to level the playing field
share in central taxes at 41% for the 2021-26 period, a
slight reduction from the 42% allocated during 2015- between domestic producers of a product and
20 by the 14th Finance Commission. Hence, statement foreign producers of the same product who can
1 is not correct. afford to sell it at a lower price because of the
subsidy they receive from their government.
z Population (1971)’ was considered only for the 14th
Finance Commission, while ‘Population (2011)’ and z The World Trade Organization (WTO) permits the
‘Tax and fiscal efforts’ were introduced by the 15th imposition of countervailing duty by its member
Finance Commission. Hence, statement 2 is correct. countries. Hence, statement 2 is correct.
z The Commission recommended that the Centre aims z Directorate General of Trade Remedies (DGTR) under
to limit its fiscal deficit to 4% of GDP by 2025-26. the Ministry of Commerce & Industry, is the single
Hence, statement 3 is correct. national authority for administering all trade remedial
measures including anti-dumping, countervailing
50. Ans: A duties and safeguard measures. Hence, statement 3 is
Exp: correct.
Periodic Labour Force Survey(PLFS):
53. Ans: B
z About:
Exp:
‹ It is a survey conducted by the NSO under the
Ministry of Statistics and Programme Common Reporting Standard (CRS):
Implementation (MoSPI) to measure the z The CRS was developed in response to the G20
employment and unemployment situation in request and approved by the OECD Council on 15th
India. Hence, statement 1 is not correct. July 2014.
‹ The NSO launched the PLFS in April 2017. Hence, z It calls on jurisdictions to obtain information from
statement 2 is correct. their financial institutions and automatically exchange
z Objective of PLFS: that information with other jurisdictions on an annual
‹ To estimate the key employment and basis. Hence, statement 1 is correct.
unemployment indicators (viz. Worker Population z India’s participation in the Tax Inspectors Without
Ratio, Labour Force Participation Rate, Borders (TIWB) programme for strengthening Saint
Unemployment Rate) in the short time interval of Lucia’s (an island country in the Caribbean) tax
three months for the urban areas only in the administration marks a significant step in international
‘Current Weekly Status’ (CWS). cooperation in tax matters.
w w w. d r i s h t i I A S.c om PT SPRINT (2024) ECONOMY QUESTIONS AND ANSWERS 37
z The focus is on strengthening Saint Lucia’s tax 56. Ans: C
administration by transferring technical knowledge Exp:
and skills, specifically emphasizing the effective use of
About Special Purpose Vehicle (SPV) :
OECD’s Automatic Exchange of Information (AEOI)
under the CRS framework. Hence, statement 2 is not z A special purpose vehicle is a separate legal entity
correct. which has all the attributes of a corporation such as
owning assets, capacity to and get sued etc.
54. Ans: A
z A SPV, also known as a special purpose entity (SPE), is
Exp:
like a separate company created by a main company to
z Status of the Telecom Sector in India :
protect against financial risks.
‹ The Telecom industry in India is the second
largest in the world with a subscriber base of z Even if the main company faces bankruptcy, the SPV’s
1.179 Billion as of August 2023 (wireless + wireline independent legal status ensures that its obligations
subscribers). Hence, statement 1 is not correct. remain secure.
‹ It is also the 4th largest sector in terms of Foreign ‹ This is why an SPV is often referred to as a
Direct Investment (FDI) inflows, contributing 6% bankruptcy-remote entity. Hence, statement 1 is
of total FDI inflow. Hence, statement 2 is correct. correct.
‹ India has an overall teledensity of 84.69%. Tele- z An SPV can be employed to take on a risky project,
density denotes the number of telephones per lessening any potential financial harm to the main
100 population and is an important indicator of
company and its investors.
telecom penetration. Hence, statement 3 is not
correct. z Venture capitalists use SPVs to gather funds and invest
in a startup. Hence, statement 2 is correct.
55. Ans: C
Exp: 57. Ans: C
Social Stock Exchange (SSEs): Exp:
z The idea of the Social stock exchanges (SSEs) as a The Union government is planning to revive tax
platform for listing social enterprise, voluntary and remission benefits on sugar exports, which were placed in
welfare organizations so that they can raise capital was the ‘restricted’ category in mid-2022.
mooted in the Union Budget 2019-20. z The Remission of Duties and Taxes on Export Products
‹ Social enterprise can be defined as a non-loss; (RoDTEP) scheme, which was launched in January
non-dividend paying company created and 2021, aims to provide a mechanism for reimbursement
designed to address a social problem. Hence,
of taxes, duties and levies that are incurred by the
statement 1 is not correct.
export entities in the process of manufacture and
z It works under the market regulator Security and
distribution of exported products.
Exchange Board of India (SEBI). Hence, statement 2 is
correct. z RoDTEP covers taxes, duties, and levies at the central,
‹ The aim of the initiative is to help social and state, and local levels, which are not refunded through
voluntary organizations which work for social any other existing mechanisms.Hence, statement 1 is
causes to raise capital as equity or debt or a unit correct.
of mutual fund. ‹ The RoDTEP Committee operates within the
z It provides new and cheaper sources of financing for Department of Revenue, Ministry of Commerce &
social welfare projects, while showcasing India’s Industry.Hence, statement 2 is correct.
independence from foreign aid .
z However, under this scheme, sugar exporters faced a
z SEBI had permitted social enterprises registered on
tax query from the customs authorities, as it is a
SSEs to raise funds through Zero Coupon Zero Principal
restricted commodity and is not eligible for export
Bonds (ZCZP). Hence, statement 3 is correct.
benefits.
‹ The ZCZP bonds do not give any interest, and
investors will not get any money back on the ‹ Industry challenges arose as sugar, previously
maturity of the bond. under free export, sought RoDTEP benefits.
38 PT SPRINT (2024) ECONOMY QUESTIONS AND ANSWERS www.d rish ti IAS.co m

58. Ans: D z Category I: AIFs that invest in startups, early-stage


Exp: ventures, social initiatives, Small and Medium
z Demand-Pull Inflation: Demand Pull inflation occurs Enterprises (SMEs), infrastructure, or sectors deemed
when the demand for goods and services exceeds socially and economically beneficial by authorities.
their supply. When the overall demand in the ‹ This includes venture capital, social venture
economy is high, consumers are willing to pay more funds, infrastructure funds, and any other
for the available goods and services, leading to a specified Alternative Investment Funds. Hence,
general rise in prices. Hence,statement 1 is correct. statement 3 is not correct.
‹ A booming economy with high consumer 60. Ans: D
spending can create excess demand, putting
Exp:
upward pressure on prices.
z The Life Insurance Corporation (LIC) of India in
z Cost-Push Inflation: Cost-push inflation is driven by an
alignment with the ‘Insurance For All by 2047’
increase in the production costs for goods and
services. This can be caused by factors such as initiative, is set to play a pivotal role by introducing
increased incomes, increased costs of raw materials, tailored products for rural areas and embracing digital
or disruptions in the supply chain. Hence, statement 2 transformation. Hence, Statement 1 is correct.
is correct. z The focus is on extending insurance coverage to
z Structural inflation: This occurs when there is a maximum rural masses, aligning with the vision of
mismatch between demand and supply, which leads making India a developed nation by 2047. Hence,
to an increase in the general price level. This can Statement 2 is correct.
happen due to factors such as a shortage of goods, an z LIC acknowledges the Insurance Regulatory and
increase in population, or a decrease in productivity. Development Authority of India (IRDAI) proposed
Hence, statement 3 is correct. ‘Bima Vistar,’ a composite product covering life, health,
z Built-In or Wage-Price Inflation: This type of inflation and property insurance. Hence, Statement 3 is correct.
is often described as a feedback loop between wages ‹ The distribution channel for these products,
and prices. When workers demand higher wages, known as ‘Bima Vahak,’ will be women-centric, in
businesses may raise prices to cover the increased line with the proposed guidelines for dedicated
labor costs. This, in turn, prompts workers to seek distribution channels at the Gram Panchayat
higher wages, and the cycle continues. Hence, level.
statement 4 is correct.
61. Ans: B
‹ Collective bargaining by labor unions can result in
Exp:
higher wages, leading to increased production
costs and subsequently higher prices for goods About the Financial Stability Board (FSB) :
and services. z The FSB is an international body that monitors and
makes recommendations about the global financial
59. Ans: B
system.
Exp:
z FSB was established in 2009 under the aegis of G20.
Alternative Investment Fund (AIF): Hence, statement 1 is not correct.
z An AIF refers to a fund established or formed in India, z India is an active Member of the FSB having three
serving as a privately pooled investment mechanism. seats in its Plenary represented by Secretary of
Hence, statement 1 is correct. Economic Affairs, Ministry of Finance, the Deputy
‹ It gathers funds from sophisticated investors, Governor of the Reserve Bank of India, and Chairperson
whether domestic or international, with the aim Securities and Exchange Board of India. Hence,
of investing according to a specific investment statement 2 is correct.
policy, ultimately benefiting its investors.
z These investment vehicles adhere to the SEBI
62. Ans: A
(Alternative Investment Funds) Regulations, 2012. Exp:
Hence, statement 2 is correct. z UBI is a social welfare proposal in which all the
‹ As of December, 2023, 1,220 AIFs were registered beneficiaries regularly receive a guaranteed income in
with the Securities and Exchange Board of India the form of an unconditional transfer payment. Hence,
(SEBI). statement 1 is correct.
w w w. d r i s h t i I A S.c om PT SPRINT (2024) ECONOMY QUESTIONS AND ANSWERS 39
z UBI is typically intended to reach all (or a very large 65. Ans: C
portion of the population) with no (or minimal) Exp:
conditions. Status of India’s Silk Industry
z Article 38 states that,State to secure a social order for
z Karnataka emerged as the leading state in India’s silk
the promotion of welfare of the people. But it doesn’t
production, contributing 32% in the fiscal year 2021-
mention UBI. Hence, statement 2 is not correct.
22. Hence, statement 1 is not correct.
63. Ans: C z India is the world’s second-largest producer of raw silk
Exp: after China, producing 33,739 MT in the fiscal year
z About: 2020-21. Hence, statement 2 is correct.
‹ The only conceptual difference between the z The Central Silk Board (CSB) is responsible for the
Worker Productivity and Labor Productivity is that overall development and promotion of the sericulture
the ‘work’ in worker productivity describes and silk industry in India. The CSB was established in
mental activities while the ‘work’ in labour 1948 and operates under the administrative control of
productivity is mostly associated with manual the Ministry of Textiles, Government of India. Hence,
activities. Hence, statement 2 is correct. statement 3 is correct.
z Productivity:
66. Ans: C
‹ Productivity of an activity is usually measured as
the quantum of output value per unit of labour Exp:
(time) cost at a micro level. z A tax haven is generally an offshore country that offers
€ At a macro level, it is measured in terms of foreign individuals and businesses little or no tax
the labor-output ratio or change in Net liability in a politically and economically static
Domestic Product (NDP) per worker in each environment. Hence, statement 1 is correct.
sector (where working hours are assumed to z An offshore company is incorporated in a jurisdiction
be 8 hours per day). Hence, statement 1 is other than its home country. Hence, statement 2 is
correct. correct.
64. Ans: B ‹ The main purpose of setting up an offshore
Exp: company is to take advantage of the favourable
Non-Banking Finance Companies (NBFCs): tax laws or economic environment in a foreign
country.
z An NBFC, registered under the Companies Act, 1956,
engages in various financial activities such as loans, 67. Ans: B
investments in securities, leasing, insurance. Exp:
z It excludes institutions whose primary business falls
Investor Risk Reduction Access (IRRA):
under agriculture, industry, goods trading, services,
or immovable property trading. z The IRRA platform has been developed to reduce risks
faced by investors in the eventuality of technical
z NBFCs are restricted from accepting demand deposits
glitches at the trading member’s end at both the
from the public, unlike banks, which commonly accept
these types of deposits that can be withdrawn on primary site and disaster recovery site. Hence,
demand without prior notice. Hence, statement 1 is statement 1 is correct.
correct. z Its purpose is to provide investors with an opportunity
z Unlike banks, NBFCs do not form part of the payment to square off/close their open positions and cancel
and settlement system. They are unable to issue pending orders using the IRRA platform in case of
cheques drawn on themselves, a standard practice technical glitches or unforeseen outages that render
offered by banks. Hence, statement 2 is not correct. the trading member’s site inaccessible.
z NBFCs primarily finance their operations through a mix z It is not meant for taking fresh positions or orders, but
of market borrowing and bank loans. Hence, only to cancel the pending orders. Hence, statement 2
statement 3 is correct. is not correct.
40 PT SPRINT (2024) ECONOMY QUESTIONS AND ANSWERS www.d rish ti IAS.co m

z Developed By: z Adjudicating authority:


‹ IRRA has been jointly developed by all the stock ‹ National Company Law Tribunal (NCLT) for
exchanges – BSE (Bombay Stock Exchange), NSE companies and LLPs
(National Stock Exchange), NCDEX (National ‹ Debt Recovery Tribunal (DRT) for individuals and
Commodity and Derivatives Exchange), MCX partnership firms
(Multi Commodity Exchange) and Metropolitan 70. Ans: D
Stock Exchange of India (MSE). Hence, statement Exp:
3 is not correct. z De-dollarisation refers to a deliberate or unintentional
68. Ans: C process undertaken by a country or region to reduce
reliance on the US dollar in its financial system or
Exp:
economy. Hence, Statement 1 is correct.
Coal in India: ‹ This can involve various measures aimed at
z Distribution of Coal in India: decreasing the use of the dollar in transactions,
z Gondwana Coal Fields (250 million years old): reserves, trade, or as a standard for pricing goods
‹ Gondwana coal makes up to 98% of the total and services. Hence, Statement 2 is correct.
reserves and 99% of the production of coal in z Reasons Associated: Governments might pursue de-
India. Hence, statement 1 is correct. dollarisation for several reasons, such as reducing
exposure to the impact of US monetary policy,
‹ Gondwana coal forms India’s metallurgical grade
asserting economic sovereignty, mitigating the effects
as well as superior quality coal. Hence, statement of dollar fluctuations, or seeking greater independence
2 is correct. in global finance.
‹ It is found in Damodar (Jharkhand-West Bengal), z Strategies for De-dollarisation: It can include
Mahanadi (Chhattisgarh-Odisha), Godavari diversifying currency reserves, promoting the use of
(Maharashtra), and Narmada valleys. alternative currencies in trade agreements,
z Tertiary Coal Fields (15 – 60 million years old): establishing currency swap agreements, or
‹ Carbon content is very low but is rich in moisture encouraging the use of regional currencies in financial
and Sulphur. transactions. Hence, Statement 3 is correct.
‹ Tertiary coalfields are mainly confined to extra- 71. Ans: A
peninsular regions Exp:
‹ Important areas include Assam, Meghalaya, z Fake loan apps are unauthorized and illegal digital
Nagaland, Arunachal Pradesh, Jammu and lending platforms that offer loans, from as little as
Kashmir, the Himalayan foothills of Darjeeling in Rs.1,000 to Rs.1 lakh targeting low-income and
West Bengal, Rajasthan, Uttar Pradesh, and financially untrained individuals.
Kerala. ‹ They claim to provide instant and hassle-free
loans without any credit checks, documentation,
69. Ans: C or collateral. Hence, Statement 1 is correct.
Exp: z Stakeholders highlight the absence of government and
z Insolvency and Bankruptcy Code, 2016: regulatory norms, enabling online platforms to conduct
‹ Insolvency and Bankruptcy Code, 2016 provides a minimal due diligence.
time-bound process for resolving insolvency in ‹ Different regulators like the Reserve Bank of India
companies and among individuals. Hence, (RBI), Ministry of Electronics and Information
statement 1 is correct. Technology (MeitY), Telecom Regulatory
Authority of India (TRAI), and state governments
‹ The Code is quite different from the earlier
lack coordination and supervision. Hence,
resolution systems as it shifts the responsibility
Statement 2 is not correct.
to the creditor to initiate the insolvency resolution
z While the RBI released guidelines for digital lending in
process against the corporate debtor. Hence,
September 2022, these guidelines only apply to
statement 2 is correct. regulated entities like banks and NBFCs. Fraudulent
‹ It Covers all individuals, companies, Limited apps, without such associations, remain largely
Liability Partnerships (LLPs) and partnership firms. unchecked. Hence, Statement 3 is not correct.
w w w. d r i s h t i I A S.c om PT SPRINT (2024) ECONOMY QUESTIONS AND ANSWERS 41
72. Ans: B 75. Ans: A
Exp: Exp:
Sovereign Gold Bond (SGB): Fair Price Shop (FPS):
z SGBs are government securities denominated in z FPS is a government-run or government-regulated
grams of gold. retail outlet or store in India. Hence, statement 1 is
correct.
z They are substitutes for holding physical gold.
‹ The primary purpose of fair price shops is to
Investors have to pay the issue price in cash and the distribute essential commodities like food
bonds will be redeemed in cash on maturity. grains, edible oils, sugar, and other basic
z The Bond is issued by the Reserve Bank on behalf of necessities to the public at subsidized or fair
the Government of India. Hence, statement 1 is not prices.
correct. ‹ These shops are typically part of government
z Persons resident in India as defined under the Foreign welfare programs aimed at ensuring food
Exchange Management Act, 1999 are eligible to invest security and reducing the economic burden on
low-income households. Hence, statement 3 is
in SGB. Hence, statement 2 is correct.
not correct.
z Eligible investors include individuals, HUFs, trusts,
€ This system has a robust mechanism for
universities, and charitable institutions. Individual verification of beneficiaries through Aadhaar
investors with subsequent change in residential status authentication and has features to monitor
from resident to non-resident may continue to hold transactions online with the help of
SGB till early redemption/maturity. electronic Point of Sale (e-POS) machines.
€ The e-PoS devices have been integrated
73. Ans: C
with electronic weighing machines to ensure
Exp: the beneficiaries get the correct amount of
z An IFSC is a financial centre that caters to customers ration. Hence, statement 2 is correct.
outside the jurisdiction of the domestic economy. € These FPSs and ePOS machines have proved
Hence, statement 1 is correct. instrumental in the effectuation and seamless
z The IFSC in India is regulated by the International implementation of the One Nation One
Ration Card Scheme (ONORC).
Financial Services Centres Authority (IFSCA), a
statutory authority that was established under the 76. Ans: C
International Financial Services Centres Authority Exp:
Act, 2019. Hence, statement 2 is correct. z A wilful defaulter means a borrower or a guarantor
‹ It is headquartered at GIFT City, Gandhinagar in who has committed wilful default and the outstanding
Gujarat. amount is Rs 25 lakh and above. Hence, statement 1 is
correct.
z At present, the GIFT IFSC is the maiden IFSC in India.
z A large defaulter refers to a borrower with an
z In IFSC, all transactions must be in foreign currency
outstanding balance of Rs 1 crore or more, whose
(except INR). However, administrative and statutory account has been categorized as doubtful or a loss.
expenses can be conducted in INR. Hence, statement 2 is correct.
74. Ans: B 77. Ans: D
Exp: Exp:
z Under the FDI policy, e-commerce comprises products, Geographical Indication (GI) tag
both digital and physical, and services traded on z A geographical indication (GI) tag is a name or sign
digital and electronic networks. Hence, statement 1 is used on certain products that correspond to a specific
correct. geographical location or origin.
z Business-to-consumer (B2C) is the retail part of z Geographical indications are recognized as a part of
e-commerce on the Internet, that is, the sale of goods intellectual property rights (IPRs) under Articles 1(2)
and 10 of the Paris Convention and also under Articles
and/or services to the end consumer through digital
22 to 24 of the Trade-Related Aspects of Intellectual
means. Hence, statement 2 is correct. Property Rights (TRIPS) Agreement. Hence, statement
Hence, option B is correct. 1 is correct.
42 PT SPRINT (2024) ECONOMY QUESTIONS AND ANSWERS www.d rish ti IAS.co m

z India, as a World Trade Organisation (WTO) member, z Initially, angel tax was applicable only to investments
implemented the Geographical Indications of Goods made by resident investors. The Finance Act, 2023
Act, 1999, which became effective on September 15, extended this provision to include foreign investors as
2003, to protect such indications. Hence, statement 2 well. Hence, statement 2 is not correct.
is correct. z In May 2023, the Finance Ministry exempted investors
z Arunachal Pradesh has recently received the from 21 countries, such as the US, UK, and France,
Geographical Indication (GI) tag for Arunachal Yak from the angel tax levy for non-resident investments in
Indian start-ups. Hence, statement 3 is correct.
Churpi, Khaw Tai (Khamti rice), and Tangsa Textiles.
Hence, statement 3 is correct. 81. Ans: C
78. Ans: B Exp:
z The Vizhinjam International Transhipment Deepwater
Exp:
Multipurpose Seaport is an ambitious project taken up
z Consumer Food Price Inflation, (CFPI), is a specific by the Government of Kerala. Hence, statement 1 is
measure of inflation that focuses exclusively on the correct.
price changes of food items in a consumer’s basket of z The port is currently being developed with a Public
goods and services. Private Partnership with a component structured on a
z It calculates the rate at which the prices of food design, build, finance, operate, and transfer (“DBFOT”)
products consumed by the average household are basis. Hence, statement 2 is correct.
increasing over time. z It is positioned to compete with global transshipment
z CFPI is a sub-component of the broader Consumer hubs like Colombo, Singapore, and Dubai, reducing
Price Index (CPI), where the Reserve Bank Of India the cost of container movement. Hence, statement 3
(RBI) uses CPI-Combined (CPI-C) for this purpose. is correct.
Hence, statement 1 is not correct. 82. Ans: B
z CFPI tracks the price changes of a specific basket of Exp :
food items that are commonly consumed by z MSP is the guaranteed amount paid to farmers when
households, such as cereals, vegetables, fruits, dairy the government buys their produce.MSP is based on
products, meat, and other food staples. Hence, the recommendations of the Commission for
statement 2 is correct. Agricultural Costs and Prices (CACP), which considers
various factors such as cost of production, demand and
79. Ans: C
supply, market price trends, inter-crop price parity, etc.
Exp: ‹ CACP is an attached office of the Ministry of
Navratna Comapnies: Agriculture and Farmers Welfare. It came into
z The Navratna status was first introduced in 1997. existence in January 1965.
There are currently 16 Navratna Companies in India. z The Cabinet Committee on Economic Affairs (CCEA)
Hence, statement 1 is correct. chaired by the Prime Minister of India takes the final
z To attain Navratna status, a firm must initially hold the decision (approve) on the level of MSPs. Hence,
statement 1 is not correct.
Miniratna designation and it needs to achieve a score
of 60 or more (out of 100) according to six performance z The Commission for Agricultural Costs and Prices
(CACP) recommends MSPs for 22 mandated crops and
criteria, which include metrics like Net Profit to Net
fair and remunerative price (FRP) for sugarcane.The
Worth, Earnings per Share, and Inter-Sectoral
mandated crops include 14 crops of the kharif season,
Performance. Hence, statement 1 is correct.
6 rabi crops and 2 other commercial crops.Hence,
80. Ans: B statement 2 is correct.
Exp: 83. Ans: C
Angel Tax: Exp:
z Angel tax is an income tax levied at the rate of 30.6% Multilateral Development Banks (MDBs):
when an unlisted company issue shares to an investor z MDBs are international institutions comprising
at a price higher than its fair market value. Hence, developed and developing countries. Hence,
statement 1 is not correct. statement 1 is correct.
w w w. d r i s h t i I A S.c om PT SPRINT (2024) ECONOMY QUESTIONS AND ANSWERS 43
z They offer financing and technical assistance for development of both low-income and middle-income
various projects in areas like transportation, energy, countries (LICs and MICs) by addressing issues such as
urban infrastructure, and waste management. poverty reduction, infrastructure development, human
z Developed countries contribute to MDB lending, while capital formation, etc.
developing nations typically borrow from them for z MDBs include the World Bank Group, the Asian
development projects. Hence, statement 2 is correct. Development Bank, the African Development Bank,
z MDBs have been instrumental in supporting the the Inter-American Development Bank, etc.

84. Ans: A
Exp:
Aspect Capital Expenditure Revenue Expenditure
Expenses related to acquiring, improving, or extending
Day-to-day operational expenses incurred for
Nature of long-term assets or investments expected to benefit for
maintaining and supporting existing assets
Expenses more than one financial year.
or services.
Hence, statement 1 is correct.
Accounting Capitalized on the balance sheet and recognized over time Fully recognized as expenses in the year
Treatment through amortization or depreciation. incurred on the income statement.
Subject to depreciation or amortization, leading to a Immediately deductible from taxable income,
Tax Treatment delayed tax impact and often lower taxable income in the providing an immediate reduction in tax
year of purchase.Hence, statement 3 is correct. liability.
Has an immediate impact on profitability,
Impact on Generally does not significantly impact short-term
as expenses are fully recognized in the year
Profitability profitability as costs are spread over several years.
incurred.
Acquiring a new manufacturing facility, research and Routine machinery maintenance, employee
Examples development for a new product, long-term license or salaries, advertising costs, utility bills.Hence,
franchise. statement 2 is not correct.

85. Ans: C ‹ India is not a member, but a key economic partner.


Exp: Hence, statement 2 is correct.
Organisation for Economic Co-operation and Development z Reports and Indices by OECD:
(OECD): ‹ Government at a Glance. Hence, statement 3 is
z About: correct.
‹ The OECD is an intergovernmental economic ‹ OECD Better Life Index.
organisation, founded to stimulate economic 86. Ans: B
progress and world trade.
Exp:
‹ Most OECD members are high-income economies
z Bidenomics is a wordplay on Reaganomics, the
with a very high Human Development Index
nickname for the Reagan administration’s economic
(HDI) and are regarded as developed countries.
policies, which emphasized four pillars: tax cuts,
Hence, statement 1 is correct.
deregulation, domestic spending cuts and inflation
z Foundation:
reduction.
‹ It was founded in 1961 with its Headquarters at
z Bidenomics, according to the White House, is a
Paris, France and total membership is 38 rejection of the trickle-down economic policies that
countries. defined Reaganomics.
‹ The most recent countries to join the OECD were ‹ It is a vision centred around three key pillars:
Colombia, in April 2020, and Costa Rica, in May making smart public investments; empowering
2021. and educating workers to grow the middle class;
44 PT SPRINT (2024) ECONOMY QUESTIONS AND ANSWERS www.d rish ti IAS.co m

and promoting competition to lower costs and z DRI serves as India’s premier anti-smuggling agency,
help entrepreneurs and small businesses thrive. operating under the Central Board of Indirect Taxes &
z Hence, option B is correct. Customs, Ministry of Finance. Hence, statement 1 is
correct.
87. Ans: C
z It is responsible for detecting and preventing various
Exp: forms of smuggling, including illicit wildlife trade.
Global Minimum Tax (GMT): Hence, statement 2 is correct.
z A GMT applies a standard minimum tax rate to a
defined corporate income base worldwide. Hence, 90. Ans: D
statement 1 is correct. Exp:
z The Organization for Economic Cooperation and Fiscal Deficit:
Development (OECD) developed a proposal featuring z About:
a corporate minimum tax of 15% on foreign profits of ‹ Fiscal deficit is the difference between the
large multinationals, which would give countries new government’s total expenditure and its total
annual tax revenues of USD 150 billion. revenue (excluding borrowings). Hence,
z In October 2021, a group of 136 countries, including statement 1 is not correct.
India, set a minimum global tax rate of 15% for MNCs ‹ It is an indicator of the extent to which the
and sought to make it harder for them to avoid government must borrow in order to finance its
taxation. Hence, statement 3 is correct. operations and is expressed as a percentage of
z The framework of GMT aims to discourage nations the country’s GDP.
from tax competition through lower tax rates that
z High and Low FD:
result in corporate profit shifting and tax base erosion.
Hence, statement 2 is correct. ‹ A high fiscal deficit can lead to inflation,
devaluation of the currency and an increase in
88. Ans: B the debt burden.
Exp: ‹ While a lower fiscal deficit is seen as a positive
Monetary Policy Committee (MPC): sign of fiscal discipline and a healthy economy.
z Section 45ZB of the amended RBI Act, 1934 provides Hence, statement 2 is not correct.
for an empowered six-member monetary policy
91. Ans: A
committee (MPC) to be constituted by the Central
Government by notification in the Official Gazette. Exp:
Hence, statement 1 is correct. z The impossible trinity, or the trilemma, refers to the
z The MPC determines the policy repo rate required to idea that an economy cannot pursue independent
achieve the inflation target. Hence, statement 2 is monetary policy, maintain a fixed exchange rate, and
correct. allow the free flow of capital across its borders all at
z MPC shall consist of 6 members:
the same time.
‹ RBI Governor as its ex officio chairperson, Hence, z In a fixed exchange rate regime, the domestic currency
statement 3 is not correct. is tied to another foreign currency such as the US
dollar, Euro, the Pound Sterling or a basket of currencies
‹ Deputy Governor in charge of monetary policy,
z An able policymaker can, at best, achieve two of these
‹ An officer of the Bank to be nominated by the
Central Board, three objectives at any given time.
‹ Three persons to be appointed by the central ‹ Hence, option a is correct.
government. 92. Ans: A
89. Ans: D Exp:
Exp: z CBDCs are a digital form of paper currency and unlike
cryptocurrencies that operate in a regulatory vacuum,
z In a recent operation named “Kachchhap,” the
these are legal tenders issued and backed by a central
Directorate of Revenue Intelligence (DRI) has
successfully rescued nearly a thousand live baby bank.
Gangetic turtles, shedding light on the ongoing battle ‹ Hence, statement 1 is correct.
against illegal wildlife trade and the protection of z It is the same as a fiat currency and is exchangeable
these unique creatures. one-to-one with the fiat currency.
w w w. d r i s h t i I A S.c om PT SPRINT (2024) ECONOMY QUESTIONS AND ANSWERS 45
‹ A fiat currency is a national currency that is not ‹ The Harrod-Domar model, created by economists
pegged to the price of a commodity such as gold Roy Harrod and Evsey Domar, asserts that
or silver. economic growth relies on the availability of
‹ Hence, statement 2 is not correct. capital for investment, and the rate of capital
z Bahamas has been the first economy to launch its accumulation is directly linked to the rate of
nationwide CBDC — Sand Dollar in 2020. savings.
‹ Hence, statement 3 is not correct. z A lower ICOR signifies greater efficiency and
productive use of capital. Hence, statement 2 is
93. Ans: B correct.
Exp: z According to an SBI report, India is experiencing an
India’s first ever UPI-ATM upward trend in savings and investments, which is
accompanied by a simultaneous decrease in the ICOR.
z India’s first UPI-ATM was launched on September 5th, ‹ The current ICOR in India stands at 4.4, indicating
2023, by Hitachi Payment Services. Hence, statement a reasonably efficient use of capital.
1 is correct.
96. Ans: C
z The UPI-ATM is a White Label ATM (WLA) developed in
collaboration with the National Payments Corporation Exp:
of India (NPCI). Hence, statement 2 is not correct. z Gresham’s Law is related to Currency Exchange Rate.
z The ATM was designed to enable “seamless cash This law is a monetary principle that states “bad
withdrawals” and eliminate the need to carry physical money drives out good.”
ATM cards. It also aims to drive financial inclusion by z It means that when there are two types of money in
making banking services more accessible in areas with circulation, one with a higher intrinsic value and one
limited traditional banking infrastructure. It offers an with a lower intrinsic value, people tend to hoard the
experience that allows customers of certain banks to more valuable money and spend the less valuable
enjoy “QR-based cashless withdrawals”. Hence, money. Hence, option C is correct.
statement 3 is correct. ‹ As a result, the less valuable money dominates
the market, while the more valuable money
94. Ans: C
disappears from circulation.
Exp:
z This phenomenon comes into play when the
z Global Fintech Fest (GFF) is the largest fintech government fixes the exchange rate between two
conference, jointly organized by the National Payments currencies, creating a disparity between the official
Corporation of India (NPCI), the Payments Council of rate and the market rate.
India (PCI), and the Fintech Convergence Council ‹ In the 2022 economic crisis in Sri Lanka, Gresham’s
(FCC). Hence, statement 1 is correct. Law played a significant role when the Central
z Its aim is to provide a singular platform for fintech Bank of Sri Lanka implemented a fixed exchange
leaders to foster collaborations and develop a rate between the Sri Lankan Rupee and the U.S.
blueprint for the future of the industry. Hence, Dollar, leading to unintended consequences in
statement 2 is correct. the currency market.
z The theme of GFF’ 2023 is “Global Collaboration for a
Responsible Financial Ecosystem”. Hence, statement 97. Ans: C
3 is correct. Exp:
z India is currently the world’s fifth largest producer of
95. Ans: C
natural rubber while it also remains the second biggest
Exp: consumer of the material globally.(About 40% of
Incremental Capital Output Ratio (ICOR): India’s total natural rubber consumption is currently
z It has evolved from the Harrod-Domar Growth Theory met through imports)
and examines the relationship between fresh ‹ As of now, India boasts approximately 8.5 lakh
investments and economic growth, indicating how hectares of rubber plantations.
much additional capital is required to generate a 1% z Major rubber producing states include: Kerala, Tamil
higher output. Hence, statement 1 is correct. Nadu, Tripura, Assam.
46 PT SPRINT (2024) ECONOMY QUESTIONS AND ANSWERS www.d rish ti IAS.co m

z The lion’s share of this rubber farming, nearly 5 lakh 100. Ans: C
hectares, is concentrated in the southern states of Exp:
Kerala and the Kanyakumari district of Tamil Nadu. z According to ‘Basic Animal Husbandry Statistics 2022’,
z Additionally, Tripura contributes around 1 lakh total milk production in India was 221.06 million
hectares to the rubber production landscape tonnes in 2021-2022, keeping it the largest milk
98. Ans: A producing country in the world. Hence, statement 1 is
Exp: not correct.
z Top five major milk producing States are Rajasthan
Insurance Regulatory and Development Authority of India
(IRDAI): (15.05%), Uttar Pradesh (14.93%), Madhya Pradesh
(8.06%), Gujarat (7.56%) and Andhra Pradesh (6.97%).
z IRDAI is a statutory body under the IRDAI Act, 1999
Hence, statement 2 is not correct.
and is under the jurisdiction of the Ministry of
Finance. z In 2022, a study published in ‘Lancet’ estimated that
increasing temperatures could reduce milk production
‹ Hence, Statements 1 & 2 are not correct.
in India’s arid and semi-arid regions by 25% by 2085.
z IRDAI, founded in 1999, is a regulatory body created
Hence, statement 3 is correct.
with the aim of protecting the interests of insurance
customers. 101. Ans: B
z It regulates and sees to the development of the Exp:
insurance industry while monitoring insurance-related z Indian pharmaceutical industry is the 3rd largest
activities. Hence, Statement 3 is correct. pharmaceutical industry in the world by volume with
99. Ans: D current market size of around USD 50 Billions. Hence,
Exp: statement 1 is correct.
z India’s GDP is calculated using two distinct methods: z Recently, the Ministry of Chemicals and Fertilizers has
Economic activity (at factor cost) and Expenditure (at launched initiatives to nurture Innovation, research,
market prices).Hence, statement 1 is not correct. and development in the Medical and Pharmaceutical
sectors.
z The factor cost method assesses the performance of
eight different industries. ‹ These Initiatives are National Policy on Research
‹ The following eight industry sectors are and Development and Innovation in Pharma-
considered in this cost: MedTech Sector in India and Scheme for
Promotion of Research and Innovation in Pharma
€ Agriculture, forestry, and fishing,
MedTech Sector (PRIP). Hence, statement 2 is
€ Mining and quarrying,
correct.
€ Manufacturing,
€ Electricity, gas, water supply, and other utility 102. Ans: C
services, Exp:
€ Construction, Surety Bonds:
€ Trade, hotels, transport, communication, and z A surety bond can be defined in its simplest form as a
broadcasting, written agreement to guarantee compliance,
€ Financial, real estate, and professional payment, or performance of an act.It is a unique type
services, of insurance because it involves a three-party
€ Public administration, defense, and other agreement. Hence, statement 2 is correct.
services. z Surety bond is provided by the insurance company on
Hence, statement 2 is not correct behalf of the contractor to the entity that is awarding
z The expenditure-based method indicates how the project. It will help contractors to have financial
different areas of the economy are performing, such as closure of their projects without depending upon only
trade, investments, and personal consumption.Hence, bank guarantees.
statement 3 is not correct. Hence, statement 1 is correct.
w w w. d r i s h t i I A S.c om PT SPRINT (2024) ECONOMY QUESTIONS AND ANSWERS 47
103. Ans: B 105. Ans: D
Exp: Exp:
Foreign Direct Investment (FDI): Cash Reserve Ratio (CRR):
z FDI is a type of cross-border investment in which an z The percentage of cash required to be kept in reserves
investor from one country establishes a lasting interest as against the bank’s total deposits, is called CRR.
in an enterprise in another country. Hence, statement 1 is correct.
‹ An Outward Direct Investment (ODI) is a business z All banks in India, except Regional Rural Banks (RRBs)
strategy in which a domestic firm expands its and Local Area Banks (LABs), have to deposit the CRR
money to RBI. Hence, statement 2 is correct.
operations to a foreign country. Hence, statement
‹ RRBs and LABs are exempted from maintaining
1 is not correct.
CRR with RBI, as per the RBI Act, 1934. However,
z FDI can take various forms, such as acquiring shares,
they have to maintain CRR with themselves in the
establishing a subsidiary or a joint venture, or providing form of cash or gold or unencumbered approved
loans or technology transfers. Hence, statement 2 is securities.
correct. z Banks can’t lend the CRR money to corporates or
‹ FDI is considered to be a key driver of economic individual borrowers, banks can’t use that money for
growth, as it can bring in capital, technology, investment purposes, and Banks don’t earn any
skills, market access and employment interest on that money. Hence, statement 3 is correct.
opportunities to the host country.
106. Ans: C
104. Ans: D Exp:
Exp: z Cotton is a versatile crop that provides food, feed, and
z Nominal GDP is not adjusted for inflation and is not fiber for various uses, including textiles, cooking oil,
considered a more precise representation of a and livestock feed. It is also a major source of income
country’s economic progress. Hence, statement 1 is and employment for millions of farmers in India. India
is the second largest cotton producer in the world,
not correct.
after China.Hence, statement 1 is correct.
‹ Nominal GDP is the total value of all goods and
‹ However, in recent years, cotton production and
services produced at current market prices over
yields have declined significantly, posing a
a time period, including the effects of inflation or challenge for the country’s agriculture and textile
deflation. sectors. Cotton is one of the most important
‹ Real GDP is a more accurate measure of the sum commercial crops cultivated in India and accounts
of all goods and services produced at constant for around 25% of the total global cotton
prices, making it inflation-adjusted. production.
z The formula for calculating GDP using the expenditure ‹ Due to its economic importance in India, it is also
method is not GDP = Real GDP (GDP at constant termed as “White-Gold”. Hence, statement 2 is
prices) - Taxes + Subsidies. correct.
‹ The correct formula is GDP = C + I + G + NX, where z Traditional insecticides had limited success in
C = consumption, I = investment, G = government controlling PBW larvae. Instead, a different method
spending, and NX = net exports. Hence, statement called “mating disruption” has been used.
2 is not correct. ‹ It entails the use of Gossyplure, a pheromone
signaling chemical that is secreted by female
z GDP calculated based on the income method is not
PBW moths to attract male adults. In this case,
known as GDI, which stands for Gross Domestic
the pheromone is artificially synthesised and
Investment. The correct term is GDI, which stands for filled into pipes or lures.
Gross Domestic Income. Hence, statement 3 is not
€ This method hinders male moths from
correct. locating females and engaging in mating,
‹ Gross Domestic Investment is another component thereby causing disruption in their
of GDP that measures the spending by businesses reproductive cycle. Hence, statement 3 is
on capital equipment, inventories, and housing. correct.
48 PT SPRINT (2024) ECONOMY QUESTIONS AND ANSWERS www.d rish ti IAS.co m

107. Ans: A ‹ In January 2023, India adopted the T+1 settlement


Exp: cycle, where T represents the trade date. Hence,
statement 1 is correct.
z Recently, the Securities and Exchange Board of India
‹ This means that trade-related settlements occur
(SEBI) has announced that it is aiming to launch One-
within one business day or 24 hours of the actual
Hour Settlement of trades by March 2024 in order to
transaction. Hence, statement 2 is not correct.
enhance the efficiency of trade settlement processes.
‹ India became the second country, after China, to
z The SEBI will launch Application Supported by Blocked
implement the T+1 settlement cycle in top-listed
Amount (ASBA)-like facility for trading in the Secondary securities. Hence, statement 3 is not correct.
Market by January 2024.
‹ This transition brought several advantages,
Trade Settlement: including increased operational efficiency, faster
z About: fund transfers, prompt share delivery, and
‹ Trade settlement is a critical process in the improved convenience for participants in the
financial markets that involves the transfer of stock market.
funds and securities between parties involved in a z One-Hour Trade Settlement:
trade. ‹ Under this scheme, when an investor sells a
‹ It ensures that the buyer receives the purchased share, the money from the sale will be credited to
securities, and the seller receives the agreed- their account within one hour, and the buyer will
upon funds. receive the purchased shares in their demat
account within the same time frame.
‹ In the context of securities trading, this settlement
‹ This represents a significant reduction in
process finalizes the transaction.
settlement time compared to the existing T+1
z T+1 Settlement Cycle: cycle.

108. Ans: D
Exp:
z MSME stands for Micro, Small, and Medium Enterprises. The MSME sector of India contributes around 33% of the
country’s total GDP and is predicted to contribute worth USD 1 trillion to India’s total exports by 2028.
z Therefore, all statements are correct.

109. Ans: A
Exp:
AMC Repo Clearing Limited (ARCL)
z ARCL is a Limited Purpose Clearing Corporation that provides clearing and settlement services to all trades made in
corporate debt securities on the National Stock Exchange (NSE) and Bombay Stock Exchange (BSE)
z It received in-principle approval from the Securities and Exchange Board of India (SEBI) under the Stock Exchanges
and Clearing Corporations (SECC) Regulations, 2018.
z ARCL offers triparty repo services and central counterparty services to facilitate repo transactions in corporate
bonds, which are short-term borrowings backed by securities as collateral. Hence, statement 1 is not correct.
w w w. d r i s h t i I A S.c om PT SPRINT (2024) ECONOMY QUESTIONS AND ANSWERS 49
‹ Triparty Repo Services: Triparty repo services are contribution was not consistent. Hence statement 2 is
financial transactions where a third party, often a not correct.
custodian or clearing agent, acts as an
111. Ans: C
intermediary between the two parties involved in
a repurchase agreement (repo). Hence, Exp:
statement 2 is correct. z Recently, the Ministry of Finance launched the scheme,
“Vivad se Vishwas II to effectively settle the pending
110. Ans: A
contractual disputes of government and government
Exp: undertakings. Hence, statement 1 is correct.
z According to the National Statistical Office’s (NSO) ‹ The scheme was announced in the Union Budget
Periodic Labour Force Survey (PLFS) report for the year 2023-24 by the Union Finance Minister.
2021-22, the unemployment rate for 2021-22 was z The Scheme aims to provide a platform for resolving
4.1%.
contractual disputes efficiently and promptly,
‹ Hence statement 1 is correct. promoting ease of doing business with the government
z EPFO’s annual reports show a stagnant or declining and encouraging fresh investments in the country.
number of regular contributors, those with consistent Hence, statement 2 is correct.
PF contributions, in recent years. z It will be implemented through an online functionality
‹ Between 2012 and 2022, the number of regular on the Government e-Marketplace (GeM) portal.
contributors to the EPF increased from 30.9 Hence, statement 3 is correct.
million to 46.3 million.
‹ Between 2017 and 2022, the number of regular 112. Ans: C
contributors increased only from 45.11 million to Exp:
46.33 million, showing a slowdown in growth Sugarcane:
during this period. z Temperature: Between 21-27°C with hot and humid
climate.
z Rainfall: Around 75-100 cm.
z Soil Type: Deep rich loamy soil.
‹ It can be grown on all varieties of soils ranging
from sandy loam to clay loam given these soils
should be well drained.
z Top Sugarcane Producing States: Uttar Pradesh,
Maharashtra, Karnataka, Tamil Nadu, Bihar. Hence,
statement 2 is correct.
‹ India is the second largest producer of sugarcane
after Brazil. Hence, statement 1 is correct.
z It is the main source of sugar, gur (jaggery), khandsari
and molasses.
z Fair and Remunerative Price (FRP) is the price set by
the government that sugar mills are obligated to pay
to farmers for the sugarcane procured from them.
Hence, statement 3 is correct.
113. Ans: D
Exp:
z India ranks 4th globally after China, the United States
and Germany, in terms of installed wind energy
z It is clear from the chart shown above that between capacity, with 42.8 GW (onshore wind) as of April
2017 to 2022 regular contribution increased from 2023. Hence, statement 1 is correct.
2017 to 2019, then decreased. This shows regular z Best Performing States:
50 PT SPRINT (2024) ECONOMY QUESTIONS AND ANSWERS www.d rish ti IAS.co m

‹ Wind Power Potential (in GW) at 120 m Above stability to the country’s economy and help in managing
Ground Level: balance of payments and currency exchange rates. The
€ Gujrat (142.56), Rajasthan (127.75), following are typically considered as part of India’s forex
Karnataka (124.15), Maharashtra (98.21), reserves:
and Andhra Pradesh (74.90). Hence, z Foreign Currencies
statement 2 is correct. z Foreign Currency Assets (FCAs)
z National Offshore Wind Energy Policy: z Gold Reserves
‹ Objective is to develop offshore wind energy in z Special Drawing Rights (SDRs)
the Indian Exclusive Economic Zone (EEZ) along
z Reserve Position in IMF
the Indian coastline of 7600 km. Hence, statement
Foreign Assets Held by Indian Individuals or Businesses
3 is correct.
do not form a part of India’s Forex. Hence, option A is the
114. Ans: C correct answer.
Exp:
117. Ans: C
z Deflation is the opposite of Inflation. It refers to a
Exp:
sustained and general decrease in the overall price
levels of goods and services in the economy. Hence, Production Linked Incentive Scheme (PLI):
statement-I is correct . z The PLI scheme was conceived to scale up domestic
z In a deflationary environment, consumers can buy manufacturing capability, accompanied by higher
more goods and services for the same amount of import substitution and employment generation.
money over time. Hence, statement-II is not correct. z Launched in March 2020, the scheme initially targeted
three industries:
115. Ans: B
‹ Mobile and allied Component Manufacturing
Exp:
‹ Electrical Component Manufacturing and
z Floating rate loans are loans that have an interest rate
‹ Medical Devices.
that changes periodically, depending on a benchmark
rate or the base rate. Hence, statement 1 is correct. z Later, it was extended to 14 sectors.
‹ This base rate, such as the repo rate - rate at z In the PLI scheme, Domestic and Foreign companies
which RBI lends money to financial institutions - is receive financial rewards for manufacturing in India,
influenced by market forces. based on a percentage of their revenue over up to five
z Floating-rate loans are also known as variable or years. Hence, statement 1 is correct.
adjustable-rate loans, as they can vary over the term z The subsidy in the scheme for Mobile and allied
of the loan. Component Manufacturing is paid only for finishing
z Floating rate loans are common for credit cards, the phone in India, not on how much value is added by
mortgages, and other consumer loans. Hence, manufacturing in India. Hence, statement 3 is correct.
statement 2 is correct. z The incentives given, are calculated on the basis of
z Floating rate loans are beneficial to borrowers when incremental sales.
interest rates are expected to drop in the future. ‹ In some sectors such as advanced chemistry cell
‹ In contrast, a fixed interest rate loan requires a batteries, textile products and the drone industry,
borrower to pay set installments during the loan the incentive to be given will be calculated on
tenure. It offers a greater sense of security and the basis of sales, performance and local value
stability in times of fluctuations in the economy. addition done over the period of five years.
z They are at least 1-2% cheaper than the fixed interest Hence, statement 2 is correct.
loans. Hence, statement 3 is not correct. 118. Ans: A
116. Ans: A Exp:
Exp: Payment Bank:
India’s foreign exchange reserves include various z Payments banks were part of the Reserve Bank of
assets that are held by the Reserve Bank of India (RBI) and India’s strategy of offering differentiated banking
denominated in foreign currencies. These reserves provide licenses.
w w w. d r i s h t i I A S.c om PT SPRINT (2024) ECONOMY QUESTIONS AND ANSWERS 51
z A committee headed by Dr. Nachiket Mor 120. Ans: A
recommended setting up of ‘Payments Bank’ to cater Exp:
to the lower income groups and small businesses.
Impact of Rising Inflation on the Indian Economy:
Hence, statement 1 is correct.
z When inflation is high, stock prices are undervalued,
‹ A payments bank is a differentiated bank, offering
and the value of gold increases.
a limited range of products.
z Creditors may lose out, as the value of the money they
z It can accept demand deposits only that is savings and
receive from debtors decreases.
current accounts, not time deposits. Hence, statement
3 is not correct. ‹ Conversely, debtors could benefit by repaying
loans with money that is worth less than when
z Payment banks are restricted to holding a maximum
balance of Rs. 2,00,000 (Rupees Two lakh only) per they borrowed it.
individual customer. z High inflation in one country can lead to a decrease in
‹ Payment Banks cannot accept Non-Resident its international competitiveness.
Indian (NRI) deposits. Hence, statement 3 is not z Inflation can sometimes trigger a cycle of rising wages
correct. and prices i.e., Wage price spiral.
z The Payment Banks cannot set up subsidiaries to Hence, option C is correct.
undertake non-banking financial services activities.
121. Ans: C
119. Ans: A Exp:
Exp: Account Aggregator (AA):
Real Estate (Regulation and Development) Act, 2016: z An AA is a framework that simply facilitates sharing of
z Real Estate Regulatory Authorities (RERAs): financial information in a real-time and data-blind
‹ The Act establishes RERAs in each state, serving manner (Data flow through AA are encrypted) between
as regulatory bodies and dispute resolution regulated entities (Banks and NBFCs). Hence,
forums. statement 1 is correct.
z Mandatory Registration: z The RBI (Reserve Bank of India) in 2016 approved AA
‹ All real estate projects with a minimum plot size as a new class of NBFC (Non Banking Financial
of 500 sq.mt or eight apartments must be Companies), whose primary responsibility is to
registered with RERAs before launch. This aims facilitate the transfer of user’s financial data with their
to enhance transparency in project marketing and explicit consent.
execution. Hence, statement 1 is not correct. z AAs enable flow of data between Financial
z Transparency and Database: Information Providers (FIPs) and Financial Information
‹ RERAs maintain a public database of registered Users (FIUs). Hence, statement 2 is correct.
projects on their websites. This includes project z The architecture of AA is based on the Data
details, registration status, and ongoing progress, Empowerment and Protection Architecture (DEPA)
providing transparency to buyers. framework.
z Funds Management: ‹ DEPA is an architecture that lets users securely
‹ Promoters are required to deposit 70% of the access their data and share the same with third
collected funds in a separate escrow account for parties.
the specific project’s construction and land costs,
122. Ans: D
preventing fund diversion. Hence, statement 2 is
correct. Exp:
z Time-Bound Adjudication: Project Worldcoin:
‹ Appellate Tribunals are mandated to adjudicate z About:
cases within 60 days, while Regulatory ‹ Worldcoin is an initiative to create a digital
Authorities must resolve complaints in the same network in which everyone can claim some kind
timeframe, ensuring faster dispute resolution. of stake, and join the digital economy. Hence,
Hence, statement 3 is not correct. option D is the correct answer.
52 PT SPRINT (2024) ECONOMY QUESTIONS AND ANSWERS www.d rish ti IAS.co m

z Worldcoin Working Process: z Reduced Operational Costs: The platform addresses


‹ The initiative uses a device called “Orb” to collect operational challenges such as multiple visits and
biometric (iris) data and help participants get a documentation requirements, leading to cost
World ID through the World app. reduction for both lenders and borrowers.
€ With the app, participants can collect a ‹ Hence, statement 3 is not correct.
cryptocurrency called Worldcoin [WLD]. z Efficiency and Scalability: The platform’s streamlined
processes lead to quicker disbursement and
‹ Users need to be willing to scan irises and/or get
scalability, resulting in a more efficient credit
their own irises scanned to make the Worldcoin
ecosystem.
network possible.
‹ Hence, statement 4 is correct.
‹ Those who have their irises scanned and collect a
World ID can use this to claim the WLD crypto,
124. Ans: A
which they may use for transactions (if possible
Exp:
and legal) or hold on to the asset in the hope that
Implications of Excess Liquidity in India
its price might rise.
z Positive Implications:
‹ Worldcoin claims that using biometric
‹ Lower Interest Rates: Excess liquidity can lead to
information to avoid duplication is a valid
lower interest rates in the economy.
method for including everyone in its network.
‹ Stimulating Investment: With lower interest
€ This process is called “proof of personhood”
rates, businesses may find it cheaper to borrow
and makes sure that people do not sign and invest in expanding their operations,
themselves up multiple times in exchange for launching new projects, and creating jobs.
crypto. z Negative Implications:
123. Ans: B ‹ Inflationary Pressure: Excess liquidity can
Exp: contribute to inflationary pressures in the
economy.
The Reserve Bank of India (RBI) has initiated a pilot
‹ Asset Price Bubbles: While excess liquidity can
programme aimed at evaluating the feasibility of a ‘Public
boost asset prices, it can also lead to the formation
Tech Platform for Frictionless Credit’, seeking to facilitate
of speculative bubbles.
seamless and efficient credit delivery by lenders for Credit
‹ Income Inequality: The benefits of excess
Appraisal, and therefore boosting Financial Inclusion in India.
liquidity, such as increased asset prices, might
z Developed by the Reserve Bank Innovation Hub (RBIH), disproportionately benefit the wealthy who
It is an end-to-end digital platform that will have an have more exposure to these assets. This can
open architecture, open Application Programming exacerbate income inequality in the economy.
Interfaces (APIs), and standards to which all banks can z Higher interest rate is not an implication of Excess
connect in a “Plug and Play” Model. Liquidity. Hence, Option A is the correct answer
Benefits and Outcomes:
125. Ans: C
z Enhanced Credit Portfolio Management: The
Exp:
platform’s data consolidation will enable improved
z Corporate Bond: Corporate bonds are debt securities
credit risk assessment and efficient credit portfolio
issued by corporations to raise capital for various
management.
purposes, such as expanding operations, funding
‹ Hence, statement 1 is not correct. projects, or refinancing existing debt. Hence, option 1
z Improved Access to Credit: Access to accurate is correct.
information supports informed and swift credit z Triparty Repo Services: Triparty repo services are
assessments. This expansion of credit availability financial transactions where a third party, often a
benefits borrowers by lowering the cost of capital custodian or clearing agent, acts as an intermediary
access. between the two parties involved in a repurchase
‹ Hence, statement 2 is correct. agreement (repo). Hence, option 2 is correct.
w w w. d r i s h t i I A S.c om PT SPRINT (2024) ECONOMY QUESTIONS AND ANSWERS 53
126. Ans: a € This can be done by using low-carbon energy
Exp: sources such as hydrogen, coal gasification,
or electricity instead of the traditional
Index of Industrial Production
carbon-intensive manufacturing route of
z IIP is an indicator that measures the changes in the coal-fired plants.
volume of production of industrial products during a € It eventually lowers GHG emissions, cuts
given period. cost and improves the quality of steel.
z It is compiled and published monthly by the National
Statistical Office (NSO), Ministry of Statistics and 128. Ans: D
Programme Implementation. Hence, statement 2 is Exp:
not correct. What is CCP?
z Base Year for IIP is 2011-2012. Hence, statement 1 is z About:
not correct. ‹ CCP is a financial institution that acts as an
z About Eight Core Sectors: intermediary between buyers and sellers in
‹ These comprise 40.27% of the weight of items various derivatives and equities markets. CCPs
included in the Index of Industrial Production are structures that help facilitate the clearing and
(IIP). Hence, statement 3 is correct. settlement process in financial markets. Hence,
‹ The eight core sector industries in decreasing statement 1 is correct.
order of their weightage: Refinery Products> ‹ The primary goal of CCPs is to increase efficiency
Electricity> Steel> Coal> Crude Oil> Natural Gas> and stability in financial markets.
Cement> Fertilizers. ‹ CCPs reduce risks associated with counterparty,
127. Ans: D operational, settlement, market, legal, and
default issues. Hence, statement 2 is correct.
Exp:
‹ CCPs act as a counterparty to both buyers and
Steel Production in India
sellers in a trade, collecting money from each
z Production and Consumption: India is currently the party involved and guaranteeing the terms of the
world’s 2nd largest producer of crude steel (as of trade.
2021) and also the 2nd largest consumer of finished
z Functions:
steel in 2021 (preceded in both cases by China).
‹ Clearing and settlement are the two main
Hence, statement 2 is correct.
functions of a CCP.
z Important Steel-Producing Centers in India: Bhilai
€ Clearing involves validating the details of the
(Chhattisgarh), Durgapur (West Bengal), Burnpur
(West Bengal), Jamshedpur (Jharkhand), Rourkela trade and ensuring that both parties have
(Odisha) and Bokaro (Jharkhand). sufficient funds to complete the transaction.
€ Settlement involves the transfer of
z Export: India is a significant exporter of steel products,
with major export destinations including the US, the ownership of the asset or security being
UAE, and Nepal. traded from the seller to the buyer.
z Government Policies: The Government has taken z Regulators in India:
various steps to boost the sector including the ‹ The Reserve Bank of India (RBI) for CCPs clearing
introduction of National Steel Policy 2017 and money market instruments and foreign exchange
allowing 100 per cent Foreign Direct Investment (FDI) derivatives.
in the steel sector under the automatic route. Hence, € A CCP is authorized by the RBI to operate in
statement 3 is correct. India under the Payment and Settlement
z Green Steel: Systems Act, 2007.
‹ The Ministry of Steel seeks to reduce CO2 in steel ‹ The Securities and Exchange Board of India (SEBI)
industry through promotion of Green Steel for CCPs clearing securities and commodity
(manufacturing steel without using fossil fuels). derivatives.
Hence, statement 1 is correct. ‹ Hence, statement 3 is correct.
54 PT SPRINT (2024) ECONOMY QUESTIONS AND ANSWERS www.d rish ti IAS.co m

129. Ans: A 131. Ans: C


Exp: Exp:
Subsidies under WTO: Foreign Contribution Regulation Act (FCRA), 2010:
z Amber Box: z About:
‹ Amber box subsidies are those that can distort ‹ The FCRA was enacted during the Emergency in
international trade by making a country’s 1976 in an atmosphere of apprehension that
products cheaper in comparison to those of other foreign powers were interfering in India’s affairs
countries. Hence, pair 1 is correctly matched. by pumping in funds through independent
€ Examples: Subsidies for inputs such as
organisations.
fertilisers, seeds, electricity, irrigation, and € These concerns had been expressed in

Minimum Support Price (MSP). Parliament as early as in 1969.


z Blue box: ‹ The law sought to regulate foreign donations to
individuals and associations so that they
‹ It is the “amber box with conditions” —
functioned “in a manner consistent with the
conditions, designed to reduce distortion.
values of a sovereign democratic republic”.
‹ Any support that would normally be in the amber
z Objectives:
box is placed in the blue box if it requires farmers
‹ It requires every person or NGO wishing to
to limit production. Hence, pair 2 is not correctly
receive foreign donations to be registered under
matched.
the Act, to open a bank account for the receipt of
z Green Box: the foreign funds and to utilise those funds only
‹ Green Box is domestic support measures that for the purpose for which they have been
don’t cause trade distortion or at most cause received and as stipulated in the Act. Hence,
minimal distortion. Hence, pair 3 is not correctly statement 1 is correct.
matched. ‹ The Act prohibits receipt of foreign funds by
‹ The Green box subsidies are government funded candidates for elections, journalists or
without any price support to crops. newspaper and media broadcast companies,
judges and government servants, members of
130. Ans: A
legislature and political parties or their office-
Exp: bearers, and organisations of a political nature.
z Debt-for-Climate Swaps Hence, statement 2 is correct.
‹ Debt-for-climate swaps can incentivize debtor
132. Ans: C
countries to take meaningful action on climate
Exp:
while reducing their debt burdens.
Competition Commission of India (CCI):
‹ These swaps involve reducing debt in exchange
for policy commitments or spending by debtor z About:
countries. Hence, statement 1 is correct. ‹ It is a statutory body of the Government of India
€ Both official bilateral and commercial debt responsible for enforcing the Competition Act,
can be involved in debt-for-climate swaps. 2002, it was duly constituted in March 2009.
Hence, statement 2 is not correct. ‹ The Monopolies and Restrictive Trade Practices
€ Bilateral debt swaps involve redirecting Act, 1969 (MRTP Act) was repealed and replaced
previously committed debt service payments by the Competition Act, 2002, on the
to official bilateral creditors towards recommendations of the Raghavan committee.
financing mutually agreed projects in areas Hence, statement 1 is correct.
such as climate action. z Composition:
‹ In the past decade, debt-for-climate swaps have ‹ The Commission consists of one Chairperson and
become relatively popular among low- and six Members who shall be appointed by the
middle-income countries. Central Government.
w w w. d r i s h t i I A S.c om PT SPRINT (2024) ECONOMY QUESTIONS AND ANSWERS 55
‹ The commission is a quasi-judicial body which z The yield gains from Bt tapering off and the regulatory
gives opinions to statutory authorities and also regime not permitting new gene technologies, the
deals with other cases. The Chairperson and country has turned from a net exporter to an importer
other Members shall be whole-time Members. of cotton.
Hence, statement 2 is correct. z In recent times, marine products, rice, and sugar have
been the driving forces behind India’s agricultural
133. Ans: C
exports. Hence, statement 1 is not correct.
Exp:
‹ Marine Products: Marine product exports have
Impacts of US Fed Rate Hike on Indian Economy: grown steadily from USD5.02 billion in 2013-14 to
z Capital Flows: A US Fed rate hike can lead to a rise in USD8.08 billion in 2022-23.  
interest rates in the US, which can attract capital ‹ Rice: Rice exports have also gone up during this
flows from other countries. This can lead to a
period, from USD7.79 billion to USD11.14 billion.  
reduction in foreign investment in India, which can
€ It’s been driven by non-basmati rice, which
affect economic growth.
has more than doubled. On the other hand,
z Depreciation of rupee: It can also lead to a
premium priced basmati rice has witnessed a
depreciation of rupee, which can have an impact on
India’s trade balance and current account deficit. decline.
€ Basmati exports are mainly to the Persian
‹ Depreciation of Indian rupee may result in
costlier imports such as crude oil and other Gulf countries and, to some extent, the US
goods. This may bring the imported inflation in and UK. Non-basmati shipments are more
Indian Economy. Hence, 1 is correct. diversified.
z Domestic Borrowing Costs: It can lead to an increase € It’s non-basmati that has made India the
in borrowing costs in India, as investors may choose biggest rice exporter, ahead of Thailand.
to invest in US securities instead of Indian securities. ‹ Sugar: The recent boom in sugar exports has been
This can lead to a reduction in domestic investment the third largest contributor – from a mere USD
and higher borrowing costs for businesses and 810.90 mn in 2017-18 to USD 5.77 bn in 2022-23
individuals. – the sugar exports have grown many folds during
z Stock Market: It can also impact the stock market in these years.  
India. Higher US interest rates can lead to a reduction € India has, in the process, emerged as
in demand for risky assets such as equities, which can the world’s No. 2 exporter after Brazil
lead to a decline in stock prices in India. Hence, 2 is 135. Ans: C
not correct.
Exp:
z External Debt: India’s external debt is mostly
denominated in US Dollars, a US Fed rate hike can Multi Model Logistics Parks (MMLP):
increase the cost of servicing that debt, as the value z About:
of the rupee may fall against the dollar. This can lead
‹ A MMLP is a transportation hub that combines
to an increase in India’s external debt burden and a
different modes of transport in one location to
negative impact on the economy. Hence, 3 is correct.
enable efficient movement of goods.
z Banks: The banking industry gets benefited by the
interest rates rise, as banks re-price their loan ‹ These logistics parks are typically located near
portfolio much quicker than their deposit rates, which major transportation nodes, such as ports,
helps them to increase their net interest margin. airports, and highways.
‹ They are designed to handle a large volume of
134. Ans: D goods, with facilities for warehousing,
Exp: distribution, and value-added services such as
z India’s basket of imported farm produce is less packaging and labeling.
dominated by agricultural products compared to its z Status of MMLP in India:
exports.   ‹ The Cabinet Committee on Economic Affairs
z Among these imports, the most significant is vegetable (CCEA) authorized the Ministry of Road Transport
oils, whose imports have more than doubled in value and Highways (MoRTH) to develop 35 MMLP
terms between 2019-20 and 2022-23.  Hence, under the Bharatmala Pariyojana. Hence,
statement 2 is not correct. statement 1 is not correct.
56 PT SPRINT (2024) ECONOMY QUESTIONS AND ANSWERS www.d rish ti IAS.co m

€ Bangalore, Chennai, Guwahati, and Nagpur z Minerals Security Partnership (MSP)


MMLPs under implementation. ‹ It is an initiative by the United States to bolster
‹ The MMLPs are to be developed under Public critical mineral supply chains.
Private Partnership (PPP) on Design, Build, z Partners:
Finance, Operate and Transfer (DBFOT) mode. ‹ Australia, Canada, Finland, France, Germany,
Hence, statement 2 is not correct. Japan, the Republic of Korea, Sweden, the United
‹ The National Highways and Logistics Kingdom, the United States, and the European
Management (NHLML), a special purpose vehicle Commission. Hence, statement 2 is not correct.
(SPV) fully owned by the National Highways
Authority of India (NHAI), plans to construct the 138. Ans: D
majority of the proposed MMLPs in PPP mode. Exp:
136. Ans; A z A series of recent scams have exposed the
Exp: vulnerabilities of the Aadhaar-enabled Payment
System (AePS) and how cybercriminals are exploiting
z Financial Action Task Force (FATF) is the global money
the loopholes in the system to defraud unsuspecting
laundering and terrorist financing watchdog set up in customers.
1989 out of a G-7 meeting of developed nations in
z Scammers have been using leaked biometric details to
Paris. Hence, statement 1 is not correct.
bypass the need for One Time Passwords (OTPs) and
z After the 9/11 attacks on the US, the FATF in 2001 drain funds from unsuspecting victims. Hence,
expanded its mandate to incorporate efforts to combat statement 1 is correct.
terrorist financing.
z Aadhaar-enabled Payment System (AePS) was taken
z FATF’s added efforts to counter the financing of up by the National Payments Corporation of India
proliferation of Weapons of Mass Destruction (WMD). (NPCI) which is a joint initiative of Reserve Bank of
Hence, statement 2 is correct. India (RBI) and Indian Banks’ Association (IBA). Hence,
137. Ans: A statement 2 is correct.
Exp: 139. Ans: D
Critical Minerals: Exp:
z About: Status of the Fisheries Sector in India:
‹ Critical minerals are elements that are the
z About:
building blocks of essential modern-day
‹ As the third-largest fish producer and the second-
technologies and are at risk of supply chain
largest aquaculture producer globally, India
disruptions.
recognizes the significance of the fisheries and
‹ These minerals are now used everywhere from
aquaculture industry. Hence, statement 1 is
making mobile phones, computers to batteries,
correct.
electric vehicles and green technologies like and
‹ The Indian Blue Revolution has led to a major
wind turbines.
improvement in the fishing and aquaculture
z Major Critical Minerals:
industries. The industries are regarded as sunrise
‹ Graphite, Lithium and Cobalt are used for making sectors and are anticipated to have a big impact
EV batteries. Hence, statement 1 is correct. on the Indian economy. Hence, statement 2 is
‹ Aerospace, communications and defence correct.
industries also rely on several such minerals as ‹ In the recent past, Indian fisheries has witnessed
they are used in manufacturing fighter jets, a paradigm shift from marine dominated fisheries
drones, radio sets and other critical equipment. to inland fisheries, with the latter emerging as a
‹ While Cobalt, Nickel and Lithium are required for major contributor of fish production from 36% in
batteries used in electric vehicles, rare earth the mid-1980 to 70% in the recent past.
minerals are critical, in trace amounts, in ‹ The fish production reached an all-time high of
semiconductors and high-end electronics 16.25 MMT during FY 2021-22 with marine
manufacturing. exports touching Rs. 57,586 Crores.
w w w. d r i s h t i I A S.c om PT SPRINT (2024) ECONOMY QUESTIONS AND ANSWERS 57
z Top Producing States: z Not Eligible:
‹ Andhra Pradesh is the largest producer of fish in ‹ The Scheme is not available to corporations,
India followed by West Bengal. Hence, statement partnership firms, Hindu Undivided Family (HUF),
3 is correct. Trusts etc. Hence, statement 2 is correct.
‹ Though there are no restrictions on the frequency
140. Ans: C
of remittances under LRS, once a remittance is
Exp: made for an amount up to USD 2,50,000 during
Wholesale Price Index the financial year, a resident individual would not
The latest data released by the Ministry of Commerce and be eligible to make any further remittances under
Industry reveals that the Wholesale Price Index (WPI) in this scheme.
India fell to a near three-year low with deflation rate of (-) z Remitted Money can be used for:
0.92% in April, marking its entry into negative territory after ‹ Expenses related to travelling (private or for
33 months. business), medical treatment, study, gifts and
donations, maintenance of close relatives and so
z WPI measures the changes in the prices of goods sold
on.
and traded in bulk by wholesale businesses to other
‹ Investment in shares, debt instruments, and buy
businesses.
immovable properties in the overseas market.
z Published by the Office of Economic Adviser, Ministry
‹ Individuals can also open, maintain and hold
of Commerce and Industry. foreign currency accounts with banks outside
z It is the most widely used inflation indicator in India. India for carrying out transactions permitted
z The base year of All-India WPI has been revised from under the scheme.
2004-05 to 2011-12 in 2017. z Prohibited Transactions:
Difference Between WPI and CPI: ‹ Any purpose specifically prohibited under
Schedule-I (like the purchase of lottery tickets,
z WPI tracks inflation at the producer level and proscribed magazines, etc.) or any item restricted
Consumer Price Index (CPI) captures changes in prices under Schedule II of Foreign Exchange
levels at the consumer level. Management (Current Account Transactions)
‹ Both baskets measure inflationary trends (the Rules, 2000.
movement of price signals) within the broader ‹ Trading in foreign exchange abroad. Hence,
economy, the two indices differ in which statement 3 is correct.
weightages are assigned to food, fuel and ‹ Capital account remittances, directly or indirectly,
manufactured items. to countries identified by the Financial Action
z WPI does not capture changes in the prices of services, Task Force (FATF) as “non- cooperative countries
which CPI does. Hence, statement 1 is correct. and territories”, from time to time.
z In WPI, more weightage is given to manufactured ‹ Remittances directly or indirectly to those
individuals and entities identified as posing a
goods, while in CPI, more weightage is given to food
significant risk of committing acts of terrorism as
items. Hence, statement 2 is correct.
advised separately by the Reserve Bank to the
141. Ans: D banks.
Exp: z Requirements:
Liberalised Remittance Scheme: ‹ It is mandatory for the resident individual to
provide his/her Permanent Account Number
z About:
(PAN) for all transactions under LRS made through
‹ This is the scheme of the Reserve Bank of India, Authorized Persons.
introduced in the year 2004. Hence, statement 1 142. Ans: A
is correct.
Exp:
‹ Under the scheme, all resident individuals,
z The withdrawal of the Rs 2000 notes is part of the RBI’s
including minors, are allowed to freely remit up to Clean Note Policy, which aims to provide the public
USD 2,50,000 per financial year (April – March) with high-quality currency notes and coins with
for any permissible current or capital account improved security features. Hence, statement 1 is
transaction or a combination of both. correct.
58 PT SPRINT (2024) ECONOMY QUESTIONS AND ANSWERS www.d rish ti IAS.co m

z The legal basis for demonetization in India is Section ‹ RBI transfers the surplus, in accordance with
26(2) of the Reserve Bank of India Act, 1934, which Section 47 (Allocation of Surplus Profits) of the
empowers the central government to declare any Reserve Bank of India Act, 1934. Hence,
series of banknotes as ceasing to be legal tender by statement 1 is not correct.
notification in the Official Gazette, on the € A technical Committee of the RBI Board
recommendation of RBI. Hence, statement 2 is headed by Y H Malegam (2013), which
correct. reviewed the adequacy of reserves and
z In India, coins function as limited legal tender. Coins surplus distribution policy, recommended a
with denominations equal to or higher than one higher transfer to the government. By and
rupee can be used as legal tender for amounts up to large, with a few exceptions, the quantum of
one thousand rupees. surplus transfer averages around 0.5% of the
z Additionally, fifty paise (half a rupee) coins can be GDP.
used as legal tender for amounts up to ten rupees.
144. Ans: B
Hence, statement 3 is not correct.
Exp:
143. Ans: C z Online gaming platforms will not be required to
Exp: deduct tax at the source for a player if the net winning
RBI Surplus Transfer does not exceed Rs 100. Hence statement 1 is correct.
The Reserve Bank of India (RBI) has approved a significant z Bonus, referral bonus, and incentives provided by the
transfer of surplus funds to the Union Government, providing online gaming company are considered taxable
a major boost to the fiscal position. deposits. These deposits will be subject to tax under
z The surplus transfer for the accounting year 2022-23 Rule 133 of the Income-tax Act. Hence, statement 2 is
amounts to Rs 87,416 crore, a 188% increase not correct.
compared to the previous year. z The Central Board of Direct Taxes (CBDT) has recently
introduced new tax rules for online gaming platforms
in India. These rules aim to bring clarity and establish
guidelines for Tax Deducted at Source (TDS) on
winnings from online gaming. Hence, statement 3 is
correct.
145. Ans: C
Exp:
IRDAI Vision 2047
The Insurance Regulatory and Development Authority of
India (IRDAI), as part of its Vision Insurance for all’ by 2047,
has allotted states and union territories to every insurer to
increase insurance penetration in India.
z IRDAI is also planning to launch Bima Trinity - Bima
Sugam, Bima Vistar, Bima Vaahaks – in collaboration
with general and life insurance firms to make insurance
activities hassle free. Hence, statement 1 is correct.
What is IRDAI Vision 2047?
z As per the above image, statement 2 is not correct. z Objective:
z Surplus of RBI: ‹ Insurance for All by 2047 aims that every citizen
‹ The difference between RBI’s income and has an appropriate life, health and property
expenditure is Surplus. insurance cover and every enterprise is supported
‹ RBI transfers the surplus to the government after by appropriate insurance solutions.
making provisions for reserves and retained ‹ It also aims to make the Indian insurance sector
earnings. globally attractive. Hence, statement 2 is correct.
w w w. d r i s h t i I A S.c om PT SPRINT (2024) ECONOMY QUESTIONS AND ANSWERS 59
z Pillars: 148. Ans: B
‹ Insurance customers (Policyholders) Exp:
‹ Insurance providers (insurers) z A Non-Banking Financial Company (NBFC) is a
‹ Insurance distributors (intermediaries) company registered under the Companies Act, 1956
z Focus Areas: engaged in the business of loans and advances,
‹ Making available right products to right customers acquisition of shares/stocks/bonds/debentures/
securities issued by Government or local authority or
‹ Creating robust grievance redressal mechanism
other marketable securities of a like nature. Hence,
‹ Facilitating ease of doing business in the insurance
statement 1 is not correct.
sector
z NBFCs do not form part of the payment and settlement
‹ Ensuring the regulatory architecture is aligned system and cannot issue cheques drawn on itself;
with the market dynamics
z Deposit insurance facility of Deposit Insurance and
‹ Boosting innovation Credit Guarantee Corporation is not available to
‹ Competition and distribution efficiencies while depositors of NBFCs, unlike in case of banks. Hence,
mainstreaming technology and moving towards statement 2 is correct.
principle based regulatory regime.
149. Ans: A
Exp:
z Recently, the Solar Energy Corporation of India Limited
(SECI) has been recognized as a Miniratna Category-I
Central Public Sector Enterprise (CPSE) by the Ministry
of New and Renewable Energy (MNRE), Govt. of India.
Hence, statement 1 is correct.
What are the Criteria for Miniratna Status for CPSEs?
z Miniratna Category-I Status: The CPSEs which have
made profit in the last three years continuously, pre-
tax profit is Rs.30 crores or more in at least one of the
three years and have a positive net worth are eligible
146. Ans: B to be considered for grant of Miniratna-I status.
Exp: z Miniratna Category-II Status: The CPSEs which have
made profit for the last three years continuously and
z In March 2023, banks from 18 countries were allowed
have a positive net worth are eligible to be considered
by the RBI to open Special Rupee Vostro Accounts
for grant of Miniratna-II status.
(SRVAs) to settle payments in Indian rupees.
‹ Miniratna CPSEs should have not defaulted in
z It includes: Botswana, Fiji, Germany, Guyana, Israel,
the repayment of loans/interest payment on any
Kenya, Malaysia, Mauritius, Myanmar, New Zealand,
loans due to the Government. Hence, statement
Oman, Russia, Seychelles, Singapore, Sri Lanka, 2 is not correct.
Tanzania, Uganda, and the United Kingdom. Hence
‹ Miniratna CPSEs shall not depend upon
option (b) is correct.
budgetary support or Government guarantees.
147. Ans: D 150. Ans: C
Exp: Exp:
z A trademark is a sign capable of distinguishing the Virtual Digital Assets (VDAs):
goods or services of one enterprise from those of z The government in the Union Budget for 2022-23
other enterprises. Trademarks are protected by introduced new provisions aimed at taxing and
Intellectual Property Rights (IPR). tracking VDAs. Along with the framework for taxation,
z In India, trademarks are governed by the Trademarks the Budget for the first time defined virtual digital
Act 1999, Hence, statement 1 is not correct. assets.
z Trademarks do not expire; the owner must make ‹ It has defined virtual digital assets in the newly
regular use of them in order to receive the protections inserted clause (47A) under Section 2 of the
associated with them. Hence, statement 2 is not Income Tax Act, 1961. Hence, statement 1 is
correct. correct.
60 PT SPRINT (2024) ECONOMY QUESTIONS AND ANSWERS www.d rish ti IAS.co m

z VDA has been defined to mean any information or 153. Ans: A


code or number or token (not being Indian currency or Exp:
any foreign currency), generated through cryptographic
Tea Board of India:
means, with the promise or representation of having
inherent value. Hence, statement 2 is correct. z About:
z VDAs mean cryptocurrencies, DeFi (decentralised ‹ It is a statutory body under the Ministry of
finance) and non-fungible tokens (NFTs). Commerce and Industries that was set up in 1953
z From April 2022, India introduced a 30% income tax for the development of tea industry in India. It
on gains made from cryptocurrencies. started functioning in 1954. Hence, statement 2
z In July 2022, rules regarding 1% tax deducted at source is not correct.
on cryptocurrency came into effect.
z Vision:
151. Ans: A ‹ Its vision and mission is to make the country a
Exp: leading producer of tea across the globe for
z Russia is India’s seventh biggest trading partner, up which it established several programmes and
from 25th position in 2021. schemes.
z The US, China, UAE, Saudi Arabia, Iraq, and Indonesia z Members:
were the six countries that recorded higher volumes
‹ The Board is constituted of 31 members
of trade with India during the first five months of
(including Chairman) drawn from Members of
2022-23. Hence, statement 1 is correct
Parliament, tea producers, tea traders, tea
z India’s Imports from Russia accounted for USD 17.23
Billion, while India’s exports to Moscow were only brokers, consumers, and representatives of
worth USD 992.73 million, resulting in a negative Governments from the principal tea producing
trade balance of USD 16,24 Billion in 2020-21. Hence, states, and trade unions
statement 2 is not correct € The Board is reconstituted every three years.
152. Ans: B z Offices in India:
Exp: ‹ The board has its headquarters located in Kolkata
z Off-Budget liabilities refer to debts taken by state-run and 17 other offices across India.
agencies to finance government programs and z Foreign Offices:
subsidies outside the traditional budget. Hence, ‹ Currently Tea Board has two overseas offices
statement 1 is correct. located at Dubai, and Moscow.
z The Indian government has ended its practice of off-
z Production:
budget borrowings through state-run agencies in FY22,
but some off-budget liabilities remain outstanding. ‹ India is the second-largest producer of tea
z The outstanding off-budget liabilities of the Centre globally.
include approximately Rs 6.7 trillion as of end-FY21. z Tea Export:
z Bondholders of off-budget liabilities are unwilling to ‹ India is among the top 5 tea exporters in the
accept prepayment offers from the government world. Hence, statement 1 is correct.
without demanding a premium or higher interest rate 154 Ans: (a) 1 only
to recoup their loss of interest income. Hence,
statement 2 is not correct. Exp:
z The investors are worried that they won’t find other z The Nifty Reits & InvITs Index launched by National
secure and highly rated bonds with similar attractive Stock Exchange Indices Ltd aims to track the
coupon rates to invest in if they accept the prepayment performance of Reits and InvITs that are publicly listed
offer. and traded on the NSE. Hence statement 1 is correct.
z Furthermore, bondholders usually demand a z The index has a base date of 1 July 2019 and a base
premium or a higher interest rate than promised to
value of 1,000.
them to recoup their loss of interest income in the
residual period of the bonds, in case an issuer wants to z The index will be reviewed and rebalanced on a
prepay. quarterly basis. Hence statement 2 is not correct.
w w w. d r i s h t i I A S.c om PT SPRINT (2024) ECONOMY QUESTIONS AND ANSWERS 61
155. Ans: B 157. Ans: B
Exp: Exp:
z An InvITs is a collective Investment Scheme similar to z GI Protection in India:
a mutual fund, which enables direct investment of z India, as a member of the World Trade Organization
money from individual and institutional investors in (WTO), enacted the Geographical Indications of Goods
infrastructure projects to earn a small portion of the (Registration & Protection) Act, 1999 which came into
income as a return. force with effect from 2003.
z InvITs are regulated by the Securities and Exchange € The Act provides registration and also
Board of India (SEBI) (Infrastructure Investment protection of GI goods in India.
Trusts) Regulations, 2014. Hence, statement 1 is not € This Act is administered by the Controller
correct. General of Patents, Designs, and
z InvITs are listed on exchanges just like stocks — through Trademarks, who is also the Registrar of
Initial Public Offering (IPO). Hence, statement 2 is Geographical Indications.
correct. z The Geographical Indications Registry for India is
156. Ans: C located in Chennai.
Exp: z The registration of a geographical indication is valid for
a period of 10 years. It can be renewed from time to
z The Indian Pharmaceuticals industry plays a prominent
time for a further period of 10 years each.
role globally. India ranks 3rd worldwide for production
z Hence, option B is correct.
by volume and 14th by value.
z The nation is the largest provider of generic medicines 158. Ans: B
globally, occupying a 20% share in global supply by Exp:
volume, and is the leading vaccine manufacturer z About QCVM:
globally. Hence, statement 1 is correct.
‹ QCVM is a cashless coin dispensation machine
z India is home to more than 3,000 pharma companies which would dispense coins against debit to the
with a strong network of over 10,500 manufacturing customer’s bank account using UPI (Unified
facilities as well as a highly skilled resource pool. Payment Interface). Hence, statement 1 is not
z According to WTO, Generic copies of drugs are legal correct.
from the patent point of view when they are made z Customers would be endowed with the option of
after the patent has expired or under voluntary or withdrawing coins in required quantities and
compulsory licence — but pirated and counterfeit denominations.
products are by definition illegal. Hence, statement 3
z It will ease accessibility to coins.
is correct.
‹ Unlike a cash-based traditional Coin Vending
z The Drugs Controller General of India (DCGI) is the
Machine, the QCVM would eliminate the need
head of the Central Drugs Standard Control
for physical tendering of banknotes and their
Organization (CDSCO) under the Ministry of Health and
authentication. Hence, statement 1 is correct.
Family Welfare.
z The DCGI is also responsible for setting standards for 159. Ans: D
manufacturing, sales, import, and distribution of drugs Exp:
and medical devices in India, as well as ensuring Foreign Contribution Regulation Act (FCRA):
uniformity in the enforcement of the Drugs and
Cosmetics Act,1940. z FCRA was enacted during the Emergency in 1976 amid
z In addition to these responsibilities, the DCGI acts as apprehensions that foreign powers were interfering in
an appellate authority in case of disputes regarding India’s affairs by pumping money into the country
the quality of drugs and prepares and maintains the through independent organisations.
national reference standard for drugs. Furthermore, z The law sought to regulate foreign donations to
DCGI is the central licensing authority for medical individuals and associations so that they functioned in
devices that fall under the Medical Device Rules 2017. a manner consistent with the values of a sovereign
Hence, statement 2 is correct. democratic republic.
62 PT SPRINT (2024) ECONOMY QUESTIONS AND ANSWERS www.d rish ti IAS.co m

z The FCRA requires every person or NGO seeking to 161. Ans: C


receive foreign donations to be: Exp:
z registered under the Act z Virtual Digital Assets service providers/businesses
z to open a bank account for the receipt of the foreign have now become the ‘Reporting Entities’ under
funds in State Bank of India, Delhi. Hence, statement PMLA Act 2002, and they must follow similar reporting
1 is not correct. standards and KYC norms as the other regulated
entities like banks, securities intermediaries, payment
z to utilize those funds only for the purpose for which system operators, etc. Hence, statement 1 is correct.
they have been received and as stipulated in the Act. z PMLA-covered activities include: Exchange of Virtual
z FCRA registration is valid for 5 years, and NGOs are digital assets (VDA) and fiat currencies, Change
expected to apply for renewal within six months of between one or more VDA types, etc.
the date of expiry of registration. Hence, statement 2 Hence, statement 2 is correct.
is not correct.
162. Ans: B
z The government can also cancel the FCRA registration
Exp:
of any NGO if it finds that the NGO is in violation of the
z The transmission sector in India is regulated by the
Act, if it has not been engaged in any reasonable
Ministry of Power. Hence Statement 1 is not correct.
activity in its chosen field for the benefit of society for
z As of 31st October 2022, India is the third-largest
two consecutive years, or if it has become defunct.
producer and consumer of electricity worldwide, with
z Once the registration of an NGO is cancelled, it is not an installed power capacity of 408.71 GW. Hence
eligible for re-registration for three years. Statement 2 is correct.
160. Ans: D 163. Ans: d
Exp:
Exp:
z The term “bank run” refers to a circumstance in which
z SWAMIH (Special Window for Affordable and Mid-
banks run out of funds due to depositors withdrawing
Income Housing) Investment Fund is India’s largest all of their money for fear of losing it. Following are
social impact fund specifically formed for completing the ways commonly used to prevent or mitigate bank
stressed and stalled residential projects. runs:
z SWAMIH is a government backed fund, set up as a ‹ To increase cash availability, banks may sell assets.
Category-II AIF (Alternate Investment Fund) debt fund ‹ Banks may impose withdrawal limitations or
registered with SEBI (Securities and Exchange Board prohibit withdrawals entirely.
of India), launched in 2019. Hence, statement 1 is not ‹ Banks may borrow money from other banks,
correct. financial institutions, or the central bank to
‹ The Fund is sponsored by the Ministry of Finance prevent bankruptcy. Hence, point 3 is correct.
and is managed by SBICAP Ventures Ltd. (a State ‹ If a bank run is imminent, banks may temporarily
close to prevent further withdrawals. Hence,
Bank Group company).
point 2 is correct.
z Eligibility Criteria:
‹ Deposit insurance provides a safety net for
‹ The real estate projects must be Real Estate depositors in case of bank failure and can limit the
(Regulation and Development) Act (RERA)- catastrophic impact of bank runs. Hence, point 1
registered which have been stalled due to a lack is correct.
of adequate funds.
164. Ans: A
€ Each of these projects must be very close to Exp:
completion.
z About:
‹ They must also fall under the ‘Affordable and ‹ Critical minerals are elements that are the
Middle-Income Project’ category (any housing building blocks of essential modern-day
projects wherein housing units do not exceed 200 technologies and are at risk of supply chain
sq.m.). Hence, statement 2 is not correct. disruptions. Hence, statement 1 is correct.
w w w. d r i s h t i I A S.c om PT SPRINT (2024) ECONOMY QUESTIONS AND ANSWERS 63
‹ Examples: Copper, lithium, nickel, cobalt, and done every day, and 8 billion UPI (Unified
rare earth elements are critical components in Payment Interface) transactions are made every
many of today’s rapidly expanding clean energy month. Hence, statement 3 is correct.
technologies, ranging from wind turbines and € India has sent upwards of ~28 lakh crore
power grids to electric vehicles. As the transition directly to its citizens through direct benefit
to clean energy accelerates, demand for these transfers.
minerals will skyrocket. Hence, statement 2 is not
correct. 167. Ans: C
€ Indian Policy: India’s Department of Science Exp:
and Technology, in collaboration with the z The evergreening of patents is a practice of tweaking
Council on Energy, Environment and Water, drugs in order to extend their patent term and thus
drafted the Critical Minerals Strategy for their profitability. Hence, statement 1 is correct.
India in 2016, with a focus on India’s resource z The Indian Patents Act 1970 introduced many
requirements till 2030. provisions to prevent the mischievous practice of
€ The Indian Critical Minerals Strategy has “evergreening” of patents. Hence, statement 2 is not
identified 49 minerals that will be vital for correct.
India’s future economic growth. z This is to aid millions of people who can’t afford the
expensive modified drugs, as well as the development
165. Ans: D
of the domestic generic drug market. Hence,
Exp: statement 3 is not correct.
z About TIES Scheme:
168. Ans: D
z The TIES scheme provides grants-in-aid to central/
state government-owned agencies or their joint Expl:
ventures for infrastructure projects with significant z IMF Bailouts: Countries seek help from the IMF
export linkages. Hence, statement 1 is not correct. usually when their economies face a major
z The infrastructure includes Border Haats, Land macroeconomic risk, mostly currency crisis (such as
customs stations, quality testing and certification labs, the ones Sri Lanka is facing). Hence, statement 1 is
cold chains, trade promotion centres, export not correct.
warehousing and packaging, Special Economic Zones, z Countries seek such assistance from the IMF to meet
and ports/airports cargo terminuses. Hence, their external debt and other obligations, to purchase
statement 2 is not correct. essential imports, and to prop up the exchange value
166. Ans: C of their currencies.
Exp: z The IMF lends money to the troubled economies often
z About:
in the form of Special Drawing Rights (SDRs). Hence,
statement 2 is not correct.
‹ India is the 2nd largest telecom market globally
with 1.2 billion digital subscribers. Hence, z SDRs simply represent a basket of five currencies,
statement 1 is not correct. namely the U.S. dollar, the euro, the Chinese yuan, the
Japanese yen, and the British pound.
‹ The past nine years witnessed an unprecedented
digital leap, whereby India’s digital economy z This lending is carried out by a number of lending
grew 2.5 times faster than the national economy. programs such as extended credit facility, flexible
Hence, statement 2 is not correct. credit lines, stand-by agreements, etc.
€ Over this period, the number of broadband
z Countries receiving the bailout can use the SDRs for
users rose to 800 million, from 60 million, various purposes depending on their individual
and the number of internet connections rose circumstances.
to 850 million from 250 million. The 169. Ans: C
government and private sector together Expl:
have laid over 2.5 million km of Optical Fiber.
z The Bretton Woods conference in 1944 established the
‹ India is the most connected democracy in the International Monetary Fund (IMF) and the
world, where 70 million e-authentications are International Bank for Reconstruction and
64 PT SPRINT (2024) ECONOMY QUESTIONS AND ANSWERS www.d rish ti IAS.co m

Development (IBRD) to regulate the international the benefits of DPIs and to empower the citizen
monetary and financial order after World War II. The with a real ability to control data. Hence,
IMF’s main objective is to promote global monetary statement 1 is correct.
stability. Hence, statement 1 is correct. ‹ These three sets become the foundation for
z The International Monetary Fund (IMF) is an developing an effective DPI ecosystem.
organization of 190 countries (not universal ‹ Each DPI layer fills a clear need and generates
membership), working to foster global monetary considerable value across sectors.
cooperation, secure financial stability, facilitate ‹ However, like in the case of physical infrastructure,
international trade, promote high employment and it is important that DPIs not succumb to
sustainable economic growth, and reduce poverty monopolization, authoritarianism and digital
around the world. Hence, statement 2 is not correct. colonization. Hence, statement 2 is correct.
z IMF Members: Any other state, whether or not a ‹ India, through India Stack, became the first
member of the UN, may become a member of the country to develop all three foundational DPIs,
IMF in accordance with IMF Articles of Agreement Digital identity (Aadhar), Real-time fast payment
and terms prescribed by the Board of Governors. (UPI) and Account Aggregator built on the Data
‹ Membership in the IMF is a prerequisite to Empowerment Protection Architecture (DEPA).
membership in the IBRD. Hence, statement 3 is Hence, statement 3 is correct.
correct. € DEPA creates a digital framework that allows
‹ Pay a quota subscription: On joining the IMF, each users to share their data on their own terms
member country contributes a certain sum of through a third-party entity, who are known
money, called a quota subscription, which is as Consent Mangers.
based on the country’s wealth and economic 172. Ans: B
performance (Quota Formula). Exp:
170. Ans: A z Assamese Gamocha is a traditional handwoven cotton
towel, which is an integral part of Assamese culture
Exp:
and tradition.
z The Government implemented the IBC, 2016 to
‹ It is a rectangular piece of cloth. The towel comes
consolidate all laws related to insolvency and
in various colors and designs, and the most
bankruptcy and to tackle Non-Performing Assets (NPA),
popular among them are the red and white ones
a problem that has been pulling the Indian economy
with a Phulam known as the ‘Gamocha design’.
down for years.
Hence, statement 1 is not correct.
z The IBC Covers all individuals, companies, Limited
z The Assamese Gamocha was granted the Geographical
Liability Partnerships (LLPs) and partnership firms. Indication (GI) tag. And it has gained national and
Hence, statement 1 is correct. international recognition for its unique design and
z Adjudicating authority: cultural significance. Hence, statement 2 is correct.
‹ National Company Law Tribunal (NCLT) for
173. Ans: D
companies and LLPs.
‹ Debt Recovery Tribunal (DRT) for individuals and Exp:
partnership firms. Hence, statement 2 is not z About:
correct. € NPCI is an umbrella organisation for all retail
payments systems in India. Its primary
171. Ans: D objective is to provide safe, secure, and
Exp: efficient retail payment systems in India.
z About DPI: z It aims to promote digital payments and financial
‹ DPIs mediate the flow of people, money and inclusion in the country.
information. First, the flow of people through a z Establishment:
digital ID System. Second, the flow of money € NPCI was established in 2008 under the
through a real-time fast payment system. And guidance and support of Reserve Bank of
third, the flow of personal information through a India (RBI) and Indian Banks’ Association
consent-based data sharing system to actualize (IBA).
w w w. d r i s h t i I A S.c om PT SPRINT (2024) ECONOMY QUESTIONS AND ANSWERS 65
z Ownership: 176. Ans: D
NPCI is a not-for-profit company, and its
€ Exp:
ownership is shared by a consortium of z About India’s Aviation Sector:
major banks in India. z India’s Civil Aviation is among the fastest-growing
z Products and Services: aviation markets globally and will be a major growth
€ NPCI offers a range of products and services, engine to make India a USD 5 trillion economy by
including Unified Payments Interface (UPI), 2024.
National Electronic Funds Transfer (NEFT), z India is currently the world’s 3rd-largest civil aviation
Immediate Payment Service (IMPS), Bharat market. Hence, statement 1 is not correct.
Bill Payment System (BBPS), RuPay card, z Over the past 6 years, India’s domestic passenger
and others. However, it is not responsible traffic has grown at a compound annual growth rate
for regulating payment and settlement (CAGR) of around 14.5% and international passenger
systems in India. That is the responsibility of traffic at around 6.5%. Hence, statement 2 is not
the RBI. Hence, Option D is correct. correct.
174. Ans: D z India’s domestic passenger traffic is projected to rise to
Exp: 16 crores in the 2023-24 fiscal year and to 35 crores by
2029-30.
z Fund of Funds for Startups (FFS) Scheme:
z The Government has established FFS with a corpus of 177. Ans: C
Rs. 10,000 crore, to meet the funding needs of startups. Expl:
z Department for Promotion of Industry and Internal z IPEF (Indo-Pacific Economic Framework) is a US-led
Trade (DPIIT) is the monitoring agency and Small initiative that aims to strengthen economic partnership
Industries Development Bank of India (SIDBI) is the among participating countries to enhance resilience,
operating agency for FFS. Hence, statement 1 is not sustainability, inclusiveness, economic growth,
correct. fairness, and competitiveness in the Indo-Pacific
z It has not only made capital available for startups at region. Hence, statement 1 is not correct.
early stage, seed stage and growth stage but also z The IPEF was launched in 2021 with a dozen initial
played a catalytic role in facilitating raising of domestic partners who together represent 40% of the world
capital, reducing dependence on foreign capital and GDP.
encouraging home grown and new venture capital z The IPEF is not a Free Trade Agreement (FTA) but allows
funds. Hence, statement 2 is not correct. members to negotiate the parts they want to. The
negotiations will be along four main “pillars”. Hence,
175. Ans: B statement 2 is not correct.
Exp: ‹ Supply-chain resilience
z The report “Falling Long-Term Growth Prospects: ‹ Clean energy, decarbonisation & infrastructure
Trends, Expectations, and Policies” (2020-2030) is ‹ Taxation & anti-corruption
released by World Bank. ‹ Fair & resilient trade.
‹ The report uses a comprehensive database of 178. Ans: C
multiple measures of potential growth. It Exp:
examines trends in potential growth and its
z About Hallmarking:
drivers, global and regional prospects for
z Hallmarking is the accurate determination and official
potential growth and investment over the 2020s,
recording of the proportionate content of precious
and a range of policy options to lift potential metal in precious metal articles. Hence, statement 1 is
growth. Hence, statement 1 is not correct. correct.
z According to the report although India has also lost its z Hallmarking scheme for Jewelry was started by BIS in
growth momentum over the past two decades, it is the year 2000. In India, at present two precious metals
and will likely remain a global leader when it comes to namely gold and silver have been brought under the
growth rates. Hence, statement 2 is correct. purview of Hallmarking.
66 PT SPRINT (2024) ECONOMY QUESTIONS AND ANSWERS www.d rish ti IAS.co m

z However, Mandatory Hallmarking has been € In all other cases, only institutional investors
successfully implemented in 288 districts of the and non-institutional investors can invest in
country with effect from 23 June 2021 by Hallmarking securities issued by SEs.
of Gold Jewellery and Gold Artefacts Order, 2022. ‹ Eligibility:
z After introduction of 6-digit HUID in 2021, hallmark € Any non-profit organisation (NPO) or for-

consisted of 3 marks viz, BIS logo, purity of the article profit social enterprise (FPSEs) that
establishes the primacy of social intent
and six-digit alphanumeric HUID. Each hallmarked
would be recognised as a SE, which will make
article has unique HUID number which is traceable.
it eligible to be registered or listed on the
Hence, statement 2 is correct. SSE.
179. Ans: A € 17 plausible criteria under SEBI’s ICDR

Exp: Regulations, 2018 include serving to


eradicate hunger, poverty, malnutrition,
z About Social Stock Exchange (SSE): promoting education, employability,
‹ The SSE would function as a separate segment equality, and environmental sustainability
within the existing stock exchange and help among others
social enterprises raise funds from the public ‹ Ineligibility:
through its mechanism. € Corporate foundations, political or religious
‹ It would serve as a medium for enterprises to organisations, professional or trade
seek finance for their social initiatives, acquire associations, infrastructure and housing
visibility and provide increased transparency companies (except affordable housing)
would not be identified as SE. Hence,
about fund mobilisation and utilisation. Hence,
statement 2 is not correct.
statement 1 is correct.
€ NPOs would be deemed ineligible if
€ Retail investors can only invest in securities dependent on corporates for more than
offered by for-profit social enterprises (SEs) 50% of its funding.
under the Main Board.

You might also like